Você está na página 1de 166

Coino Resolver Probleinas

Mateináticos
Uma Perspectiva Pessoal
Terence Tao

■!SBM
COLEÇÃO DO PROFESSOR DE MATEMÁTICA
E scrito por um destacado matemático, esse estimulante e
claramente apresentado texto conduz o leitor através de várias
táticas para a resolução de Problemas Matemáticos, em nível de
Olimpíadas. Cobrindo teoria de números, álgebra, análise,
geometria euclidiana e geometria analítica, éomo Resàlver
Problemas Matemáticos inclui ao longo de seu corpo numerosos
exercícios e modelos de solução. Tendo como pré-requisito apenas
conhecimentos de matemática básica do ensino médio, o texto é
ideal para um amplo público e para estudantes com idade a partir
dos 14 anos, interessados em matemática pura.

ISBN 9 78 - 85-858 1 8-94 -4

■!ISBM 97..JJL >


Como resolver· problemas matemaucos - uma pen,pecuvn 11 .,.,.,.,n,
Copyright © 2013 Terence Tao
Direitos cedidos para Sociedade Brasileira de Matemática, SBM para esta edição. A reprodução
não autorizada desta publicação, no todo ou em parte, constitui violação de direitos autorais.
(Lei 9.610/98).
Solving Mathematical Problems: A Personal Perspective, First Edition was originally published
in English in 2006. This tanslation is published by arrangement with Oxford University Press.
A primeira edição do livro " Como resolver problemas matemáticos - Uma perspectiva pessoal,
foi originalmente publicada em inglês em 2006. Essa tradução é publicada por acordo com a
Oxford University Press.
Tradução
Paulo Ventura
Sociedade Brasileira de Matemática
Presidente: Paolo Piccione
Vice-Presidente: Jaqueline Godoy Mesquita
Diretores:
Walcy Santos
Jorge Herbert Soares de Lira
Daniel Gonçalves
Roberto Imbuzeiro
Editor Executivo
Ronaldo Alves Garcia
Assessor Editorial
Tiago Costa Rocha
Coleção Professor de Matemática
Comitê Editorial ·
Bernardo Lima
Djairo de Figueiredo
Ronaldo Garcia (Editor- Chefe)
José Espinar
José Cuminato
Sílvia Lopes
Capa
Pablo Diego Regino
Distribuição e vendas
Sociedade Brasileira de Matemática
Estrada Dona Castorina, 110 Sala 109 - Jardim Botânico 22460-320 Rio de Janeiro RJ
Telefones: (21) 2529-5073 / 2529-5095
http://www.sbm.org.br / email:lojavirtual@sbm.org.br
ISBN 978-85-85818-94-4
FICHA CAT4LOGRÁFICA PREPARADA PELA SEÇÃO DE TRATAMENTO DA
INFORMAÇAO DA BIBLIOTECA PROFESSOR ACHILLE BASSI - ICMC/USP

Tao, Terence
Tl 71c Como resolver problemas matemáticos - Uma perspectivà
pessoal/ Terence Tao; tradução de Paulo Ventura. - Rio de
Janeiro: SBM, 2013.
168 p. (Coleção do Professor de Matemática; 31)
ISBN 978-85-85818-94-4
1. Estratégias de resolução de problemas. 2. Teoria
qos números - ejtE:mplos. 3. Análise - exemplos. 4.
Algebra - análise. I. Verrtura, Paulo, trad. II. Título.
Com.o Hesolver Problem.as
Matem.áticos
Uma Perspectiva Pessoal
Terence Tao

1' edição
2013
;ltio de Janeiro

■!SBM
COLEÇÃO DO PROFESSOR DE MATEMÁT'ICA
■!SBM
SOCEDADE BRASLEIRA DE MATEMÁTEA

COLEÇÃO DO PROFESSOR DE MATEMÁTICA


Logaritmos - E. L. Lima
Análise Combinatória e Probabilidade com as soluções dos exercícios -
A. C. Morgado, J. B. Pitombeira, P. C. P. Carvalho e P. Fernandez
Medida e Forma em Geometria (Comprimento, Área, Volume e Semelhança) -
E. L. Lima
Meu Professor de Matemática e outras Histórias - E. L. Lima
Coordenadas no Plano com as soluções dos exercícios - E. L. Lima com a
colaboração de P. C. P. Carvalho
Trigonometria, Números Complexos - M. P. do Carmo, A. C. Morgado e E. Wagner,
Notas Históricas de J. B. Pitombeira
Coordenadas no Espaço - E. L. Lima
Progressões e Matemática Financeira - A. C. Morgado, E. Wagner e S. C. Zani
Construções Geométricas - E. Wagner com a colaboração de J. P. Q. Carneiro
Introdução à Geometria Espacial - P. C. P. Carvalho
Geometria Euclidiana Plana - J. L. M. Barbosa
Isometrias - E. L. Lima
A Matemática do Ensino Médio Vol. 1 - E. L. Lima, P. C. P. Carvalho, E. Wagner e
A. C. Morgado
A Matemática do Ensino Médio Vol. 2 - E: L. Lima, P. C. P. Carvalho, E. Wagner e
A. C. Morgado .
A Matemática do Ensino Médio Vol. 3 - E. L. Lima, P. C. P. Carvalho, E. Wagner e
A. C. Morgado
Matemática e Ensino - E. L. Lima
Temas e Problemas - E. L. Lima, P. C. P. Carválho, E. Wagner e A. C. Morgado
Episódios da História Antiga da Matemática - A. Aaboe
Exame de Textos: Análise de livros de Matemática - E. L. Lima
A Matemática do Ensino Medio Vol. 4 - Exercícios e Soluções - E. L. Lima, P. C. P.
Carvalho, E. Wagner e A. C. Morgado
Construções Geométricas: Exercícios e Soluções - S. Lima Netto
Um Convite à Matemática - D.C de Morais Filho
Tópicos de Matem_ática Elementar - Volume 1 - Números Reais - A. Caminha
Tópicos de Matemática Elementar - Volume 2 - Geometria Euclidiana Plana - A.
Caminha
Tópicos de Matemática Elementar - Volume 3 - Introdução à Análise - A. Caminha
'/iipicos de Matemática Elementar - Volume 4 - Combinatóri<L - A. Caminha
'/iii,i1·<1s rfo Matemática Elementar·- Volume 5 - Teoria dos N,í.meros - A. <.'aminha
'fi;1 1w11r< ,/,· Matemática Elementar - Voluml' (;. l'o/i111i111iri.~ • A ('11111inl,11
1'1 ,·,,· \'1111w11H pt'lo Mu.náo da M<1temcítica - <:. Crirn·i11 ,/,· 811 , .• /. /(0,./111 (l'í/i/1Jres)
Sumário

Prefácio Vll

Prefácio da primeira edição xiii

Prefácio da segunda edição xvii

1 Estratégias de resolução de problemas 1

2 Exemplos da teoria dos números 13


2.1 Algarismos . . . . . . 16
2.2 Equações diofantinas 29
2.3 Somas de potências . 33

3 E~emplos da álgeb;ra e da análise 49


3.1 Análise de funções 50
3.2 Polinômios . . . . . 58

4 Geometria euclidiana 67

V
Vl SUMÁRIO

5 Geometria analítica 95

6 Exemplos variados 117

Referências 139

Índice Remissivo 141


Prefácio

Em 22 de Agosto de 2006, em Madri, o Rei de Espanha entregou a


medalha Fields ao matemático australiano Terence Tao, no primeiro
dia do 25º Congresso Internacional de Matemática. Para a maioria
dos leitores este será um prêmio obscuro. Outros te:r:ão ~mvido di-
zer que é algo como o prêmio Nobel da matemática. Na verdade, a
medalha Fields é muito mais difícil de obter do que o prêmio Nobel.
Em primeiro lugar, é atri~mída nos Congressos Internacionais de Ma-
temática, e estes realizam-se apenas de quatro em quatro anos. Em
segundo lugar, só pode ser candidato à medalha Fields num Congresso
quem não tiver completado 40 anos até ? fim do ano anterior ao da
realização do Congresso. Ou seja: a medalha Fields, destinada a reco-
nhecer trabalhos matemáticos excepcionais, é um prêmio para pessoas
relativamente jovens. Por exemplo, Andrew Wiles, q~e nos anos 90
demonstrou o Último Teorema de Fermat, recebeu muitos prêmios,
mas não a medalha Field~, por causa da idade.
Os premiados com a medalha Fields são escolhidos por comissões
nomeadas pela União Matemática Internacional. Os nomes dos mem-
bros dessas comissões, desde que a medalha foi atribuída pela primeira

vii
viii Prefácio

vez, formam uma lista que é um verdadeiro who's who da matemática


mundial no século XX.
As primeiras duas medalhas Fields foram entregues em 1936, no
Congresso de Oslo. As duas seguintes em 1950, em Paris. A partir
de 1966, o número máximo de medalhas a atribuir em cada Congresso
subiu para quatro e, desde então, o número de premiados tem variado
entre dois e quatro (o total é de 48 em 70 anos). Em Madri foram
quatro: além de Terence Tao, Andrei Okounkov, Wendelin Werner e
Grigori Perelman. Na altura, as mídias de todo o mundo encheram-se
de notícias sobre um dos medalhados, Grigori Perelman, não tanto
pelas contribuições científicas como pelas suas características pessoais,
algo excêntricas, que culminaram na recusa da medalha.
Nas semanas anteriores ao Congresso de Madri, Perelman era uma
aposta óbvia para.a medalha, por ter provado a famosa conjectura de
Poincaré. Mas também a medalha de Tao era !Ilais do que esperada,
pelos resultados espetaculares em várias áreas que tinha obtido nos
anos anteriores. A um colega. que - apostando ele próprio em Perel-
man e Tao - me desafiou para uma opinião respondi que seriam dadas
quatro medalhas: uma a Perelman e três a Tao. A razão era sim-
ples: Tao era autor não de um mas de vários trabalhos matemáticos
excepcionais, alguns dos quais resolvendo problemas antigos e difíceis.
A citação oficial que acompanhou a atribuição da sua medalha ex-
plicita algumas dessas contribuições. A primeira, e a mais famosa, é
um trabalho sobre números primos. Há muito tempo que se observou
que a sucessão dos números primos contém progressões aritméticas
- isto é, sequências em que a diferença entre cada número e o se-
guinte é constante - de vários comprimentos. Por exemplo, 3, 5, 7 é
uma progressão aritmét!ca de comprimento três. Outra é 3, 7, 11. É
muito difícil encontrar progressões aritméticas nos primos, e a maior
que atualm~nte se conhece tem comprimento 24. O que Tao provou,
em colaboração com ~en Green, foi que, na sucessão dos 111"u111•ro:-;
primo:,;, exist<~m progressões }1ritrn6ticas de qualquer c·o111p1 í1111•11l11 (n
Prefácio IX

demonstração não é construtiva, isto é, não exibe explicitamente tais


progressões).
A segunda contribuição citada refere-se a trabalhos de Tao sobre
o problema de Kakeya, outra questão famosa que começa com uma
pergunta muito simples: se num plano fizermos uma agulha rodar
180 (continuamente e admitindo translações), qual é a menor área
possível percorrida pela agulha? Este problema está resolvido há 80
anos (faça o leitor algumas experiências, e depois informe-se sol;,re a
solução, que é muito surpreendente). Tao investigou profundamente
a generalização do problema para n dimensões, que tem ligações com
importantes áreas da matemática.
A terceira e a quarta contribuições de Tao mencionadas na citação
oficial são trabalhos mais próximos da Física, respectivamente sobre
relatividade geral e versões não lineares da equação de Schrodinger.
No fim da citação é referido outro trabalho notável de Tao: em
colaboração com Allen Knutson, ele resolveu complet~me~te, usando
técnicas combinatórias, o problema da descrição dos valores próprios
possíveis da soma de duas matrizes simétricas quando se conhecem
os respectivos valores pr?prios. Este trabalho, conjugando resultados
anteriores de Andrei Zelevinsky e Alexander Klyachko, permitiu res-
ponder afirmativamente a uma conjectura que Alfred Horn fizera em
1962.
Com dois colegas de Coimbra, passei bastante tempo, nos anos 90,
a pensar na conjectura de Horn. Tendo feito alguns progressos, e tendo
sabido da importante contribuição de Klyachko, decidimos organizar
um encontro em Coimbra sobre o assunto, no Verão de 1999. Já a or-
ganizàção estava em andamento quando soubemos, em finais de 1998,
dos resultados espetaculares de Knutson e Tao. Logo os convidamos a
vir participar no encontro. Ainda me lembro da mensagem que enviei
a Tao, que começava com Dear Professor Tao. Não sabia então que,
do outro lado do correio eletrônico, em Los Angeles, estavà um jovem:
d<· 2J anos, dmüorado aos.20. Ele acabou por me dizer que não podia
X Prefácio

vir, mas estiveram no encontro Zelevinsky, Klyachko e Knutson, os


outros protagonistas do assalto final à conjectura de Horn.
Tao tem mais trabalhos de grande impacto. Uma investigação cu-
jas consequências poderão um dia chegar às mãos do leitor é a que
realizou, em colaboração com Emmanuel Candés, sobre técnicas de
compressão de imagens ou, mais geralmente, sobre a substituição in-
teligente - com uma nova técnica a que chamaram compressed sensing
- de enormes coleções de dados por conjuntos mais pequenos contendo
o essencial da informação. Uma aplicação possível - em relação à qual
o próprio Tao é um pouco cético - será à concepção de máquinas fo-
tográficas digitais com um processamento das imagens mais eficiente.
Sobre Terence Tao já muito foi escrito, em particular sobre a sua
extraordinária capacidade para resolver problemas difíceis em áreas
muito diversas, ngrmalmente em colaboração com especialistas nessas
áreas. A citação da medalha Fields fala mesmo de um engenho do
outro mundo e de um ponto de vista su~preendentemente natural que
deixa outros matemáticos .a, ·perguntar: por que é que ninguém se
lembrou daquilo antes? A página de Tao na Internet é um prodígio de
criatividade e transmissão de ideias novas, que vale a pena consultar
(incluindo um blogue matemático mantido- com regularidade, tanto.
em posts como em respostas a comentários e perguntas): por alguma
razão já lhe chamaram o Mozart da matemática.
Há muitos clichês sobre os grandes matemáticos e a sua vida. Mas
mesmo quem, como eu, não conhece Tao pessoalmente, facilmente se
apercebe, por entrevistas e testemunhos, de que se trata de uma pessoa
com uma vida normal, consciente dos seus talentos invulgares mas
usando-os naturalmente -- ele próprio gosta de insistir ·que o essencial
em matemática é o trabalho - e relacionando-se com maturidad(• ('
sem excentricidades com o mundo à sua volta.
Só depois do convite frustrado a Tao me apercebi d(• q11,· :>1· l 11tl 11\'/I
da mesma pessoa que fiC:ara famosa muito a11t.1's. ('Ili 1~IS;,.i_ 011 11,1111!1111·
11111a 11wdalha d,~ ouro nas ()li111píadas l11t.(•1'11:wi1111111'., d,· \111!1•111111 li 11
Prefácio Xl

- uma competição extremamente exigente pensada para jovens no fim


do Ensino Secundário com 13 anos de idade e na sua terceira par-
ticipação (em 1986, ainda antes.de completar 11 anos, ganhara uma
medalha de bronze, e em 1987 uma de prata).
Tanto Tao como os seus dois irmãos foram crianças e jovens ex-
cepcionalmente brilhantes e precoces, tendo sido acompanhados pelos
melhores especialistas mundiais nesses casos. Terence, em particular,
teve um percurso escolar delineado com cuidado pelos seus pais (uma
professora de matemática e um pediatra emigrados de Hong Kong para
a Austrália), que lhe permitiu um progresso acelerado na disciplina de
matemática.
Aos 15 anos, já depois das suas três participações nas Olimpíadas
Internacionais de Matemática, escreveu o livro que o leitor tem nas
mãos. Nele coligiu vários problemas de matemática, que organizou
tematicamente em quatro capítulos, mais um com exemplos diversos
(nomeadamente de combinatória). Antes dos quatro capítulos princi-
pais - sobre teoria dos números, álgebra e análise, geómetria euclidi-
ana, e geometria analítica -- há um interessante capítulo sobre Estra-
tégias de resolução de problemas, onde o autor analisa, com exemplos,
vários princípios e regras gerais para abordar e resolver problemas de
matemática: compreender o problema, compreender os dados e o ob-
jetivo, escolher símbolos adequados, escrever o que se sabe, modificar
o problema, ir provando alguma coisa, etc.
Numa entrevista que deu em 2006, Tao afirmou: Quando eu era
criança, tinha uma ideia romântica da matemática, a ideia de que os
problemas difíceis eram resolvidos em momentos 'Eureka' de inspira-
t;ão. Depois, acrescentou': Hoje, comigo, é sempre assim: 'Vamos ten-
tar esta ideia. Isso leva-me a algum progresso, ou então não funciona.
Agora tentemos aquilo. Oh, há aqui um pequeno atalho.' Trabalha-
mos durante tempo suficiente e, a certa altura, conseguimos progredir
1111m problema difícil entrando pela porta das traseiras. ·No final, o
q1w 11orrnalmente acontece é: 'Olha, resolvi o problema.' É este tipo
xii Prefácio

de atitude e de estratégia que está presente logo no primeiro capítulo


do livro.
Os problemas que Tao analisa ao longo desta obra são do tipo dos
que se encontram nas Olimpíadas de Matemática: são elementares no
que se refere ao nível dos conhecimentos matemáticos necessários, mas
exigem reflexão e engenho para a sua resolução. Com grande clareza,
Tao explica como resolver os problemas seleccionados, discute estra-
tégias, exemplifica truques comuns. Depois inclui, como exercícios,
problemas que o leitor pode e deve experimentar por si mesmo.
O público para um livro destes é formado por quaisquer pessoas,
em particular jovens, que gostem de matemática e estejam dispostas
a fazer algum esforço mental. Essas pessoas achá-lo-ão interessante,
útil e formativo.
Esqueça o leitor, que o autor deste livro, tinha 15 anos quando
o escreveu. A idade não é importante para a :,;natemática. Esqueça
também, tudo o que sabe sobre o passado de criança-prodígio do
autor. Os raciocínios podem ser os mesmos para todos. Concentre-se
apenas na matemática.
A excelente tradução de Como Resolver Pr'oblemas Matemáticos
deve-se a Paulo Ventura Araújo, matemático da Universidade do Porto,
que é autor de um bom Curso de Geometria.
Fala-se muito na crise do ensino da matemática em Portugal, mas
de vez em quando convém prestarmos atenção às coisas positivas. En-
tre elas está decerto o fato de muitos jovens portugueses gostarem de
matemática. Para esses jovens, poucos livros serão melhor escolha do
que este. Leiam-no, acompanhem o jovem autor nos seus desafian-
tes problemas, nos seus engenhosos raciocínios, nas suas inesperadas
soluções. Dificilmente p9deriam estar em melhor companhia.

Coimbra, Abril de 200~


Prefácio da primeira edição

Proclus, um antigo filósofo grego, disse:


É isto, pois, a matemática: ela recorda-nos as formas invisíveis da
alma; ela dá vida a suas próprias descobertas; ela despertá a mente e
purifica o intelecto; ela traz à luz nossas ideias escondidas; ela elimina
o esquecimento e a ignorância que são nossos desde o nascimento ...
Mas eu simplesmente gosto da matemática porque ela é divertida.
Os problemas ou puzzles matemáticos são importantes para a ma-
temática a sério (aquela que os matemáticos praticam), do mesmo
modo que os contos, fábulas e historietas são importantes para as
crianças entenderem a vida real. Os problemas matemáticos são ma-
temática desinfetada, em que uma solução elegante foi já encontrada
(por outra pessoa, claro), a questão foi extirpada dê tudo quanto é
supérfluo, e ela nos é apresentada de um modo interessante e (espe-
ra-se) estimulante. Se compararmos a matemática com a busca do·
ouro, resolver um bom problema matemático é semelhante a um curso
do tipo esconde-esconde em prospecção de ouro: fazem-nos procurar
uma certa pepita; conhecemos-lhe o aspecto, sabemos que·está em al-
µ;m11 lugar, que não é difícil chegar a ela, que está ao nosso alcance
xiv Prefácio da primeira edição

descobri-la, e que (muito convenientemente) nos foi fornecido o equi-


pamento certo (ou seja, os dados do problema) para a encontrarmos.
Pode estar escondida em sítio astucioso, mas sua descoberta requererá
habilidade em vez de grandes escavações.
Apresento neste livro resoluções de problemas de diversos ramos da
matemática e com níveis de dificuldade variados. Problemas assina-
lados com uma estrela (*) são de dificuldade maior, seja por exigirem
matemática mais avançada, seja por obrigarem a raciocínios engenho-
sos; e aqueles com duas estrelas (**) são ainda mais difíceis. A alguns
problemas seguem-se exercícios adicionais que podem ser resolvidos de
modo semelhante ou envolvem matemática do mesmo tipo. Enquanto
vou apresentando as resoluções, tento exemplificar alguns truques do
ofício de resolver problemas. Duas das principais armas - experiência
e conhecimento - não são fáceis de pôr num livro; têm que ser adqui-
ridas com o tempo. Mas há muitos outros truques mais simples que
levam menos tempo a aprender. Há modos de encarar um problema
que tornam mais fácil encontrar um plano de ataque praticável. Há
maneiras sistemáticas de reduzir um problema a subproblemas cada
vez mais simples. Mas, por outro lado, resolver o problema não é
tudo. Para voltarmos à analogia da pepita. de ouro, esventrar toda
a região circundante com explosões e escavadoras é mais grosseiro· do
que fazer um levantamento cuidadoso do terreno, usando alguma geo-
logia e abrindo pequenas covas em locais escolhidos. Uma solução deve
ser relativamente curta, compreensível, e se possível ter um toque de
elegância. Deve também ser divertido encontrá-la. Usar a geometria
das coordenadas para transformar um pequeno e simpático problema
geométrico numa equação monstruosa e voraz não tem ·o mesmo gosto
de vitória que uma solução vetorial em duas linhas.
Como exemplo de elegância, eis um resultado bem conhecido <la
geometria euclidiana:
Mostre que as media_trizcs <los lados <lc um triâ.ngulo s;1.o <·1111<·or,
l'('llL<~S.
Prefácio da primeira edição XV

Para este problema de uma linha só poderíamos usar coordenadas.


O leitor pode tentar isso durante alguns minutos (ou horas), e depois
observar esta solução:

Demonstração. Chamemos ABC ao triângulo. Seja P a interseção


da mediatriz de AB com a de AC. Visto que P está na mediatriz de
AB, temos IAPI = IPBl 1 . Visto que P está na mediatriz de AC, temos
IAPI = IPCI. Combinando as duas igualdades, IPBI = IPCI. Mas
então P está na mediatriz de BC, o que significa que as três mediatrizes
são concorrentes. (A propósito, P é o circuncentro de ABC.) ■
O seguinte excerto da figura anterior mostra por que razão IAPI =
IPBI quando P está na mediatriz de ÃB: é só usar congruência de
triângulos.

A ~ B

··-·-··· .
1 N. do T. Denotamos por AB o segmento de reta de extremidades A e B, e por
IAHI o c:0111pri11w11to desse) scgirwnto.
XVI Prefácio da primeira edição

Este tipo de resolução - e o modo singular como uns tantos fatos


óbvios se entrelaçam para dar forma a um fato não assim tão óbvio -
é parte da beleza da matemática. Espero que o leitor também aprecie
essa beleza.
Agradecimentos
Obrigado a Peter O'Halloran, Vem Treilibs e Lenny Ng por con-
tribuírem com problemas e conselhos.
Obrigado a Basil Rennie por suas correções e por suas sugestões
engenhosas para encurtar várias resoluções. E, finalmente, obrigado a
minha família por seu apoio, encorajamento, correções ortográficas, e
por me ter pressionado quando me atrasava neste trabalho.
A maioria dos problemas neste livro vêm de recolhas publicadas
em livro de problemas de várias competições matemáticas. Essas fon-
tes estão indicada& no texto e listadas detalhadamente na bibliografia
final. Usei ainda alguns problemas fornecidos por amigos ou tirados
de várias publicações matemáticas, e des~es não indico as fontes.
Prefácio da segunda edição

Este livro foi escrito há quinze anos; para mim, literalmente, me-
tade da vida. Nos anos entretanto decorridos saí de casa, mudei de
país, fiz estudos de pós-graduação, ensinei, escrevi artigos de pesquisa,
fui orientador de alunos de doutorado, casei com minha mulher, tive-
mos um filho. É claro que minha perspectiva da vida e da matemática
é hoje diferente do que era há quinze anos. Há muito tempo que não
estou envolvido em competições de problemas matemáticos, e se fosse
escrever agora um livro sobre o assunto ele seria muito diferente deste
que o leitor tem em mãos.
A matemática é um assunto multifacetado, e o modo como a expe-
rimentamos e apreciamos vai mudando com o tempo e a experiência.
Como aluno da escola primária, a matemática me atraiu pela beleza
abstrata da manipulação formal, e por sua habilidade notável em usar
repetidamente regras simples para obter respostas não triviais. Como
aluno de liceu e participante de competições matemáticas, a matemá-
tica me dava o prazer de um esporte, em que eram propostos problemas
ou puzzles habilmente formulados (como os deste livro) ·e o objetivo
Pra encontrar o truque certo que revelasse a solução. Como aluno uni-
xvm Prefácio da segunda edição

versitário, me impressionaram os primeiros vislumbres das teorias e


estruturas - ricas, profundas e fascinantes - que formam hoje o núcleo
da matemática moderna. Como estudante de pós-graduação, experi-
mentei o orgulho de ter meu próprio projeto de pesquisa e a satisfação
única que advém de criar um argumento original para resolver uma
questão antes em aberto. Quando iniciei minha carreira como mate-
mático e pesquisador profissional, comecei a ver a intuição e motivação
por trás das teorias e dos problemas da matemática moderna, e fiquei
radiante quando compreendi o modo como, muitas vezes, problemas
muito complexos e profundos são, no fundamental, guiados por prin-
cípios muito simples e até pelo senso comum. A experiência Aha! de
entender um destes princípios, e de repente ver como ele faz luz sobre
um vasto campo da matemática e lhe dá forma, é de fato inesquecível.
E há sempre mais aspectos da matemática por descobrir: só recente-
mente dominei um número suficiente de ramos .da matemática para
começar a apreender a matemática moderna como um esforço unifi-
cado, e para entender sua relação com as ciências e outras disciplinas.
Tendo escrito o livro antes de minha carreira profissional como
matemático, não dispunha ainda de muitas dess·as revelações e expe-
riências, e portanto em muitos pontos a exposição revela uma certa
ingenuidade ou mesmo inocência. Tive alguma relutância em m~xer
muito nisso, até porque esse eu mais jovem estava por certo mais sin-
tonizado com o mundo das competições matemáticas liceais do que
eu estou hoje. Aproveitei, contudo, para fazer algumas mudanças na
organização do livro: formatar o texto em LATEX, rearranjar as ma-
térias por uma ordem que me pareceu mais lógica, e consertar aquelas
partes do texto que estavam incorretas, mal expressas; confusas, ou
sem um rumo claro. Acrescentei ainda alguns exercícios. Em alguns
lugares, o texto está um pouco datado (o último teorema de Fermat,
por exemplo,. foi já rigorosamente demonstrado), e vejo agora qtw vfr-
rios dos problemas no livr<? poderiam ser tratados de modo mai:-; 1i111 po
e rápido com técnicas matemá.ticas mais sofisticada:-;. Pori'•111. n fll 11
Prefácio da segunda edição xix

pósito do texto não é apresentar a solução mais hábil ou fornecer um


panorama atualizado de resultados, mas antes mostrar como se aborda
inicialmente um problema mate.mático, e como o processo exaustivo e
sistemático de experimentar algumas ideias e eliminar outras, manipu-
lando convictamente os dados, pode levar, finalmente, a uma solução
satisfatória.
Agradeço profundamente a Tony Gardiner por encorajar e apoiar
a reedição deste livro, e a meus pais todo o apoio ao longo dos anos.
Sinto-me grato também a todos os amigos e conhecidos que encontrei
ao longo dos anos e que leram a primeira edição do livro. Por último,
mas não menos importante, devo um agradecimento especial a meus
pais e à equipe de apoio informático do Flinders Medical Centre por
terem extraído de nosso venerável Macintosh Plus uma cópia eletrônica
deste livro com quinze anos de idade.

Departamento de Matemática
Universidade da Califónia, Los Angeles
Dezemb.ro de 2005

Terence Tao
XX Prefácio da segunda edição
CAPÍTULO 1

Estratégias de resolução de problemas

A viagem de mil quilômetros começa com um passo.


Lao Tzu

A semelhança (ou não) do provérbio acima, a solução de um pro-


blema começa (e continua, e termina) com passos simples e lógicos.
Mas, desde que avancemos numa direção clara e firme, com passadas
longas e visão apurada, precisaremos de bem menos que as centenas
de milhares de passos necessários para a viagem de mil quilômetros. E
a matemática, abstrata como é, não sofre de constrangimentos físicos:
podemos sempre recomeçar do zero, experimentar novas vias de ata-
que, ou retroceder a qualquer instante. Nem sempre dispomos de tais
luxos na resolução de problemas de outro tipo (como o de encontrar
o caminho para casa quando nos perdemos).
É claro que isso não a torna necessariamente fácil; se-o fosse, este
livro seria bem mais curto. Mas torna-a possível.
2 Estratégias de resolução de problemas

Há diversas perspectivas e estratégias gerais para a resolução cor-


reta de problemas, a referência clássica para muitas delas é Polya
(1957). Discutimos algumas dessas estratégias em baixo, ilustrando
brevemente como se pode usar cada uma delas no seguinte problema:

Problema 1.1. Os comprimentos dos lados de um triângulo formam


uma progressão aritmética de razão d. A área do triângulo é t. Calcule
os lados e os ângulos do triângulo.

Perceber o problema. Que tipo de problema é este? Há três tipos


principais de problemas:

• questões do tipo mostre que ... ou calcule ... , em que se deve pro-
var que uma certa afirmação é verdadeira, ou manipular uma
expressão para obter um certo resultado;

• questões do tipo encontre ... ou enco!ltre todos ... , em que se pede


para determinar algum-objeto (ou todos os objetos) satisfazendo
certas condições;

• questão do tipo existe ou não ... , em que se tem que provar uma
afirmação ou fornecer um contraexemplo (e portanto cai num
dos dois tipos anteriores).

O conhecimento do tipo de problema é importante pois determina


a abordagem básica. Nos problemas do tipo mostre que ... ou calcule ... ,
os dados nos são fornecidos e o objetivo é deduzir deles alguma afir-
mação ou calcular o valor de uma expressão; um problema deste tipo
é em geral mais fácil do gue um dos outros dois, pois há um objetivo
claramente à vista para o qual podemos dirigir os esforços. Questões
do tipo encqntre ... funcionam por tentativa e erro; em geral há um
primeiro palpite que qué!,se funciona, e depois o ajustamos um pouco
para que fique mais correto; ou então modificamos os requisitos a q11t•
3

o objeto a encontrar deve obedecer, de modo que eles sejam mais fá-
ceis de satisfazer. Os problemas do tipo existe ou não ... são em regra
os mais difíceis, pois primeiro temos que decidir se um certo objeto
existe ou não, e depois fornecer uma prova ou um contra-exemplo.
É claro que nem todos os problemas encaixam de modo claro numa
destas categorias, mas ainda assim o formato geral de uma pergunta
indica a estratégia básica a seguir. Se o problema a resolver for, por
exemplo, o de encontrar nesta cidade um hotel onde pernoitar, po-
demos primeiro modificar os requisitos, procurando antes um hotel a
menos de 5 quilômetros com um quarto vago por não mais que 50
euros por noite, e em seguida usar eliminação simples. Esta estratégia
é melhor do que provar que um tal hotel existe ou não existe, e é pro-
vavelmente também melhor do que escolher ao acaso um hotel mais a
mão e tentar demonstrar que é possível dormir nele.
O Problema 1.1 é do tipo calcule.... Há que exprimir certas in-
cógnitas à custa das variáveis dadas. Isto sugere que se tente urna
abordagem algébrica em vez de geométrica, com muitas équações re-
lacionando d, t e as incógnitas - incógnitas essas que são os lados
e ângulos do triângulo, e em relação às quais queremos resolver as
equaçoes.
Entender os dados. Quais são os dados do problema? Usualmente,
a questão é acerca de uns tantos objetos com certas propriedades espe-
cíficas. Para entendermos os dados do problema, precisamos de saber
como interagem esses objetos com tais propriedades. Isto é impor-
tante para focarmos a atenção nas técnicas e notaçôes apropriadas ao
problema. Em nossa questão modelo, por exemplo, ·os dados são um
triângulo, sua área, e o· fato de seus lados formarem uma progressão
aritmética de razão d. Dado que temos um triângulo, e consideramos
sua área e seus lados, vamos precisar, para lidar com o problema, de
teoremas que relacionem lados, ângulos e áreas; por exemplo, as leis
dos senos e dos cossenos e a fórmula da área.
Entender o objetivo. · O que é que queremos? Podemos precisar
4 Estratégias de resolução de problemas

encontrar um objeto, demonstrar uma afirmação, determinar a exis-


tência de um objeto com certas propriedades, ou sabe-se lá que mais.
Tal como a estratégia de entender os dados, a de entender o objetivo
ajuda a focar a atenção em quais as melhores armas a usar. Conhecer
o objetivo também nos ajuda a estabelecer metas parciais que, como
sabemos, nos aproximam da solução do problema. Em nossa questão
modelo o objetivo é encontrar os lados e ângulos de um triângulo. Isto
significa, como já dissemos, que precisamos de teoremas e fórmulas so-
bre lados e ângulos; mas também nos dá a meta parcial de encontrar
equações envolvendo os lados e os ângulos do triângulo.
Escolher uma boa notação. Agora que dispomos dos dados e de um
objetivo, devemos representá-los de forma eficiente, para que tanto uns
como outros apareçam da forma mais simples possível. Isto envolve
em geral as considerações das duas estratégias anteriores. Na questão
modelo, estamos já pensando em equações que envolvam d, t e os lados
e ângulos do triângulo. Precisamos exprimir lados e ângulos à custa
de variáveis: por exemplo, chamamos a, b e e aos lados e a, /3 e 'Y
aos ângulos. Mas, usando os dados, podemos ~implificar a notação:
os lados estão em progressão aritmética, e em vez de a, b e e podemos
usar a, a + d e a + 2d. Mas a notação ainda pode melhorar se a
fizermos mais simétrica, chamando b - d, b e b + d aos comprimentos
dos lados. O único (ligeiro) óbice desta notação é que ela força b a
ser maior do que d. Mas, se refletirmos, vemos que isto não é na
verdade uma restrição; e que dé fato o conhecimento de que b > d é
um dado extra para o nosso problema. Poderíamos reduzir ainda mais
a notação pondo os ângulos iguais a a, /3 e 180º - a - /3, mas isto é
feio e assimétrico - talvez seja melhor manter a notação· anterior, mas
tendo em conta que a + (3 + 'Y = 180º.
Escrever o que sabemos na notação que escolhemos, fazer um dia-
grama. Há três vantagens em passar·tudo para o papel:

(a) depois relembramos mais facilmente o problema;


5

(b) o papel é bom para fixar o olhar quando emperramos na resolu-


çao;

(c) o ato físico de escrever aquilo que sabemos pode detonar novas
ideias e conexões.

Mas há que ter cuidado para não escrevermos coisas supérfluas


nem enchermos o papel com detalhes miúdos; em vez disso, podemos
destacar os fatos que nos parecem mais úteis, e pôr aquelas ideias
mais dúbias, redundantes ou malucas num outro canto da folha de
rascunho. Eis algumas equações e desigualdades que podemos extrair
de nossa questão modelo:

• restrições físicas: a, (3, 'Y, t > Oe b 2: d; também podemos supor,


sem perda de generalidade, que d 2: O;

• soma dos ângulos internos dum triângulo: a+ (3 + 'Y = 180º;


• lei dos senos: (b - d)/ sen a = b/ sen (3 = (b + d)/ sen 'Y ;

• lei dos cossenos: b2 = (b- d) 2 + (b + d) 2 - 2(b - d)(b.+ d) cos(3,


etc.;

• fórmula da área: t = (1/2) (b - d) b sen 'Y (1/2)(b - d)(b +


d) sen (3 = (1/2) b (b + d) sena;

• fórmula de Heron: t 2 = s(s - b + d)(s - b)(s - b - d), onde o


semi perímetro s é dado por ( (b - d) + b + (b +d)) /2;

• desigualdade triangular: b +d~ b + (b - d).

Muitos destes fatos podem revelar-se redundantes ou desviar-nos


do caminho certo. Mas podemos usar de algum discernimento para
separar os fatos valiosos daqueles que não ajudam. É de esperar que
aH igualdades sejam mais úteis do que as desigualdades, pois tanto
6 Estratégias de resolução de problemas

nossos dados como nosso objetivo têm a forma de igualdades. E a


fórmula de Heron é especialmente promissora, pois o semiperímetro s
fica simplesmente igual a 3b/2. Podemos portanto destacar a fórmula
de Heron como sendo provavelmente útil.
É claro que também podemos desenhar uma figura, coisa que cos-
tuma ajudar em problemas geométricos, mas que neste caso não parece
nos adiantar muito:

b-d b
área t

ª·
b+d

Modificar ligeiramente o problema. Há muitas maneiras de int~o-


duzir variações no problema para tentar torná-lo mais acessível:

(a) Considerar casos especiais do problema, como por exemplo casos


extremos ou degenerados;

(b) Resolver uma versão simplificada do problema;

(c) Formular uma conjectura que implicaria na solução do problema,


e tentar resolvê-la primeiro;

(d) Deduzir algumas consequências do problema, e tentar começar


por demonstrá-las;

(e) Reformular o probl~ma (por exemplo, tomar o contrapositivo,


clemo11:-;trar por absurdo, ·ou tentar alguma substituiçào);
7

(f) Estudar soluções de problemas análogos;

(g) Generalizar o problema.

Isto é útil quando não conseguimos sequer iniciar um ataque ao


problema, pois a solução de um problema aparentado mais simples
pode mostrar-nos a via para a solução do problema maior. De modo
semelhante, a consideração de casos extremos ou a resolução do pro-
blema sob hipóteses adicionais podem fazer luz sobre o caso geral. Mas
atenção: os casos especiais são, por natureza, especiais, e uma técnica
elegante que por acaso lhes seja aplicável pode não ter a mínima uti-
lidade no caso geral. Há tendência para que isso aconteça quando o
caso especial é especial demais. É melhor começarmos com hipóteses
modestas, pois assim restingimo-nos tanto quanto possível ao espírito
do problema.
No Problema 1.1, podemos tentar o caso especial d = O. Nesse
caso precisamos de encontrar o comprimento do lado de um triângulo
equilátero de área t. O valor, fácil de calcular, é b = ·2 t 11-2 /3 114 . Isto
indica que a solução geral também deve envolver raízes quadradas ou
raízes quartas, mas tirando isso não nos sugere como abor9-ar o pro-
blema. A consideração de problemas análogos também pouco ajuda,
tirando que reforça a convicção de que um ataque algébrico em força
é o que é preciso.
Modificar grandemente o problema.· Neste tipo de estratégia mais
agressiva, submetemos o problema a mudanças importantes como a
de subtrair alguns dados, trocar os dados com o objetivo, ou negar o
objetivo (por exemplo, tentando provar que uma afirmação é falsa em
vez de verdadeira). Errl'suma, tentamos forçar o problema até que ele
quebre, e depois procuramos o lugar onde quebrou; isto identifica quais
são as componentes-chave dos dados, e aponta onde se encontram as
maiores dificuldades. Estes exercícios também podem ajudar-nos a
intuir quais são as estratégias mais promissoras e quais· são as mais
falíveis.
8 Estratégias de resolução de problemas

Em relação ao nosso problema em particular, poderíamos substi-


tuir o triângulo por um quadrilátero, um círculo, etc. Não que isto
ajude muito: o problema só fica mais complicado. Mas, por outro
lado, vemos que não precisamos de um triângulo em nossa questão,
mas apenas das medidas· do triângulo: não nos faz falta saber sua
posição. Isto é mais uma confirmação de que nos devemos concentrar
nos lados e ângulos (i.e., a, b, e, a, /3, 'Y) e evitar o uso de coordenadas
no plano ou abordagens semelhantes.
Poderíamos omitir alguns objetivos: em vez de calcularmos todos
os lados e ângulos ficávamos pelos lados, por exemplo. Mas depois
observamos que, pelas leis dos senos e dos cossenos, os ângulos do
triângulo ficam determinados em qualquer caso. Portanto só é neces-
sário calcular os lados. Mas sabemos que os lados têm comprimentos
b - d, b e b + d, e P.Or isso só precisamos encontrar b para acabar com
o problema.
Poderíamos também omitir alguns dados, como a diferença d en-
tre os lados, mas então ficaríamos com vãrias soluções possíveis e sem
dados suficientes para resolver o problema. Analogamente, omitir a
área t também nos deixaria dados insuficientes· para especificar uma
solução. (As vezes podemos omitir dados parcialmente, como dizer
apenas que a área é maior ou menor do que um certo patamar t 0 ;
mas isto já é complicado. Fiquemos primeiro pelas opções simples.)
Mas inverter o problema (trocar os dados pelos objetivos) já conduz
a algumas ideias interessantes. Suponhamos que temos um triângulo
em que os lados diferem de d, e que queremos encolhe-lo (ou seja lá
o que for) até que tenha área t. Podemos imaginar o triângulo a en-
colher e a ser deformado, mas mantendo a diferença d entre os lados.
De modo semelhante, poc;lemos considerar um triângulo com uma área
fixa, e moldá-lo até que tenha os lados na progressão aritmética cor-
reta. Com tempo, estas ideias poderiam funcionar, mas para resolv<!r
o problema usarei outra fi,bordagem. Não esqueçamos, porém, qt1<' 11111
Jiroblema pode ser resolv'ido êfo diversas nH1neiras, <) rw11!111111;1 d1·l11,s;
9

pode realmente ser considerada a melhor em absoluto.


Estabelecer resultados sobre o problema. Os dados são para se
usarem, e portanto devemos pe~ar neles e manipulá-los. Será que eles
assim produzem mais dados relevantes? Além do mais, estabelecer
pequenos resultados pode ser benéfico mais tarde, quando tentarmos
demonstrar o resultado principal ou encontrar a resposta. Por muito
pequeno que seja o resultado, não o esqueçamos - poderemos vir a
aplicá-lo mais tarde. E esse trabalho miúdo dá-nos o que fazer quando
estamos emperrados no problema.
Num problema do tipo calcule, como o do triângulo, esta tática
não é assim tão útil. Mas podemos tentar. Por exemplo, nossa meta
é exprimir b à custa dos parâmetros dados d e t. Por outras palavras,
b é na verdade uma função: b = b(d, t). (Se esta notação parece des-
locada num problema geométrico, é apenas porque a geometria tende
a ignorar a dependência funcional entre os objetos. Por exemplo, a
fórmula de Heron nos dá, de forma explícita, a área A em função dos
lados a, b, e - ou, em outras palavras, exprime a função A(a, b, e).)
Podemos estabelecer alguns mini resultados sobre esta função b(d, t),
por exemplo b(d, t) = b(-d, t) (porque, para cada progressão aritmó-
tica, há uma progressão equivalente com razão simétrica dà primeira)
ou b(kd, k 2 t) = k b(d, t) (isto obtém-se dilatando pelo fator k o tri-
ângulo com b = b(d, t)). Podemos até derivar bem relação a d ou a
t. Para este problema em particular, esta tática permite normalizar
alguns parâmetros, por exemplo fazendo t = 1 ou d= 1, e ainda for-
nece um modo de verificar a solução final. Acontece, porém, que neste
problema estes truques só levam a pequenos progressos, e por isso não
faremos uso deles aquL
Simplificar, explorar os dados, atingir metas parciais. Agora q1-1e
fixamos a notação e dispomos de umas quantas equações, devemos
considerar seriamente como atingir as metas parciais que estabelece-
mos. Em problemas simples, já há em geral procedimentos padrão.
(Por exemplo, as simplificações algébricas costumam ser discutidas
10 Estratégias de resolução de problemas

exaustivamente em textos escolares.) Em geral, esta é a parte mais


longa e mais difícil do problema, mas podemos evitar nos perdermos
se tivermos em mente os teoremas pertinentes, os dados e o modo de
os usar, e - o mais importante - o objetivo. É também boa ideia não
aplicar um certo método ou técnica às cegas, mas pensar e tentar pre-
ver onde tal método nos levaria; desse modo podemos evitar grandes
desperdícios de tempo, eliminando direções de investigação improfí-
cuas antes de esbanjarmos nelas enormes esforços, e dando prioridade
às direções mais promissoras.
No Problema 1.1, focalizamo-nos já na fórmula de Heron. Podemos
usá-la para atingir nossa meta parcial de uma fórmula para b. Afinal,
já sabemos que as leis dos senos e dos cossenos determinam a, /3, 'Y
uma vez conhecido b. Como indício adicional de que isto vai ser um
passo em frente, notemos que a fórmula de Heron envolve d e t - que
são, na prática, todos os nossos dados (já incorporamos na notação o
fato de os lados estarem em progressão aritméti~a). Seja como for, a
fórmula de Heron em função de d, te b fica

t2 = 3; ( 3; - b+ d) (32b - b) (3; ~ b- d) '


o que podemos simplificar para
t 2 _ 3b 2 (b - 2d)(b + 2d) _ 3b2(b2 - 4d2)
- 16 - 16 .
Temos agora que tirar o valor de b. O membro direito é um po-
linômio em b (considerando d e· t como constantes), e é de fato de
segundo grau em b2 • As equações de segundo grau resolvem-se fa-
cilmente: se nos livrarmos do denominador e mudarmos tudo para o
mesmo membro, obtemos
3b4 - 12d2b2 - 16t2 = O
e daqui, usando a fórmula resolvente,.

b2 = 12d2 ± J144d4 + 192t2 = 2d2 ± ✓4 d 4 + 16 t2.


6 . 3
11

Visto que b tem que ser positivo, obtemos

16
2d2 + 4d4 + -3 t2 .
Como verificação, podemos ver que, quando d = O, isto está de
acordo com nosso cálculo anterior de b = 2 t 112 /3 114 . Uma vez calcu-
lados os lados b - d, b, b + d, os valores dos ângulos a, /3, 'Y seguem
pelas leis dos senos e dos cossenos, e está feito!
12 Estratégias de resolução de problemas
CAPÍTULO 2

Exemplos da teoria dos números

Os números ímpares têm algo de divino, seja no nascimento, na


sorte, ou na morte.
William Shakespeare, As Alegres Comadres de Windsor

A teoria dos números poderá não ser divina, mas há a sua volta
uma aura de misticismo. Ao contrário da álgebra, cuja espinha dor-
sal são as regras de manipulação das equações, a teoria dos números
parece tirar seus resultados de uma fonte desconhecida. Veja, por
exemplo, o teorema de Lagrange (conjecturado primeiro por Fermat)
de que qualquer inteiro positivo é a soma de quatro quadrados perfei-
tos (por exemplo, 30 = 42 + 32 + 22 + 12 ). Em termos algébricos, esta é·
uma equação extremamente simples; mas, como nos restringimos aos
inteiros, as regras da álgebra fracassam. O ar inocente do resultado
ó irritante, e alguma experimentação mostra que ele parece funcio-.
nar, mas não sabemos porquê. De fato, o teorema de Lagrange não
14 Exemplos da teoria dos números

é facilmente demonstrável pelos métodos elementares tratados neste


livro: seria necessária uma incursão pelos inteiros gaussianos ou coisa
semelhante.
Mas há problemas menos profundos. Aqui vão alguns exemplos,
todos eles envolvendo um número natural n:

(a) O último algarismo de n é o mesmo do de sua quinta potência n 5 ;

(b) n é um múltiplo de 9 se e só se a soma de seus algarismos for


um múltiplo de 9;

(c) Teorema de Wilson: para n > 1, (n - 1)! + 1 é um múltiplo de


n se e só se n for um primo;

(d) Se k for um inteiro positivo ímpar, então lk + 2k + ••• + nk é


divisível por n + 1;

(e) Há exatamente quatr~ púmeros dé n algarismos (completados se


necessário com zeros à esquerda) que coincidem com os últimos
n algarismos dos respectivos quadrados. ·Por exemplo, os quatro
números de três algarismos com esta _propriedade são 000, 001,
376 e 625.

Todas estas afirmações podem ser demonstradas usando teoria ele-


mentar dos números; todas elas giram em torno da ideia básica da
aritmética modular, que é a de usar o poder da álgebra numa estru-
tura limitada a um número finito de inteiros. Diga-se a propósito que
a tentativa de resolver o problema (e) pode conduzir-nos aos núme-
ros p - ádicos, que formam uma espécie de aritmética modular em
dimensão infinita.
A teoria elementar dos números é uma simpática aldeola da mate-
mática. Mas as aplicaç_ões que derivam dos conceitos básicos de inteiro
e divisibilidade são de rim poder e diversidade espantosos. O conceito
15

de divisibilidade leva naturalmente ao de número primo, o que con-


duz ao estudo detalhado da fatoração e deu origem a uma das jóias
da matemática do final do século dezenove: o teorema dos números
primos, que prevê, com bom grau de aproximação, quantos primos
há menores do que um número dado. Por outro lado, o conceito das
operações entre inteiros adapta-se à aritmética modular, a qual, apli-
cando-se a subconjuntos dos inteiros, é generalizada pela álgebra dos
grupos, anéis e corpos finitos, e leva à teoria algébrica dos números,
onde o conceito de número é alargado aos irracionais quadráticos, aos
elementos dos corpos ciclotômicos, e aos números complexos. A teoria
dos números é uma pedra angular que sustenta uma apreciável porção
do edifício matemático. E, claro, também é divertida.
Antes de irmos aos problemas, recapitulemos alguma notação bá-
sica. Um número natural é um inteiro positivo (zero não é para nós
um número natural). O conjunto dos números naturais é denotado por
N. Um número primo é um número natural que é divisível por exata-
mente dois números naturais: ele mesmo e 1; não consideramos 1 como
primo. Dois números naturais m e n são primos entre si (também se
diz m é primo com n) se seu único divisor comum for 1. A notação
x = y (mod n), que lemos x é igual a y módulo n, significa que x e y
diferem por um múltiplo de n, sendo por exemplo 15 = 65 (mod 10).
A notação (mod n) significa que estamos trabalhando na aritmética
modular em que o módulo n foi identificado com O; assim, a aritmé-
tica modular (mod 10) é aquela em que 10 = O. Por exemplo, temos
65 = 15+10+10+10+10+10 = 15+0+0+0+0+0 = 15 (mod 10). A
aritmética modular também se distingue da aritmética usual por não
admitir desigualdades, é por nela todos os números serem inteiros.
Por exemplo, 7/2 =J 3,5 (mod 5), mas sim 7/2 = 12/2 = 6 (mod 5)
pois 7 = 12 (mod 5). Parece estranho efetuar a divisão deste jeito
arrevesado, mas podemos verificar que não há qualquer contradição,
c\111hora algumas divisões sejam ilegais, assim como a divis·ão por zero
{\ ilc-gal 110 corpo tra<liciorial dos números reais. A regra geral é que a
16 Exemplos da teoria dos números

divisão é legal se o denominador for primo com o módulo n.

2 .1 Algarismos
Mencionamos em cima que é possível aprender algo sobre um dado
número (em particular, se é divisível por 9) somando seus algarismos.
Na matemática mais avançada, sabe-se que esta operação não é afinal
particularmente importante (tem sido muito mais eficaz estudar os
números diretamente em vez de se usar sua expansão decimal), mas é
muito popular na matemática recreativa e há mesmo quem lhe tenha
atribuído conotações místicas! O que é certo é que a soma dos algaris-
mos surge frequentemente em problemas de competições matemáticas,
como este a seguir.
Problema 2.1 (Taylor, 1989, p. 7). Mostre 9-ue, entre quaisquer 18
números consecutivos com três algarismos, há pelo menos um que é
divisível pela soma de seus. algarismos.
Trata-se este de um problema finito: há 900 números com três
algarismos, e portanto em teoria poderíamos resolver o problema ma-
nualmente. Mas vejamos se podemos poupar algum trabalho. Em
primeiro lugar, o objetivo parece um pouco estranho: queremos que
um número seja divisível pela soma dos algarismos. Comecemos por
traduzir esse objetivo numa fórmula matemática, de modo a poder-
mos manipulá-lo mais facilmente. Um número de três algarismos pode
ser escrito na forma abc 10 , onde a, b, e são os algarismos; escrevemos
abc 10 para evitar confusão com abc; assim, abc 10 = 100 a+ 10 b + e,
mas abc = a x b x e. Empregando a notação usual alb para significar
que a divide b, o que queremos é obter a relação

(a+ b + ~) labc10, (1)


onde a, b, e são os algarismos de um dos 18 números consecutivos.
Podemos reduzir, simplificar, ou de algum modo tornar usávc~I c•sta
2.1 Algarismos 17

equação? É possível que sim, mas não há como simplificá-la para algo
minimamente decente (ou seja, uma equação que relacione a, b e e de
modo útil e direto). Na verdade~ (1) é horrível de manipular, mesmo
depois de substituirmos abc 10 por 100 a + 10 b + e. Olhemos para as
soluções abc 10 de (1):
100, 102, 108, 110, 111, 112, 114, 117, 120, 126, ... , 990, 999.
Elas parecem ser aleatórias, mas suficientemente frequentes para
que cada sucessão de 18 números consecutivos contenha alguma delas.
E, seja como for, que significado tem 18? Supondo que não seja uma
pista falsa (talvez 13 números consecutivos já cheguem, e o 18 seja
só para despistar), por que darem-nos 18? Pode ocorrer a alguns de
nós que a soma dos algarismos de um número está relacionada com
o número 9 (por exemplo, cada número dá o mesmo resto quando
dividido por 9 que sua soma de algarismos) e que 9 está relacionado
com 18, e por isso pode haver aqui alguma vaga conexão. Ainda assim,
números consecutivos e divisibilidade não combinam bem. Para que
tenhamos uma esperança de resolver a questão, parece· que· temos que
reformulá-la ou substituí-la por outra relacionada.
Agora que estamos em alerta para tudo o que tenha a ver com o
número 9, devemos notar que a maioria dos números que realmente
verificam (1) são múltiplos de 9, ou pelo menos múltiplos de 3. De fato,
na lista acima há apenas três exceções (100, 110 e 112), e praticamente
todos os múltiplos de 9 satisfazem (1). 'Assim, em vez de tentarmos
provar diretamente que
Em quaisquer 18 números consecutivos, há no mínimo uma solução
de (1);
podíamos tentar algo como
Em quaisquer 18 números consecutivos, há um múltiplo de 9 que
ó solução de (1).
Esta via parece quebrar a barreira entre nossos dados (18 núme-
roH consecutivos) e nosso objetivo (uma solução de (1)), pois em 18
11úmcroH consecutivos há sempre algum múltiplo de 9 (de fato há até
18 Exemplos da teoria dos números

dois); e, pelos exemplos numéricos e pelas propriedades heurísticas do


número 9, dá ideia de que os múltiplos de 9 satisfazem (1). Esta es-
tratégia de usar um passo intermédio é a melhor forma de conciliar
duas afirmações díspares.
Acontece que este passo intermédio (o de considerar múltiplos de
9) funciona mesmo, mas exige algum trabalho extra para cobrir todos
os casos. Na verdade é preferível considerar múltiplos de 18:
18 números consecutivos =} um múltiplo de 18 =} uma solução de (1)
As razões para esta mudança são de dois tipos:

• 18 números consecutivos contêm exatamente um múltiplo de 18,


mas já contêm dois múltiplos de 9. Parece mais limpo e mais
apropriado usar múltiplos de 18 em vez de múltiplos de 9. Afi-
nal, se os I?últiplos de 9 resolvessem o problema, a questão só
precisaria de 9 números consecutivos e não de 18;

• Deve ser mais fácil demonstrar (1) para múltiplos de 18 do que


para múltiplos de 9, visto que os múltiplos de 18 são casos espe-
ciais dos múltiplos de 9. De fato, acont.ece que os múltiplos de
9 nem sempre funcionam (veja-se o 909), ao contrário do que,
como veremos, sucede com os de 18.

Seja como for, a experimentação sugere que os múltiplos de 18


funcionam. Mas por quê? Tomemos, por exemplo, o 216, que é um
múltiplo de 18. A soma dos algarismos é 9, e 9 divide 216 porque 18
divide 216. Mais um exemplo: 882 é múltiplo de 18, e sua soma de
algarismos é 18. Daí que 18 seja obviamente divisível por sua soma
de algarismos. Manipulando mais uns quantos exemplos, vemos que
a soma dos algarismos. de um múltiplo de 18 é sempre 9 ou 18, que·
dividem o número dado quase por definição. E com esses palpites a
demonstração fica a um passo:
Demonstração. E~ 18 números consecutivos, um deles, digamos
abc 10 , tem que ser múltiplo ·de 18. Visto que abc 10 é também múltiplo
2.1 Algarismos 19

de 9, a+ b + e tem que ser múltiplo de 9. (Recordemos a regra de


divisibilidade por 9: um número é divisível por 9 se e só se sua soma
de algarismos for divisível por 9 . .) Visto que a + b + e varia entre 1 e
27, tem que ser igual a 9, 18 ou 27. O valor 27 só ocorre com 999, que
não é múltiplo de 18. Logo a + b + e é 9 ou 18, e portanto a + b + e
l 18. Mas 18 Iabc 10 por definição, e portanto a+ b + e Iabc 10 , como
queríamos. ■
Devemos ter em mente que, em questões envolvendo algarismos e
coisas semelhantes, uma abordagem direta em geral não é a melhor.
Uma fórmula complicada deve ser simplificada para ficar mais tratável.
Neste exemplo, substituímos a frase de entre quaisquer 18 números
consecutivos, um deles deve ser por qualquer múltiplo de 18 deve ser -
que é mais fraca, mas igualmente mais simples e mais pertinente para
a nossa questão (que dizia respeito à divisibilidade). Mas tratou-se
afinal de um bom palpite. E lembremo-nos de que, em problemas
finitos, as estratégias não são iguais às que se usam em matemática
avançada. Por exemplo, a fórmula
(a + b + e) 1abc10
não foi tratada do modo usual em matemática (por exemplo, como
uma aplicação da aritmética modular), mas em vez disso impusemos
restrições à soma a+ b + e (9, 18 ou 27) pelo fato de os números terem
apenas três algarismos, o que nos deixou- reduzidos às fórmulas muito
mais simples
9 labc10, 18 labc10 ou 27 labc10.
De fato, nunca tivemos que converter abc 10 em 100 a + 10 b + e;
embora pela lógica esse· pudesse parecer o primeiro passo indicado,
verificamos afinal que é uma pista falsa que não torna de modo nenhum
mais clara a solução do problema.
Uma nota final. Acontece que 18 é o número mínimo de números
consecutivos para garantir que um deles satisfaz (1). Dezessete nú-
rnerm; não chegam, como ·mostra a sucessão entre 559 e 575. (Para
20 Exemplos da teoria dos números

chegar a essa conclusão usei um computador e não algum artifício


matemático.) É claro que não precisamos saber isso para resolver o
problema.

Exercício 2.1. Num jogo de salão, o mágico pede a um dos par-


ticipantes para pensar num número de três algarismos, abc 10 . Em
seguida o mágico pede ao participante para somar os cinco números
acb 10 , bac 10 , bca 10 , cab 10 e cba 10 , e anunciar o resultado. Suponha que
essa soma é 3194. Qual era o número original abc 10 ? (Sugestão: ob-
tenha uma expressão mais manejável para a soma dos cinco números,
e em seguida use aritmética modular para restringir os valores de a, b
e e.)

Problema 2.2 (Taylor, 1989, p. 37). Há alguma potência de 2 cujos


algarismos possain ser reordenados para formarem outra potência de
2? (Não são admitidos zeros à esquerda: por exemplo, não se permite
0032.)

Este é o tipo de combinação que parece i~solúvcl: potências de 2


e reordenamento de algarismos. Isto porque:

(a) há muitas maneiras de reordenar os algarismos;

(b) e não é fácil determinar cada um dos algarismos de uma potência


de 2.

Isto leva a crer que o que aqui faz falta é uma estratégia ardilosa.
A primeira habilidade é tentarmos adivinhar a resposta. As provas
circunstanciais (o fato .de o problema ter aparecido numa competição·
matemática) sugerem que esta não é uma questão para resolver por
tentativas, e portanto a resposta provável deverá ser não. (Por outro
lado, quem sabe se não será possível, por alguma construção parti-
cularmente engenhosa, obter uma hábil reordenação dos algarismos?
2 .1 Algarismos 21

Mas uma tal construção não deve ser fácil de encontrar. É melhor ten-
tarmos primeiro as vias mais simples. Se acertarmos, teremos poupado
muito tempo ao não enveredarmos por vias difíceis. Se não acertar-
mos, estaríamos em qualquer caso condenados a um caminho penoso.
Isso não significa que devamos descartar uma via promissora por ser
difícil, mas apenas que devemos lançar um olhar comedido à nossa
volta antes de mergulharmos em águas profundas.)
Tal como no Problema 2.1, os algarismos são de algum modo uma
pista falsa. No Problema 2.1, só precisávamos saber duas coisas sobre
a soma dos algarismos: a primeira era um critério de divisibilidade,
a segunda uma majoração de seus possíveis valores. Com isso, evita-
mos as complicações de uma equação exata. Provavelmente ocorrerá
aqui coisa muito semelhante: temos que simplificar o problema ge-
neralizando o processo de permutação dos algarismos. De um ponto
de vista estritamente lógico, ficamos em situação mais difícil porque
temos que provar mais; mas ganhamos terreno em termos de clareza
e simplicidade. (Para que nos sobrecarregarmos com· dados que não
podemos usar e servem apenas para nos confundir?)
Temos portanto que selecionar as propriedades mais relevantes das
potências de 2 e das permutações de algarismos - com sorte, algumas
propriedades de umas serão incompatíveis com as das outras. Trate-
mos primeiro das potências de 2, mais fáceis de analisar. Ei-las:
1, 2,4, 8, 16, 32, 64,128,256,512, 1024, 2048,4096, 8192, 16384,
32 768, 65 536, ...
Bom, não há muito que aqui possamos dizer sobre os algarismos. O
último algarismo de qualquer potência de 2 (com exceção do número
1) é obviamente par, rrias os restantes algarismos parecem bastante
aleatórios. Tomemos o número 4096, por exemplo: um algarismo·
ímpar, os outros pares, e um deles até é O. O que nos impede de
o reordenar em outra potência de 2? Poderíamos transformá-lo, por
exemplo, em 24256 = 1523 ... 936? Claro que não, responderíamos. E
por quê? Porque esse número é grande demais. Então o tamanho
22 Exemplos da teoria dos números

conta? Sim - há milhares de algarismos em 24256 , mas apenas quatro


em 4096. Ahá - então o reordenamento dos algarismos não pode
mudar seu número total. (Ponhamos por escrito todos os fatos que
possam ser úteis no problema, mesmo que sejam simples - pois os
fatos óbvios podem de outro modo não nos ocorrer quando precisarmos
deles. Mesmo o ouro que está quase à superfície tem que ser procurado
- e agarrado uma vez que se encontre.)
Com este grão de informação, podemos então avançar com nosso
plano de generalização? Nossa pergunta generalizada é agora a se-
guinte:
Há alguma potência de 2 que tenha o mesmo número de algarismos
que outra potência de 2?
Infelizmente, salta a vista que a resposta a esta pergunta é sim:
veja-se o 2048 e o 4096. Generalizamos demais. (Observemos que, de
uma resposta afirmativa a esta pergunta, não resulta necessariamente
uma resposta afirmativa à pergunta original.) Olhemos novamente o
Problema 2.1. O mero conhecimento de que a soma dos algarismos
de um múltiplo de 18 tem que ser um múltiplo de 9 não bastou para
resolver o problema: também precisamos do fato de a soma dos alga-
rismos de um número de três algarismos ser.no máximo 27. Em suma,
ainda não coligimos fatos suficientes sobre nosso problema para o re-
solvermos. Contudo, já tivemos um sucesso parcial ao restringirmos as
possibilidades do reordenamento dos algarismos. Tomemos novamente
o 4096: ele pode apenas ser reordenado num outro número de quatro
algarismos. E quantas são as potências de 2 com quatro algarismos?
Apenas quatro: 1024, 2048, 4096 e 8192. É que as potências de dois
se obtêm por duplicação sucessiva, e por isso não pbdem manter-se
no mesmo escalão por JTIUito tempo. De fato, depressa vemos que no·
máximo quatro potências de 2 podem ter o mesmo número de alga-
rismos. (A quinta potência consecutiva de 2 seria igual a 16 vezes a
primeira, e por isso teria mais algarismos do que ela.) Isto significa
que, por cada potência ·de 2·, há no máximo três outras que poderiam
2.1 Algarismos 23

ser reordenações dos algarismos da primeira. Uma vitória parcial: fi-


camos só com três ou menos suspeitos por eliminar por cada potência
de 2, em vez da infinidade que tínhamos no início. Com um pouco
mais de trabalho, talvez possamos também eliminar esses suspeitos.
Dissemos que, quando permutamos os algarismos, o número com
que ficamos tem o mesmo número de algarismos que o original. Mas o
recíproco anda longe de ser verdadeiro, e por isso esta propriedade da
reordenação dos algarismos não vai, por si só, resolver o problema. Isto
significa que generalizamos mais do que devíamos e forçamos demais
nossa sorte. Recomecemos do início. Algo mais deve ser preservado
quando permutamos os algarismos. Atentemos em alguns exemplos -
usando outra vez o 4096, pois já temos experiência com ele. Eis suas
possíveis reordenações:

4069, 4096, 4609, 4690, 4906, 4960, 6049, 6094, 6409,


6490, 6904, 6940, 9046, 9064, 9406, 9460, 9604, 9640.

O que há de comum nelas? Têm o mesmo conjunto de algarismos.


Isso é muito bonito, mas o conjunto de algarismos não é um o~jeto ma-
temático especialmente útil (não há muitos teoremas ou métodos que
usem esse conceito). Contudo, a soma dos algarismos já é uma arma
mais convencional. E, bem, se dois números têm o mesmo conjunto de
algarismos, então têm necessariamente a mesma soma de algarismos.
Juntamos mais um grão à informação disponível: a permutação dos
algarismos preserva a soma de algarismos. Combinando com o grão
anterior temos uma nova pergunta alternativa:
Há alguma potência de 2 que tenha o mesmo número de algarismos
e a mesma soma de algarismos que outra potência de 2?
Se a resposta a esta pergunta for negativa, será também negativa a
resposta à pergunta original. Acontece que esta questão é mais tratável
do que a original, pois número de algarismos e soma de algarismos são
conceitos-padrão em teoria dos números.
so 1~
C/)
ü o.. Q
i;l:>
,...
i;l:>
i;l:>
C/) so
'O ('O
O"
(1) o ,...
,... C/)

j, ('O> i;l:>

::l
~
......
'O
('O
,...
o §i ~
~ o.. .::
('O g_
o
C/) •
2k I; algarismos 2k E algarismos 2k I: algarismos 1:-.j i;l:>
..o· ('O

.:: o s
('O

o 1 128 11 16384 22 s
('O

2 2 256 13 32768 26 ,...


::l
51>
. 4 4 512 8 65536 25 ......
::l
C/)
8 8 1024 7 131072 14 'O
('O

16 7 2048 14 262144 19 ......


()
o
::l
32 5 4096 19 524288 29 ('O

64 10 8192 20 1048 576 31


so
C/)
o..
§i Ili
,...
C/) ('!)
o o"1
s§i ~-
o..
o.. o
Ul
('O

-
i;i,
oq
i;i,
......
>-$
::,
.;:,
s
('!)
"1
C/)
1
oUl
2.1 Algari8mos 25

• As somas de algarismos tendem a ser bem pequenas. Por exem-


plo, a de 217 é apenas 14. É algum azar que seja assim, pois
os números pequenos têm .maior probabilidade de se repetir do
que os grandes; (Se cada uma de dez pessoas escolher à sorte
um número de dois algarismos, a probabilidade de repetições é
apreciável (9,5 %), mas se os números forem de dez algarismos
ela já só é de um num milhão, o que é quase tão irrisório como
a probabilidade de ganhar a loteria.)

• Alguns números têm a mesma soma de algarismos: 16 e 1024,


por exemplo. Mas parece que as somas de algarismos vão ainda
assim crescendo lentamente: é de esperar que uma potência de 2
com 10 algarismos tenha uma soma menor do que a de uma outra
com 100. Lembremos, porém, de que estamos nos restringindo a
potências de 2 com o mesmo número de algarismos, e portanto
esta ideia não será de grande ajuda.

A conclusão que tiramos destas observações é que as potências de 2


têm uma estrutura macroscópica fácil de apreender (crescem devagar
com n; de fato, é altamente provável - mas ainda não foi provado! -
que a soma dos algarismos de 2n seja aproximadamente igual a (4,5
log 10 2)n ~ 1,355n para n grande), mas uma estrutura microscópica
deplorável. Os algarismos oscilam demais. Dissemos antes que con-
junto dos algarismos não era um conceito de fácil manejo; agora parece
que soma de algarismos também não será muito espetacular. Haverá
alguma outra simplificação do problema que produza algo com que
possamos trabalhar?
U mmm. Mencionamos antes que soma de algarismos era um con:-
ceito padrão em matemática; veja por exemplo nosso problema ante-
rior. Mas o único modo como as somas de algarismos têm sido empre-
gadas regularmente com sucesso é pela redução módulo 9.. Recordemos
q1w qualquer número é igual módulo 9 a sua soma de algarismos; por
26 Exemplos da teoria dos números

exemplo,

3297 = 3 x 103 + 2 x 102 + 9 x 10 1 + 7 x 10° (mod 9)


= 3 x 13 + 2 x 12 + 9 x 11 + 7 x 1° (mod 9)
= 3 + 2 + 9 + 7 (mod 9)

uma vez que 10 é igual a 1 (mod 9).


A nova pergunta modificada é então a seguinte:
Há alguma potência de 2 que tenha o mesmo número de algarismos
e a mesma soma de algarismos módulo 9 que outra potência de 2?
Fazendo uso do fato de qualquer número ser igual (mod 9) a sua
soma de algarismos, podemos ainda reformular a pergunta:
Há alguma poMncia de 2 que tenha o mesmo número de algarismos
e o mesmo resto módulo 9 que outra potência de 2?
Com isso eliminamos completamente os incômodos conceitos de
reordenamento de algarismos,· conjunto de algarismos e soma de alga-
rismos, o que parece promissor. Modifiquemos ~gora a anterior tabela
das somas de algarismos das potências de 2 e vejamos o que acontece.
O que temos que provar agora é que não há duas potências de 2
com o mesmo resto (mod 9) e o mesmo número de algarismos. Ora
bem, vemos pela tabela que há várias potências com o mesmo resto:
1, 64, 4096 e 262144, por exemplo. Mas todos estes têm d~ferentes
números de algarismos. De fato, as potências de 2 com o mesmo
resto (mod 9) parecem estar tão afastadas uma da outra que não há
esperança de terem igual número de algarismos. Mais,: as potências
de 2 com o mesmo resto parecem estar regularmente espaçadas... e
depressa notamos que os restos (mod 9) se repetem a cada seis passos.
Demonstra-se facilmente esta conje<?tura usando aritmética modular:
~
~

-
o►
~
q

u.i"
s
oUl

. 2k resto.(mod 9) 2k resto (mod 9) 2k resto (mod 9)


o
1 1 128 2 16384 4
2
4
2
4
256
512
4
8
32768
65536
8
7
.
8 8 1024 7 131072 5
16 7 · 2048 5 262144 1
32 5 4096 1 524 288 2
64 1 8192 2 1048 576 4

~
-.J
28 Exemplos da teoria dos números

Este resultado significa que os restos das potências de 2 repe-


tir-se-ão indefinidamente, como uma dízima periódica: 1, 2, 4, 8, 7,
5, 1, 2, 4, 8, 7, 5, 1, 2, 4, 8, 7, 5, ... Por sua vez, isto implica que
duas potências de 2 com o mesmo resto têm que estar pelo menos à
distância de seis passos uma da outra. Mas então elas não podem
ter o mesmo número de algarismos, porque uma seria, no mínimo, 64
vezes maior que a outra. Logo, não há duas potências de 2 com o
mesmo número de algarismos e a mesma soma de algarismos (mod 9).
Resolvemos nosso problema modificado, de modo que agora podemos
refazer os passos pela ordem inversa e responder à pergunta original:

Demonstração. Suponhamos que duas potências de 2 se obtêm


uma da outra por permutação dos algarismos. Então elas têm igual
número de algarismos, e também igual soma de algarismos (mod 9).
Mas as somas de algarismos (mod 9) são periódicas com período 6 e
sem repetições dentro de cada período, de modo que as ditas potências
estão separadas por um mínimo de seis passos. Mas nesse caso é
impossível que tenham o mesmo número de aJgarismos, o que é uma
contradição. ■

Este problema foi simplificado por etapas, até que suas compo-
nentes menos manuseáveis ou menos amigáveis foram substituídas por
outras mais naturais, flexíveis .e cooperantes. Esta simplificação pode
ser um caso de tentativa e erro: há sempre o perigo de simplificarmos
demais, ou (atraídos por uma pista falsa) de simplificarmos no sen-
tido errado. Mas, numa pergunta como esta, quase tudo era melhor
do que tentarmos trab_alhar com permutações de algarismos, e por
isso a simplificação não poderia fazer grande estrago. É possível que
simplificaç9es e outras manobras nos lancem numa busca fútil, mas,
se estivermos mesmo ~mperrados, vale sempre a pena experimentar
alguma ideia.
2.2 Equações diofantinas 29

2.2 Equações diofantinas


Uma equação diofantina é uma equação algébrica (o exemplo clás-
sico é a 2 + b2 = c2 ) onde todas· as variáveis representam inteiros. O
objetivo usual é encontrar todas as soluções da equação. Em geral
há mais do que uma solução, mesmo com a exigência de elas serem
inteiras. Essas equações podem ser resolvidas algebricamente, mas
também podem-se usar métodos da teoria dos números como a divi-
sibilidade, a aritmética modular e a fatoração. Eis um exemplo:

Problema 2.3 (Australian Mathematics Competition, 1987, p.15).


Determine todos os inteiros n para os quais a equação 1/a + 1/b =
n/(a+b) tenha alguma solução inteira a e b (com a, b e a+b não-nulos).
Este parece ser um caso padrão de equação diofantina, e por isso
talvez devamos começar por nos livrar de denominadores, obtendo

(a+ b)/ab = n/(a + b)

e em seguida
(a+ b) 2 = nab. (2)
E agora? Poderíamos eliminar o n, e dizer que

ab (a+ b?
1

(usando o símbolo de divisibilidade I que introduzimos no Problema


2.1), ou tentar nos concentrarmos no fato de nab ser um quadrado.
Estas técnicas são boas, mas não parecem funcionar neste problema.
A relação entre os membros esquerdo e direito de (2) não é suficiente-
mente forte. De um lado temos um quadrado, do outro um produto.·
Uma das regras na resolução de problemas é estarmos preparados
para abandonar temporariamente uma abordagem interessante, mas
sem resultados, em favor de outra mais promissora. Neste problema
podemos tentar um ataque algébrico, e regressar à teoria dos números
30 Exemplos da teoria dos números

se a álgebra não funcionar. Expandindo (2) e agrupando os termos,


obtemos
a2 + (2 - n)ab + b2 = O,
o que dá, se tivermos a ousadia de usar a fórmula resolvente,

Isto parece complicado, mas de fato podemos virar a complicação


a nosso favor. Sabemos que a, b e n são inteiros, mas há uma raiz
quadrada na fórmula. Ora, isto obriga a que o termo debaixo da raiz,
(n - 2) 2 - 4, seja um quadrado perfeito, o que significa que um certo
quadrado menos 4 é também um quadrado. Isto é muito restritivo.
Como os intervalos entre quadrados sucessivos ficam maiores do que
4 logo após os primeiros quadrados, precisam~s apenas de testar os
valores pequenos de n. Concluímos que (n - 2) 2 tem que ser 4, e
portanto n é O ou 4. Podemos agora tratar separadamente cada caso,
encontrando, para cada um deles, ou uma solução ou uma prova de
que ela não existe.
Caso n = O. Substituindo este valor em (2), obtemos (a+ b) 2 = O,
e portanto a+b = O. Mas isto é impossível, pois ficaria 0/0 na equação
original, o que é proibido. Logo n não pode ser O.
Caso n = 2. Desta vez (2) dá-nos (a+b) 2 = 4ab, o que, agrupando
termos, fornece a 2 - 2ab + b2 = O. Fatorando, obtemos (a --,b) 2 = O,
e portanto a tem que ser igual a b. Isto não é uma contradição, mas
sim uma solução: a= b e n = 4 dão uma igualdade quando usadas na
equação original.
A resposta é portantp n = 4, mas ela foi obtida pelo método pouco
elegante da fórmula resolvente. Usá-la é em geral desajeitado, mas,
como introduz uma raiz quadrada,· o que faz com que o termo de-
baixo da raiz tenha que ser um quadrado perfeito, ela pode por vezes
revelar-se útil.
2.2 Equações diofantinas 31

Os problemas diofantinos podem tornar-se extremamente difíceis


quando uma das variáveis aparece em expoente; o caso mais famoso
é o último teorema de Fermat, que afirma não haver números natu-
rais satisfazendo an + bn = cn quando n > 2. Felizmente, há outros
problemas que envolvem expoentes mas são de trato mais fácil.
Problema 2.4 (Taylor, 1989, p. 7). Determine todas as soluções de
2n + 7 = x 2 , onde n e x são inteiros.
Esta pergunta obriga mesmo a ir por tentativa e erro até encon-
trarmos o caminho certo. Os métodos mais elementares com equações
diofantinas são a aritmética modular e a fatoração. A aritmética mo-
dular reduz toda a equação a um módulo que seja conveniente, às
vezes constante (como (mod 7) ou (mod 16)), outras vezes variável
(como (mod pq)). O método da fatoração converte o problema numa
igualdade do tipo (fator)x(fator)=(expressão bacana), onde o mem-
bro direito pode ser uma constante (o melhor resultado possível), um
primo, um quadrado, ou qualquer outra coisa com uma-escolha restrita
de fatores. No Problema 2.3, por exemplo, ambos estes métodos fo-
ram tentados inicialmente, mas acabaram sendo descartados cm favor
duma abordagem algébr:ica, onde na verdade a fatoração intervinha
sob disfarce (pois a certa altura tínhamos (n - 2) 2 - 4 = (quadrado)).
É melhor tentarmos primeiro as técnicas elementares, dado que
isso pode evitar que, mais tarde, andemos às voltas sem sair do lu-
gar. Poderíamos abandonar estes métodos e tentar analisar a equação
aproximada
x=J2n+7~2nf2,
que nos poderia levar a tópicos avançados da teoria dos números como
as frações contínuas, a equação de Pell e as relações de recorrência. É·
um caminho que funciona; mas vamos ao invés disso procurar a via
mais elegante (e mais preguiçosa).
Obter urna fatoração útil é praticamente impossível, exeeto quando
n (~ par. Nesse caso temos uma diferença de dois quadrados (um tipo
32 Exemplos da teoria dos números

de fatoração vital em equações diofantinas):

onde m = n/2. Daqui vemos que x - 2m e x + 2m, sendo fatores de


7, têm que ser -7, -1, 1 ou 7; e, considerando os diferentes casos,
vemos rapidamente que não há soluções (quando n é par). Mas mais
do que isso não nos poderá dar o método de fatoração: ele não nos diz
onde estão as soluções nem quantas são. (Sabemos agora, porém, que
n tem que ser ímpar.)
É a vez de a aritmética modular intervir. A estratégia é usar um
módulo que nos livre de um ou mais termos. Podemos, por exemplo,
reduzir a equação módulo x, obtendo

2n + 7 = O (mod X),
ou talvez módulo 7, o que dá

Infelizmente, estes métodos não funcionam bem de modo algum.


Mas, antes de desistirmos, falta tentar um outro módulo. Experimen-
tamos eliminar os termos 7 e x 2 ; será que, em vez destes, podemos
eliminar o 2n7 Sim, escolhendo, por exemplo, o módulo 2. Temos
então
O+ 7 = x 2 ( mod 2)
quando n > O, e
1+ 7 = x 2 ( mod 2)
quando n = O. Isto _não é mau, pois conseguimos eliminar quase·
completamente o papel do n. Mas ainda não funciona, já que o termo
x 2 à direita pode tomar ambos os valores O ou 1, e portanto não
excluímos nenhuma p_ossibilidade. Para restringir os valores de x 2 ,
temos que usar um módulo diferente. Com esta preocupação c111 lll('IIÜ)
2.3 Somas de potências 33

- a de restringir os valores possíveis do membro direito - ocorre-nos


tentar o módulo 4 em vez do 2:

2n + 7 ~ x 2 (mod 4).
Em outras palavras, temos

O + 3 = x 2 ( mod 4) quando n > 1, (3)


2 + 3 = x 2 ( mod 4) quando n = 1, (4)
1 + 3 = x 2 ( mod 4) quando n = O. (5)

Dado que x 2 (mod 4) tem que ser O ou 1, a hipótese (3) fica


eliminada. Sendo assim, n só pode ser O ou 1. Uma verificação rápida
mostra-nos que só serve n = 1, e que x tem que ser 3 ou -3.
A ideia-chave, quando temos uma equação diofantina de que pro-
curamos todas as soluções, é eliminar todas as possibilidades à exceção
de um número finito delas. E esta é outra das razões I?ªra. os módulos
7 ou x não funcionarem: pois se o fizessem, eles eliminariam todos os
casos, ao invés do que acontece com (mod 4), que nos deixa ainda um
punhado deles.

Exercício 2.2. Determine o maior inteiro positivo n tal que n:J + 100
seja divisível por n + 10. (Sugestão: use (mod n + 10), eliminando n
através da igualdade n = -10 (mod n-+ 10).)

2. 3 Somas de potências
. .
Problema 2.5 (Hajós et al., 1963, p. 74). Demonstre que, para
qualquer inteiro não-negativo n, o número 1n + 2n + 3n + 4n é divisível
por 5 se e só se n não for divisível por 4.

À primeira vista este problema é meio intimidatório: equaçoes


como a de cima podem fazer-nos lembrar o último teorema de Fermat,
34 Exemplos da teoria dos números

célebre por sua dificuldade. Mas nossa pergunta é muito menos dura.
Queremos mostrar que um certo número é (ou não é) divisível por 5.
A menos que haja uma fatoração evidente a efetuar, teremos que usar
a abordagem modular. (Ou seja, teremos que mostrar que 1n + 2n +
3n+4n = O (mod 5) se 4 não dividir n, e 1n+2n+3n+4n -:f. O (mod 5)
se 4 dividir n.)
Já que estamos lidando com números pequenos, podemos calcular
à mão alguns valores de 1n + 2n + 3n + 4n (mod 5). O melhor é
estudarmos separadamente os termos 1n (mod 5), 2n (mod 5), 3n
(mod 5) e 4n (mod 5) antes de os somarmos:

(mod 5)
n 1n 2n 3n 4n 1n + 2n + 3n + 4n
o 1 1 1 1 4
1 1 2 3 4 o
2 1 4 4 1 o·
3 1 3 2 4 o
4 1 1 1 1 4
5 1 2 3 4 o
6 1 4 4 1 o
7 1 3 2 4 o
8 1 1 1 1 4

A periodicidade da tabela_ é evidente: cada um dos termos 1n ,


2n, 3n e 4n é periódico com período 4. Para demonstrarmos esta
conjectura, só temos que apelar à definição de periodicidade.
Tomemos por exemplo 3n. Dizer que ele é periódi~o com período
4 significa simplesmente que
3n+4 = 3n (mod 5).

Mas provar isto é fácil, pois


.
3n+4 = 3n X 81 = 3n (mod 5),
2.3 Somas de potências 35

dado que 81 = 1 (mod 5).


De jeito semelhante, mostramos que 1n, 2n e 4n são periódicos com
período 4. Resulta então que 1n .+ 2n + 3n + 4n é periódico com período
4, e isto implica que só tenhamos que verificar nosso problema para
n = O, 1, 2, 3: a periodicidade resolve todos os restantes casos. Mas
para estes valores de n já resolvemos a questão (basta ver a tabela em
cima), de forma que está tudo feito. (A propósito, se supusermos que
n é ímpar, dispomos de um método mais elementar: basta agrupar e
cancelar os termos dois a dois.)
De cada vez que deparamos com expressões que envolvam parâ-
metros (n neste caso), a periodicidade faz sempre jeito, porque então
já não precisamos de lidar com todos os valores do parâmetro: basta
considerar um período (neste caso n = O, 1, 2, 3).

Exercício 2.3. Mostre que a equação x 4 + 131 = 3y4 não tem solução
se x e y forem inteiros.

Viramo-nos agora para um problema mais intricado, ·ainda sobre


somas de potências.

Problema 2.6 (Shklarsky et al., 1962, p. 14). (**) Sej'am k e n


números naturais, com k ímpar. Mostre que a soma lk + 2k + •••+ nk
é divisível por 1 + 2 + · · · + n.

Diga-se que esta questão se resolve habitualmente usando polinô-


mios de Bernoulli (ou aplicando de modo hábil a divisão com resto),
uma interessante área da matemática com muitas· aplicações. Mas,
sem a artilharia pesada dos polinômios de Bernouili (ou da função
zeta'de Riemann), teremos que nos contentar com a velha e desar-
mada teoria dos números.
Para começar, sabemos que 1 + 2 + · · · + n também se escreve
na forma n(n + 1)/2. Qual das duas formas usar? A primeira é
mais bonita, mas talvez inútil em questões de divisibilidade (sempre
mais fáceis se o divisor for expresso como produto e não como soma);
36 Exemplos da teoria dos números

poderia ser útil se houvesse alguma fatoração simpática de 1k + 2k +


· · ·+nk que envolvesse 1+2+· · ·+n, mas não há (pelo menos não uma
que seja óbvia). Se tivéssemos um modo de relacionar a divisibilidade
por 1 + 2 + · · · + n com a divisibilidade por 1 + 2 + · · · + (n + 1),
então o método de indução poderia ser o indicado, mas isso também
não parece provável. Vamos portanto tentar usar a forma n(n + 1)/2.
Usando então a aritmética modular (que é a forma mais flexível de
provar que um dado número divide outro), nosso objetivo é mostrar
que
lk + 2k + ... + nk = O (mod n(n + 1)/2).
Ignoremos para já o 2 em n( n + l) /2. O que tentamos provar é
então qualquer coisa do tipo

. (fator 1) x (fator 2) 1 (expressão).

Se os dois fatores forem primos entre si, ndsso objetivo equivale a


demonstrar separadamente as duas condições

(fator 1) 1 (expressão) e (fator 2) 1 (expressão).

Isto deve ser mais simples de fazer: é mais fácil demonstrar a


divisibilidade quando o divisor é menor. Más tem aquele 2 em n(n +
1) /2 atravancando. Para resolver isso teremos de dividir em casos,
conforme n seja par ou ímpar. 1 Os dois casos são muito semelhantes,
e por isso trataremos apenas do caso em que n é par. Podemos então
escrever n = 2m (de modo a evitar a ocorrência nas equações de termos
desajeitados em n/2 - pequenas tarefas de limpeza como esta ajudam
a resolução a deslizar suavemente!). Substituindo, na ~xpressão, n por
2m, temos que mostrar que
lk + 2k + · • · + (2mt = O (mod m(2m + 1)),
1 Outra possibilidade é multiplicarmos· ambos os membros por 2, tendo então

que provar que 2 (1 k + 2k +: ••+ nk)= O (mod n(n + 1)). Isto acaba por ser mais
ou menos equivalente à abordagem que seguimos aqui.
2.3 Somas de potências 37

mas, como m é primo com 2m + 1, isto é equivalente a mostrar que

e
lk + 2k + • • • + (2ml = O(mod m).
Tratemos primeiro do (mod 2m + 1). .Este caso é semelhante ao
Problema 2.5 mas um pouco mais fácil, pois sabemos que k é ímpar.
Usando o módulo 2m + 1, 2m fica igual a -1, 2m - 1 a -2, e assim
por diante, de forma que nossa expressão 1k + 2k + · · · + (2m )k fica

Com as coisas dispostas desta maneira, podemos cancelar tudo de


uma vez: k é ímpar, de modo que (-l)k é igual a -1; logo, (-a)k é
igual a -ak. A conclusão é que, nesta soma, os termos se cancelam
aos pares: 2k cancela com (-2)\ 3k com (-3)\ etc., o que nos deixa
com O (mod 2m + 1), como queríamos.
Falta agora tratar do (mod m): ou seja, falta mostrar que

lk + 2k + 3k + · · · + (m - 1t + mk + (m + 1t + · · ·
+ (2m - ll + (2ml = O (mod m).

Porém, como trabalhamos em (mod m), alguns dos termos acima


podem ser simplificados: m e 2m são ambos equivalentes a zero, m + 1
é equivalente a 1, m+2 é equivalente a 2, etc. A somq, acima fica então

Alguns termos aparecem duplicados; se os agruparmos (e se des-


cartarmos os termos O), obtemos
38 Exemplos da teoria dos números

Podemos quase fazer como no caso (mod 2m + 1), mas há um pe-


queno obstáculo quando m é par. Sem for ímpar, podemos reescrever
a última expressão como

2(1k + 2k + 3k + ••• + ((m-1)/2l + (-(m -1)/2l + · · ·


+ (-2l + (-ll) (mod m),
e proceder, tal como antes, ao cancelamento dos termos. Mas, se m
for par (m = 2p, digamos), o termo central, pk, não cancela com
nenhum outro. Em outras palavras, nesse caso a expressão não fica
imediatamente igual a O, mas antes se reduz a

2pk (mod 2p),

que, obviamente,. também é O. Demonstramos portanto que, quando


n é par (igual a 2m), lk + 2k + ••• + nk é diyisível por n(n + 1)/2,
independente de m ser par ou ímpar.

a
Exercício 2.4. Complete demonstração precedente, averiguando o
que acontece quando n é par.

No próximo problema lidamos com um tipo especial de somas de


potências: as somas de inversos.

Problema 2. 7 (Shklarsky et al., 1962, p. 17). Seja pum primo maior


que 3. Mostre que o numerador da soma

1/1 + 1/2 + 1/3 + · · · + 1/(p - 1)


(escrita como fração irredutível) é divisível por p 2 . Qu'ando pé 5, por
exemplo, a soma fica 1/1 + 1/2 + 1/3 + 1/4 = 25/12, e o numerador
é obviamente divisível por 52 .

Esta é uma questão, do tipo mostre que, e não do tipo encontre ou


prove que existe, e por isso não será impossível de resolver. Contu<lo,
2.3 Somas de potências 39

temos que demonstrar algo acerca do numerador de uma fração irre-


dutível - coisa com que não é fácil lidar! Esse numerador terá de ser
convertido em algo mais amigáxel, como uma expressão algébrica, de
modo que o possamos manipular melhor. Além do mais, a pergunta
não exige apenas divisibilidade por um primo, mas sim pelo quadrado
de um primo, o que é substancialmente mais difícil. Para tornar a
questão mais acessível, gostaríamos de a reduzir de algum modo a da
simples divisibilidade por um primo.
Tendo em conta a forma da questão, devemos ter em mente os
seguintes objetivos:

(a) Escrever o numerador como uma expressão matemática que pos-


samos manipular;

(b) Tentar reduzir o problema, se possível, a uma questão de divisi-


bilidade por p e não por p 2 •

Tratemos primeiro de (a). É fácil de obter um numerador, mas que


não é necessariamente o da fração irredutível. Reduzindo as frações
ao mesmo denominador e somando, obtemos

2 X 3 X · ·· X (p - 1) + 1 X 3 X · ·· X (p - 1) + · · · + 1 X 2 X 3 X ·· · X (p - 2)
(p - 1)!
Suponhamos que conseguimos mostrar que este numerador é divi-
sível por p 2 • De que modo isto nos ajuda a mostrar que o numerador
reduzido é também divisível por p 2 ? Afinal, o que ê o numerador re-
duzido? É o mesmo que tínhamos antes após cance1armos os fatores
comuns com o denomin'ador. Este cancelamento pode destruir a divi-
sibilidade por p 2 ? Sim, se for cancelado algum múltiplo de p. Mas não
há como cancelar tais múltiplos, pois o denominador é primo com p (p
é primo, e (p- 1)! exprime-se como produto de números menores que
p). Aha! Isto quer dizer que só temos que provar que nosso impressi-
cmante numerador é divisível por p 2 , o que é melhor do que trabalhar
40 Exemplos da teoria dos números

com o outro numerador, porque agora dispomos de uma equação para


resolver:

2 X 3 X··· X (p - 1) + 1 X 3 X ··· X (p - 1) + · · ·
· · · + 1 x 2 x 3 x · · · x (p - 2) = O (mod p 2 ).
(Retomamos a aritmética modular, que usualmente fornece a melhor
forma de mostrar que um dado número divide outro. Contudo, quando
a pergunta em causa envolve mais do que uma questão de divisibilidade
- por exemplo, se envolve todos os divisores de um dado número -
então há outras técnicas que podem ser melhores.)
Temos agora uma equação, mas falta simplificá-la, pois é muito
confusa. O que temos no membro esquerdo é uma soma indetermi-
nada de produtos indeterminados (este indeterminado significa apenas
que tais expressões contêm reticências). Mas podemos representar es-
ses produtos indeterminados de forma mais elegante: cada um deles
contém basicamente todos.º~ fatores de 1 a p - 1 exceto um deles,
digamos i, e exprime-se de forma compacta como (p-1)!/i; é legítimo
dividir por i (mod p 2 ) porque i é primo com p2 : Assim, nosso objetivo
agora é mostrar que

_(p_-_1_)! + _(p_-_1_)! + _(p_-_1_)! + ... + _(p_-_1_)! = O (mod p2)


1 2 3 p-1

Fatorando, obtemos

(p - 1) ! [ -1 + -1 + -1
1 2 3 p-1
l
+ · · · + -1- = O (mod
.
p 2) . (6)

(Estamos a lidar com módulos, e portanto um número como 1/2 é


equivalente a algum inteiro. Por exemplo, 1/2 = 6/2 = 3 (mod 5).)
Observemos nossa equação: ela é da forma
.
(fator)(fator) = O (mod p 2 ).
2.3 Somas de potências 41

Se não fosse a aritmética modular, poderíamos dizer de imediato


que um dos fatores é O. Em aritmética modular, podemos dizer prati-
camente o mesmo, mas com algmJ1 cuidado. O primeiro fator, (p-1)!,
é, por sorte, primo com p 2 (por ser primo com p), e portanto podemos
descartá-lo. A conclusão é que (6) é equivalente a

!1 + !2 + !3 + · · · +-1-
p-1
= O (mod p 2 ).
-
(Isto parece muito nossa equação original, com a diferença de es-
tarmos considerando toda a soma e não apenas o numerador. Mas
não podemos saltar de uma formulação para a outra sem tomarmos
cuidados. As complicações em cima eram necessárias.)
Estamos reduzidos a uma questão em aritmética modular de as-
pecto relativamente benigno. Mas para onde devemos avançar? Tal-
vez um exemplo nos ajude. Considerando o mesmo exemplo que nos
é dado no enunciado - ou seja, p = 5 - , temos
1 1 1 1 .
1 + 2 + 3 + 4 = 1 + 13 + 17 + 19 = O (mod 25),

como queríamos. Mas por que razão isto funciona? Os números 1, 13,
17 e 19 parecem ser aleatórios, mas dão a soma certa como p'or magia.
Talvez tenha sido um golpe de sorte. Tentemos o 7:
1 1 1 1 1 1 ·
1 + 2 + 3 + 4 + 5 + 6 = 1 + 25 + 33 ~ 37 + 10 + 41 = O (mod 49).

Outra vez a mesma sorte! Como é que isto funciona? Não é claro
como é que tudo se cancela (mod p 2 ). Talvez, tendo em conta nosso
objetivo (b), possamos primeiro verificar a equação (mod p); ou seja,
vamos primeiro mostrar. que
1 1 1 1
-1 + -2 + -3 + · · · + - - = O (mod
p-1
p). (7)'

Se outra vantagem isto não tiver, sempre nos mantém ocupados.


(Além do mais, se não conseguimos resolver o problema para o módulo
p, de modo nenhum podemos resolvê-lo para o módulo p 2 .)
42 Exemplos da teoria dos números

Acontece que o problema simplificado (7) é muito mais fácil de


resolver que o original. Por exemplo, quando p = 5, temos

enquanto que, para p = 7, temos


1 1 1 1 1 1
1 + 2 + 3 + 4 + 5 + 6 (mod 7) = 1 + 4 + 5 + 2 + 3 + 6 (mod 7)
= 1 + 2 + 3 + 4 + 5 + 6 (mod 7)
= O (mod 7).
Vemos daqui surgir um padrão: os inversos 1/1, 1/2, ... , 1/(p-1)
(mod p) parecem cobrir todos os possíveis restos módulo p exatamente
uma vez. Na equação acima, por exemplo, a soma 1 + 4 + 5 + 2 + 3 + 6
pode ser reordenada para dar 1 + 2 + 3 + 4 + 5 + 6, que é O. Para
confirmarmos com um exemplo mais extenso, p = 11 dá
1 1 1
1 + 2 + .. · + 10 (mod 11) =1. +_ 6 + 4 + 3 +. 9 + 2 + 8 + 7 + 5 + 10 (mod 11)
=1 + 2 + 3 + 4 + 5 + 6 + 7 + 8 + 9 + 10 (mod 11)
=Ü (mod 11).
Esta tática de rearrumar os inversos desta forma ordenada funci-
ona bem para o módulo p, mas não é fácil adaptá-la ao módulo p 2 •
Em vez de tentarmos ajustar à força bruta uma peça quadrada em
um buraco circular (embora isso se possa fazer se empurrarmos com
ânimo suficiente), é preferível procurarmos uma peça que seja mais ar-
redondada. Nossa tarefa agora é portanto encontrarmos outra prova
de que 1/1 + 1/2 + 1/3 + · · · + 1/(p - 1) = O (mod p); uma prova que
possa ser generalizada, ainda que parcialmente, para o módulo p 2 .
Está na hora de apçlarmos à experiência que já temos neste tipo·
de problemas. Por exemplo, se acabamos agora mesmo de resolver o
Problema 2.6, sabemos que a simetria ou a antissimetria nos podem
ser úteis, especialment.e em aritmética modular. Na resolução de (7)
podemos tornar a som~ mais antissimétrica se substituirmos -1 por
2.3 Somas de potências 43

p - l, -2 por p - 2, etc., obtendo assim


111 1 111 1 1 1
-+-+-+···+-- = - + - + - + · · · + - + - + - (mod p).
1 2 3 p- l 1 2. 3 -3 -2 -1
Podemos agora emparelhar os termos e cancelá-los (não há termo
central que fique sem par, pois pé ímpar). Será possível fazer o mesmo
com o módulo p 2 ?
A resposta é quase. Quando resolvemos ·o problema para o módulo
p, emparelhamos 1/1 com 1/(p - 1), 1/2 com 1/(p - 2), e assim por
diante. Quando tentamos o mesmo emparelhamento com o módulo
p 2 , o que obtemos é isto:

1 1 1 1
-+- + - + ... + - -
1 2 3 p-1

=( +pt ~ l) + ( 1+ p ~ 2) + ... + ( (p _ \) /2 + (p + \) /2)


p + p +···+ p
lx(p-1) 2x(p-2) (p-1)/2x(p+l)/2

1 l 1 ]. 2
- [ + +···+ mod
-P lx(p-1) 2x(p-2) (p-1)/2x(p+l)/2 ( p)
A primeira vista, parece que complicamos ein vez de simplificar.
Mas lucramos um fator p no membro direito, o que é muito importante.
Agora, em vez de termos que mostrar que

(certa expressão)= O (mod p2 ),

temos que lidar com um·a coisa do tipo

p x (certa expressão)= O (mod p 2 ),

que é equivalente a

(certa expressão) = O (mod p).


44 Exemplos da teoria dos números

Em outras palavras, estamos reduzidos a trabalhar com o módulo p


em vez do módulo p 2 . Alcançamos assim nosso objetivo (b): reduzir a
questão a uma outra num módulo menor, o que vale bem um pequeno
aumento de complexidade.
E o aumento de complexidade, como rapidamente se vê, é apenas
ilusório, pois o módulo p permite livrarmo-nos de muito mais termos
do que permitiria o módulo p 2 • Neste momento, só falta ver que
1 1 1
1 x (p - 1) + 2 x (p - 2) + ... + (p - 1)/2 x (p + 1)/2 = O (mod p).
Mas p - l é o mesmo que -1 (mod p), p - 2 o mesmo que -2
(mod p), e assim por diante, de forma que a equação fica reduzida a
1 1 1
_ 12 + _ 22 + · · · + _ ((p- l)/ 2)2 = O (mod p)

ou, equivalentemente, a
1 1 1
12 + 22 + .. : ~ ((p- 1)/2)2 = O (mod p).

Esta equação não é totalmente má, tirando que a soma à esquerda


termina num ponto pouco usual (em 1/ ((p __: 1)/2) 2, em vez de, por
exemplo, terminar em 1/(p - 1)2, o que seria mais natural). Mas,
usando (-a) 2 = a 2 , podemos duplicar a soma, obtendo
1 1 1
!2+2 +···+ ((p-1)/2) 2
2

1 [ 1 1 1
= 2 12 + 2_2 + ... + ( (p - 1) / 2) 2

1 1 1 ]
+ (-1)2 + (-2)2 + ... + (-(p - 1)/2)2 (mod p)

l[l
=- - 1 + ··· + --
+ -- 1 - ] (mod p).
2 12 22 · . (p-1) 2
2.3 Somas de potências 45

Assim, mostrar que 1/1 2 + 1/2 2 +- ••+ 1/ ((p-1)/2) 2 = O(modp)


é equivalente a mostrar que 1/1 2 + 1/22 + · • •+ 1/(p-1) 2 = O(mod p).
Esta segunda versão é preferível. por ser mais simétrica. (A simetria
é boa para guardar - até que se possa extrair dela todo o proveito -
enquanto que a antissimetria é boa para cancelar.)
Resta-nos então demonstrar que

1 1 1
-+-+··•+---=O
12 22 (p-1)2
(mod p) (8)

para ficarmos com nossa questão resolvida. Taticamente, esta formu-


lação é muito melhor do que a original, que envolvia numeradores e
a divisibilidade por p 2, condição muito mais forte (e portanto mais
difícil de demonstrar) do que a simples divisibilidade por p.
Já atingimos nossas metas parciais, e reduzimos a dificuldade da
questão a níveis aceitáveis. Mas por onde devemos prosseguir? Bom,
a pergunta parece estar estreitamente relacionada com o problema
(7) que consideramos atrás. Mas não estamos a andar em círculos.
Nosso objetivo atual (8) resolve o problema original, ao passo que (7)
era uma versão mais simples do problem~ e não mais que uma questão
lateral. Em vez de andarmos em círculos, temos progredido em espiral
e estamos cada vez mais perto da solução. Já demonstramos (7): será
que podemos demonstrar (8) pelos mesmos métodos?_
De fato estamos com sorte, pois houve dois métodos que usamos
para mostrar (7): o primeiro foi o reordenamento dos inversos, o se-
gundo o cancelamento das parcelas duas a duas. Infelizmente, o se-
gundo método não funciona tão bem com (8) como com (7), princi-
palmente por causa dos quadrados nos denominadores, que produzem
simetria em vez de antissimetria; mas o primeiro método é promissor.
46 Exemplos da teoria dos números

Tomemos novamente p = 5 (aproveitando assim cálculos já feitos):


1 1 1 1
l2 + 22 + 32 + 42 = 12 + 32 + 22 + 42 (mod 5)

=1 2 +2 2 +3 2 +42 (mod5)

= O (mod 5).
O modo como isto funciona com p = 5 mostra o caminho no caso
geral. Parece, à luz dos exemplos anteriores, que as classes dos resíduos
1/1, 1/2, 1/3, ... , 1/(p- 1) (mod p) são apenas uma reordenação dos
números 1, 2, 3, ... , p- 1 (mod p); daremos uma demonstração desse
fato no final desta discussão. Assim sendo, podemos afirmar que os
números 1/ 12, 1/ 22, ... , 1/ (p - 1) 2 são simplesmente uma reordenação
de 12 , 22 , ... , (P. - 1)2. Em outras palavras,
1 1 1 1 2 2 2· 2
12 + 22 + 32 + · · · + (p _ 1) 2 = 1 + 2 + 3 + · · · + (p - 1) (mod p).

Esta expressão é mais cômoda de manipular, pois nos livramos


dos inversos, que são uma chatice quando tentamos calcular somas.
De fato, podemos livrar-nos também da soma, usando a conhecida
fórmula
2 2 2 n (n + 1) (2n + 1)
1 + 2 +···+n = - - -- - - 6
(que se prova facilmente por indução). A fórmula (8) fica assim redu-
zida a
(p - 1) p (2p - 1)
6 = O (mod p),
e é fácil ver que esta igualdade é verdadeira quando pé um primo maior
do que 3 (porque nes~e caso o fator (p - 1) (2p - 1) / 6 é inteiro).
E é tudo. Fomos reduzindo o problema original a formulações cada
vez mais.simples, até que ele se desfez em nada em nossas mãos. Foi
uma estirada considerável, mas as vezes é este o único modo de resolver
questões muito complicadas como esta: por redução passo a passo.
2.3 Somas de potências 47

Vamos à tal prova de que os inversos 1/1, 1/2, 1/3, ... , 1/(p - 1)
(mod p) são uma permutação dos números 1, 2, 3, ... , p- 1 (mod p):
isso é equivalente a dizer que ca.da classe não-nula de resto (mod p) é
a inversa de uma e uma só classe não-nula (mod p), o que é óbvio.

Exercício 2.5. Seja n ~ 2 um inteiro. Mostre que 1/1+1/2+· · ·+1/n


não é um inteiro. (Sugestão: você pode usar o postulado de Bertrand
- que é na verdade um teorema, e garante que, para cada natural n,
há pelo menos um primo entre n e 2n. ) 2

Exercício 2.6. (*) Seja p um primo, e seja k um inteiro positivo não


divisível por p-1. Mostre que lk+2k+3k+· · ·+(p-l)k é divisível por
p. (Sugestão: dado que k pode ser par, nem sempre podemos usar o
truque do cancelamento. Mas o método da reordenação já funciona.
Seja a um gerador de 71.,/pZ: então ak-=/=- O (mod p) sempre que k não
seja múltiplo de p-1. Calcule agora ak+(2at +(3at +· · ·+((p - l)at
(mod p) de dois modos diferentes. )3

2 N. do T. Eis uma resolução que não usa o postulado de Bertrand. Seja 2m a


maior· potência de 2 que não excede n. A soma das restantes parcelas é da forma
a/2kb, onde a e b são ímpares e k < m (pode ser b = I ou k ~ O). Quando a isto"
somamos I/2m, obtemos (2m-ka + b) /2mb, que não é inteiro pois é o quociente
<le um ímpar por um par.
3 N. do T. Em alternativa, como as p-1 primeiras potências de a (ou seja, 1, a,

a 2 , ... , aP- 2 ) dão todos os resíduos módulo p, a expressão 1k + 2k +· •••+ (p - l)k


( mod p) pode ser vista como á soma de uma progressão geométrica de razã.o ak.
48 Exemplos da teoria dos números
CAPÍTULO 3

Exemplos da álgebra e da análise

Há um sentimento inesmpável ... de que essas fórmulas matemáticas


têm existência autônoma e uma inteligência própria... de que elas
são mais sábias do que nós, mais sábias até do que quem as
descobriu... de que o que tiramos dela5 é mais do que aquilo que de
início nelas foi posto.
Heinrich Hertz, citado por F. J. Dyson

A· álgebra é aquilo a que a maioria das pessoas associa a mate-


mática. E isto, num certo sentido, justifica-se. A matemática é o
estudo de entidades abstratas, que podem ter caráter numérico, lógico
ou geométrico, e obedecem a um conjunto de axiomas cuidadosamente
escolhido. E a álgebra básica é, na prática, a coisa mais simples com
alp;um conteúdo que satisfaz essa definição de matemática. A cerca de
50 Exemplos da álgebra e da análise

dúzia de postulados que a governam são suficientes para a tornar atra-


entemente simétrica. A título de exemplo, minha identidade algébrica
favorita é

Isto significa, entre outras coisas, que a soma dos primeiros cubos é
sempre um quadrado; por exemplo, 1 + 8 + 27 + 64 + 125 = 225 = 25 2 •
Existe, porém, mais do que uma álgebra. A álgebra básica é o es-
tudo dos números e de suas operações: adição, subtração, multiplica-
ção e divisão. A álgebra das matrizes, por exemplo, faz praticamente
o mesmo, mas trata com conjuntos de números em vez de números
simples. Outras álgebras usam uma grande diversidade de operações
sobre toda a espécie de números - mas o que é por vezes surpreendente
é como essas álgebras tendem a ter muitas das propriedades da álge-
bra normal. Por exemplo, uma matriz quadrada A pode satisfazer,
sob certas condições, a equação algébrica

A álgebra fornece a base fundamental de boa parte da matemá-


tica aplicada. Problemas em mecânica, economia, química, eletrônica,
otimização e outros ramos têm sua resposta na álgebra e no cálculo
diferencial, que é uma forma avançada de álgebra. De fato, a álgebra
é tão importante que a maioria
.
de seus segredos já foi descoberta
..
- e
por isso pode ser incluída sem risco no currículo liceal. Contudo, ela
ainda guarda aqui e ali algumas jóias por revelar.

3.1 Análise de funções


A análise é também um assunto intensamente explorado, e sua
generalidade é tanta c..omo a da álgebra. A análise é, em suma, o estudo
das funções e de suas propriedades. Quanto mais complicadas forem
3 .1 Análise de funções 51

essas propriedades, mais avançada é a análise. A forma mais básica


da análise é o estudo de funções que satisfaçam simples condições
algébricas. Como exemplo, consjderamos uma função f(x) tal que

fé contínua, f(O) = 1, e f(m + n + 1) = f(m) + f(n)


(9)
para todos os números reais m e n,

e depois deduzimos algumas propriedades da função. Neste caso, por


exemplo, há exatamente uma função nestas condições, que é f(x) =
x + 1; deixamos a verificação como exercício. Problemas como este
são um bom modo de treinar o raciocínio matemático, pois, como há
apenas um ou dois dados de que nos podemos servir, deve ser claro que
direção tomar. É uma espécie de matemática de bolso, onde, em vez
de lidarmos com três dúzias de axiomas e alguns milhares de teoremas,
nosso arsenal se reduz a um punhado de axiomas (ou seja, de dados).
Mas ainda assim pode ter surpresas.

Exercício 3.1. Seja f uma função dos números reais· em· si próprios
que satisfaz (9). Mostre que, para todos os números reais x, f(x) =
x + 1. (Sugestão: comece por provar a afirmação para .x inteiro,
depois para x racional, é finalmente para x real.)

Problema 3.1 (Greitzer, 1978, p. 19). (*) Suponha que f é uma


função dos inteiros positivos nos inteiro~ positivos, e que f(n + 1) >
J(f(n)) para todo o n. Mostre que f(n) = n para todos os inteiros
positivos n.

Esta inequação parece ser insuficiente para mostràr o que nos pe-
dem.· Afinal, como podé uma desigualdade implicar uma igualdade?
Outros problemas do mesmo tipo (como o do Exercício 3.1) envolvem
equações funcionais, e são mais fáceis de manejar porque podemos
aplicar várias substituições e coisas do gênero, de modo a moldar gra-
dualmente os dados em formas mais tratáveis. Este problema parece
ser completamente diferente.
52 Exemplos da álgebra e da análise

Contudo, ao lermos o enunciado com cuidado, vemos que, ao con-


trário da maioria das questões que envolvem equações funcionais (em
que as funções tomam valores reais), a função em causa só toma valores
inteiros. Podemos tirar daí proveito imediato fortalecendo a desigual-
dade:
J(n + 1) 2: J(f(n)) + l. (10)
Vejamos agora o que podemos daqui deduzir. O método usual de
atacar estas desigualdades é atribuir alguns valores à variável; come-
cemos por n = 1:
f(2) 2: f(f(l)) + 1.
À primeira vista isto não nos diz muito sobre f(2) ou f(l), mas
o +1 à direita sugere que f(2) não pode ser muito pequeno. Como f
toma valores inteiros positivos, J(f(l)) é pelo menos 1, logo f(2) é no
mínimo 2. Como temos que provar que f(2) é exatamente 2, talvez
estejamos na pista certa. (Usemos sempre as· táticas que nos aproxi-
mem do objetivo, a menos que todas as abordagens diretas estejam
esgotadas. Só então deverrios pensar em desviar o rumo ou - por vezes
- em recuar.)
Será que conseguimos mostrar que f(3) é no mínimo 3? Podemos
usar (10) novamente, obtendo J(3) 2: J(f(2)) + 1. Repetindo o ar-
gumento em cima, podemos dizer que f(3) é no mínimo 2. Mas não
podemos dizer algo mais forte? Dissemos atrás que J(f(l)) é pelo me-
nos 1. Talvez J(f(2)) seja pelo menos 2. (Na verdade, como sabemos
secretamente que f(n) há-de· ser igual a n, sabemos que f(f (2)) é 2
- mas não podemos ainda usar esse fato, pois é trapaça fazermos uso
do que estamos tentando demonstrar.) Com esse objetivo em mente,
aplicamos de novo ( 10):

f(3) 2: J(f(2)) + 1 2: J(f(f(2) - 1)) + 1 + 1 2: 3.


A substituição em.( 1O) de n por f (2) -1 é legítima, pois já sabemos
que f(2) - 1 é pelo menos 1.
3.1 Análise de funções 53

Parece portanto que conseguimos deduzir que f(n) ~ n. Visto que


usamos o fato de f(2) ser pelo menos 2 para provar que f(3) é pelo
menos 3, a prova do caso geral tresanda a indução.
Mas essa indução tem sua sutileza. Consideremos o caso a seguir, o
de provar que f(4) ~ 4. Já sabemos que f(3) ~ 3, mas gostaríamos de
deduzir que J(f(3)) ~ 3 para daí tirarmos que J(f(3)) + 1 ~ 4. Para
isso, seria bom termos à mão uma proposição do tipo se n ~ 3, então
f (n) ~ 3. O mais fácil é incluir essa afirmação na prova por indução
que estamos tentando fazer. Mais precisamente, vamos mostrar o
seguinte:
Lema 3.1. f(m) ~ n para todo o m ~ n.
Demonstração. Usamos indução em n.

• Caso inicial n = l. Isto é óbvio: dizem-nos que f(m) é um


inteiro positivo, logo J(m) é no mínimo 1.

• Passo de indução. Suponhamos que o lema é· válído para n;


vamos tentar provar que f (m) ~ n + l para todo o m ~ n + l.
Para cada m ~ n + l, podemos usar (10) para obter f(m) ~
J(f(m- l)) + 1. Como m-1 ~ n, a hipótese de indução nos dá
f (m - 1) ~ n. Podemos ir mais longe: visto que f (m - 1) ~ n,
temos J(f(m~ 1)) ~ n, também pela hipótese de indução. Logo,
f(m) ~ J(f(m - 1)) + 1 ~ n + ·1, e o passo de indução está
concluído. ■

Se tomarmos o caso m = n no lema 3.1, atingimos nosso objetivo


pardal:
f(n) ~ n para todos os inteiros positivos n. (11)"
E agora? Como é hábito em todas as questões com equações funci-
onais, o que temos a fazer, agora que dispomos de um novo resultado,
ó jogar com ele e tentar combiná-lo com os anteriores. Nosso único
resultado anterior é (10); e por isso podemos inserir em (10) nossa
54 Exemplos da álgebra e da análise

nova relação (11). A única relação útil que obtemos é

f(n + 1) ~ J(f (n)) + 1 ~ f(n) + 1,

que resulta de substituirmos n por f(n) em (11). Em outras palavras,

f(n + 1) > f(n).

Esta fórmula é muito útil, pois significa que f é uma função cres-
cente (fato nada óbvio a partir de (10)). Isto quer dizer que f(m) >
f(n) se e só sem> n; logo, nossa equação original, que é

f(n + 1) > J(f(n)),

pode ser reformulada como

n + 1 > f(n)

e isto, juntamente com (11·), ·prova o que queríamos.

Problema 3.2 (Australian Mathematics Competition, 1984, p. 7).


Suponha que a função f está definida no~ inteiros positivos, toma
valores inteiros, e tem as seguintes propriedades:

(a) f (2) = 2;

(b) f(mn) = f(m)f(n) para todos os inteiros positivos m·e n;


(c) f(m) > f(n) sem> n.

Calcule !(1983) (justificando a resposta, obviamente).

Temos portanto que encontrar um valor específico de f. O melhor


é tentarmos calcular todos os valores de f, e não apenas !(1983).
(1983 foi apenas o ano_ em que a questão foi posta.) Isto SllJHHHlo,
evidentemente, que há uma só funçã.o .f nas condições cl<):--writas. Ma:--;
3.1 Análise de funções 55

a pergunta tem implícito o fato de só haver um valor possível para


!(1983) (caso contrário a resposta não seria única); e, dado o caráter
arbitrário de 1983, é razoável coJ}jecturarmos que há apenas uma tal
função f.
Quais são então as propriedades de f? Sabemos que f(2) = 2;
aplicando (b) repetidamente, vemos que f(4) = f(2)f(2) = 4, J(8) =
f(4)f(2) = 8, etc. De fato, uma indução fácil mostra-nos que f(2n) =
2n para todo o n. Assim, f (x) = x quando x é uma potência de
2. Talvez seja f(x) = x para todo o x. De fato, se regressarmos a
(a), (b) e (c) vemos que isto funciona: a função f(x) = x obedece a
essas condições. Se acreditamos que (a), (b) e (c) determinam uma só
função, então ela terá que ser f(x) = x. Podemos portanto preferir
demonstrar a seguinte afirmação, que é mais geral mas também mais
clara:
A função identidade (ou seja, f (n) = n para todo o n) é a única
função que está definida nos inteiros positivos, toma valores inteiros e
satisfaz (a), (b) e (c).
É preciso então mostrar que, se f satisfaz (a), (b) e (c), então
J(l) = 1, J(2) = 2, J(3) = 3 e assim por diante. Tentemo$ primeiro
provar que f(l) = 1 (para ganharmos familiaridade com equações fun-
cionais, devemos primeiro experimentar pequenos valores). Sabemos
por (c) que J(l) < J(2), e é nos dito que J(2) = 2; logo J(l) é menor
do que 2. De (b), pondo n = 1 e m = 2,' tiramos
f(2) = f(l)f(2),
e portanto
2 = 2f(l).
Isto implica que f(l) tem que ser 1, como queríamos.
Já temos f(l) = 1 e J(2) = 2. E quanto a J(3)? (a) não nos
ajuda, e (b) só exprime f(3) à custa de outros números como f(6) ou
.f (9) . (c) dá-nos
f(2) < f(3) < f(4)
56 Exemplos da álgebra e da análise

mas J(2) é 2 e f(4) é 4, de modo que

2 < f(3) < 4.

Mas o único inteiro entre 2 e 4 é o 3, e portanto J(3) tem que ser 3.


Isto nos dá uma ideia: f(3) teve que ser 3 só porque se tratava
de um inteiro (o que é análogo ao que se fez com a desigualdade
f(n + 1) > J(f(n)) no problema anterior). Sem esta restrição, J(3)
poderia ter sido 2,1 ou 3,5 ou sabe-se lá o que. Vejamos se podemos
usar mais vezes esta ideia.
Já sabemos que f(4) é 4; tentemos calcular f(5). Usando (c) na
esperança de repetir o que fizemos com f(3), obtemos

f(4) < f(5) < J(6).

f(4) é 4, mas quanto é f(6)? Nada a tem~r: 6 é 2 vezes 3, logo


f(6) = f(2)f(3) = 2 x 3 = 6. J(5) está assim entre 4 e 6, e tem
que ser 5. Parecemos estar avançando.bem: conseguimos já todos os
valores de f(n) até n = 6.
Uma vez que nos baseamos em resultados anteriores para chegar
aos novos, a prova do caso geral cheira fortemente a indução. E, como
estamos usando não apenas um resultado anterior mas vários deles,
vamos provavelmente precisar de usar indução forte.
Lema 3.2. f(n) = n para todo o n.
Demonstração. Usamos indução forte. Primeiro verificamos o caso
inicial: será que f(l) = 1? Já mostramos que sim. Suponhamos
agora que m 2: 2, e que f(n) = n para todos os n menores do que
m. Queremos mostrar que f(m) = m. Observando alguns exemplos,
depressa concluímos que temos que separar dois casos: m par e rri
ímpar.
Caso 1: m é par. Podemos então escrever m = 2n para algum
inteiro n; n é menor. do que m e, pela hipótese de indução, temos
f(n) = n. Daí f(m) =°J(2n) = f(2)J(n) = 2n = m, como queríamos.
3.1 Análise de funções 57

Caso 2: m é ímpar. Desta vez pomos n = 2m + 1. Por (c),


f(2n) < J(m) < J(2n + 2). Pela hipótese de indução, f(2n) = 2n e
J(n + 1) = n + 1, pois tanto ~n como n + 1 são menores do quem.
Por (b), J(2n + 2) = J(2)J(n + 1) = 2(n + 1) = 2n + 2, de modo que
nossa desigualdade fica

2n < J(m) < 2n + 2,


e portanto f (m) = 2n + 1 = m, como queríamos. Em qualquer dos
casos, o passo de indução está completo. ■
A indução forte obriga assim f(n) a ser igual a n. A resposta à
pergunta incial é pois que !(1983) é 1983.
Exercício 3.2. Mostre que o problema 3.2 pode ainda ser resolvido
se substituirmos (a) pela seguinte condição mais fraca:
(a') J(n) = n para pelo menos um n ~ 2.
Exercício 3.3. (*) Mostre que o problema 3.2 pode ainda ser resol-
vido se admitirmos que f toma valores reais, e não· apenas inteiros.
(Sugestão: tente primeiro mostrar que J(3) = 3, comparando J(2n)
com J(3m) para vários inteiros m e n.) Como desafio ad~cional, re-
solva o Problema 3.2 com esta nova hipótese e com (a') em substituição
de (a).
Exercício 3.4 (Olimpíadas Internacionais de Matemática, 1986, ques-
tão 5). (**) Encontre (caso existam) todas as funções sobrejetivas que
levam os números reais não-negativos em si próprios e tais que:
(a) J(xf(y))J(y) = J(x + y) para todos os x, y ~- O;

(b) J(2)=0;

(c) J(x) =/- O para todo o O:::; x < 2.


(Sugestão: a primeira condição envolve produtos de valores da fun-
<;ão, e as restantes condições falam de a função tomar (ou não tomar)
o valor O. O que é que se pode afirmar quando um produto é zero?)
58 Exemplos da álgebra e da análise

3.2 Polinômios
Muitos problemas algébricos têm a ver com polinômios de uma ou
mais variáveis, e por isso fazemos uma pausa sobre eles para recapitu-
larmos algumas definições e resultados.
Um polinômio de uma variável é uma função, digamos f (x), da
forma

ou, para sermos mais formais,


n
f(x) = L aixi.
i=O

Os coeficientes· ai são constantes (que neste livro serão sempre nú-


meros reais), e supomos que an não é O. Chamamos a n o grau de f.
Polinômios de mais que uma variável,. admitamos que três para fins
de exemplificação, não têm•uma forma tão simpática como a de um
polinômio unidimensional, mas são, apesar diss,o, bastante úteis. Seja
como for, f(x, y, z) é um polinômio de três variáveis se tiver a forma

J(x, y, z) = L ak,l,mXkylzm,
k,l,m

onde os ak,l,m são constantes (reais), a soma percorre todos os índices


não-negativos k, l, m tais que ·k + l + m ~ n, e pelo menos·um dos
ak,l,m com k + l + m = n é diferente de O. A este n chamamos também
o grau de f; os polinômios de grau 2 são quadráticos, os de grau 3
cúbicos, e assim por diante. Se o grau for O, o polinômio é dito trivial
ou constante. Se todof3 os coeficientes não-nulos ak ,I,m satisfizerem
k + l + m = n, f é homogêneo. Os polinômios homogêneos têm a
propriedade de ser
3.2 Polinômios 59

para todos os x 1 , x 2 , ... , Xm, t. Por exemplo, x 2 y + z 3 + xz é um


polinômio de três variáveis (x, y e z) e de grau 3; mas não é homogêneo
porque o termo xz tem grau 2. .
Dizemos que um polinômio f de m variáveis se fatora em dois
polinômios p e q se for J(x1, ... , Xm) = p(x1, ... , Xm) · q(x1, ... , Xm)
para todos os x 1 , ... , Xm; p e q são chamados então os fatores de f. É
fácil de ver que o grau de um polinômio é igual à soma dos graus de
seus fatores. Um polinômio é dito irredutível se não puder ser fatorado
em fatores não-triviais.
As raízes de um polinômio f(x 1, ... , Xm) são os valores de
(x 1 , ... , Xm) para os quais J se anula, de modo que f(x 1 , ... , Xm) = O.
Polinômios de uma só variável têm no máximo um número de raízes
igual a seu grau; de fato, se contarmos multiplicidades e raízes com-
plexas, o número de raízes é exatamente igual ao grau. Por exemplo,
as raízes de um polinômio quadrático f (x) = ax 2 + bx + e são dadas
pela conhecida fórmula resolvente:

-b ± ✓b 2 - 4ac
X=
2a
Também há fórmulas· para as raízes dos polinômios cúbicos e quár-
ticos, mas são muito mais confusas do que esta e, na prática, não
muito úteis. Quando passamos aos polinômios de grau 5 ou superior,
já não existe qualquer fórmula elementàr! E os polinômios de duas
ou mais variáveis são ainda piores: em regra têm uma infinidade de
raízes.
As raízes de um fator são um subconjunto das raízes do polinômio
original: esta parte da informação pode ser útil para decidir se um
dado polinômio divide outro. Em particular, x - a divide J(x) se e só•
se f(a) = O, visto que a é uma raiz de x - a; mais geralmente, para
qualquer polinômio f (x) de uma variável e para qualquer número real
t, x - t divide f(x) - J(t).
Vamos agora a alguns'problemas que envolvem polinômios.
60 Exemplos da álgebra e da análise

Problema 3.3 (Australian Mathematics Competition, 1987, p. 13).


Sejam a, b, e números reais tais que
1 1 1 1
-+-+-=---. (12)
a b e a+b+c
onde todos os denominadores são não-nulos. Mostre que
1 1 1 1
-+-
5
+5- = - - - -5 (13)
a5 b c (a+b+c) .
A primeira vista isto parece simples. Somos informados de um
único fato, e portanto deve haver uma sequência fácil de passos lógi-
cos que nos conduza ao resultado desejado. Bom, uma tentativa inicial
de obter a segunda equação da primeira poderá ser elevar ambos os
membros de (12) à quinta potência, o que produz algo semelhante ao
pretendido, mas com uma profusão de termos complicados no mem-
bro esquerdo. Não parece existir qualquer outra manipulação óbvia.
Quanto à abordagem direta estamos cop_versados.
Num segundo relance, ·a· primeira equação tem um ar suspeito,
lembrando aquelas igualdades que os alunos dq ensino médio são aler-
tados para não usar por serem, em geral, .falaciosas. Isto nos dá uma
primeira pista a sério: a primeira equação deve restringir bastante os
valores de a, b e e. Pode ser proveitoso reformular a equação (12).
Reduzir ao mesmo denominador parece ser um bom começo. Com-
binando os três inversos no membro esquerdo, obtemos
ab+ bc+ ca 1
abc a+b+c
e, por multiplicação cruzada,

Neste momento talvez nos lembremos de algumas desigualdades


que poderíamos aqui usar: a de Cauchy-Schwarz, a das médias arit-
mética e geométrica, et·c. (Hardy, 1975, pp. 33-34). Isso não s<'ria
3.2 Polinômios 61

muito má ideia se a, b, e só pudessem ser positivos, mas não existe


esta restrição: de fato, (12) seria impossível se a, b e e fossem posi-
tivos, pois então 1/(a + b + e) seria menor do que qualquer um dos
inversos no membro esquerdo.
Visto que (14) é equivalente a (12) mas mais simples (pois não
contém inversos), poderíamos tentar deduzir (13) de (14). Mais uma
vez a abordagem direta não é exequível. Em regra, as únicas outras
maneiras de deduzir uma equação a partir de outra são demonstrar
um resultado intermédio ou proceder a alguma substituição útil. (Há
alternativas mais rebuscadas, como considerar (12) como definindo
uma superfície de nível da função (1/a) + (1/b) + (1/c) -1/(a + b+ e),
e usar o cálculo infinitesimal para determinar a forma e as propriedades
dessa superfície. Mas é melhor ficarmos inicialmente nas opções mais
simples.)
As substituições não parecem ser a coisa apropriada: as equações
(12) e (14) já são simples tal como estão, e as substituições não iriam
simplificá-las muito. Tentemos então demonstrar algum résultado in-
termédio. O melhor tipo de resultado intermédio é uma parametriza-
ção, já que ela pode ser inserida diretamente no resultado pretendido;
e um modo de paramefrizar é resolver a equação em relação a uma
das variáveis, digamos a. (14) não é fácil de resolver como equação em
a (a menos que estejamos dispostos a usar a fórmula resolvente), mas
o mesmo já não acontece com (12), e a'questão pode ser solucionada
se usarmos (12) para exprimir cada um dos a, b e e à custa dos outros
dois e deduzirmos daí um resultado intermédio (o qual, a propósito,
é equivalente ao resultado que apresento abaixo - nias teria que ser,
não?). Mas vou tentar uma outra coisa.
Se a parametrização falha, podemos simplesmente dar a (14) uma
forma melhor. As soluções de (14) são, na prática, as raízes do polinô-
mio ab2 +a2 b+ac2 +a2 c+bc2 +b2 c+2abc. A melhor forma de lidar com
raízes de um polinômio é fatorar esse polinômio (e vice-versa). Quais
são os fatores? Como sabemos que (14) deve de algum modo implicar
62 Exemplos da álgebra e da análise

(13), devemos estar relativamente confiantes na existência de alguma


forma manejável de (14) que nos leve a (13), e a única forma manejável
de um polinômio é tê-lo decomposto em fatores. Mas, para os encon-
trarmos, teremos que ir por tentativas. O polinômio é homogêneo,
logo seus fatores também o são. O polinômio é simétrico, logo seus
fatores devem ser simétricos uns dos outros. O polinômio é cúbico,
logo terá que haver um fator linear. Devemos agora tentar fatores da
forma a + b, a - b, a, a + b + e, a + b - e, e assim por diante. ( Coisas
como a + 2b também podem funcionar, mas não são tão apresentáveis
e, se for preciso, podem ficar para mais tarde.) Depressa notamos
que a = -b é uma raiz da cúbica (considerada como polinômio em
a), e que também b = -e e e= -a são raízes. Daí, pelo teorema da
fatoração, deduzimos que (14) se fatora como (a+ b) (b + e) (e+ a).
Isso significa que.. (12) é verdadeira se e só se a + b = O, b + e = O
ou e+ a = O. Inserindo cada uma destas possibilidades em (13), o
problema fica resolvido.

Exercício 3.5. Fatore a 3 +b + c


3 3 - 3abc.

Exercício 3.6. Determine todos os inteiros a,


b, e, d tais que a+
b +e+ d = O e a + b + c + d = 24 .. (Sugestão: não é difícil
3 3 3 3

adivinhar algumas soluções da equação, mas, para provar que tem


todas, substitua a primeira equação na segunda e fatore.)

A fatoração de polinômios, ou sua impossibilidade, é um campo


fascinante da matemática. Nosso próximo problema é instrutivo por-
que, para o resolvermos, recorremos a praticamente todos os truques
do catálogo.
Problema 3.4. (**) Mostre que nenhum polinômio da forma f(x) = ·
(x - a 1 )2(x - a2 ) 2 • • • (x - an) 2 + 1, onde os ai são inteiros distintos,
pode ser escrito como produto de dois polinômios não triviais, ambos
com coeficientes inteirQs. 1
1 N. do T. O re:-mltado {\ fal8o H<;rn a hipútese de os codicient<'H 11; s1•rc•111 dist i11t os,
3.2 Polinômios 63

Esta afirmação tem um caráter bastante geral: diz, por exemplo,


que o polinômio

(x - 1)2(x + 2) 2 + 1 ~ x 4 + 2x 3 - 3x 2 - 4x + 5

não pode ser fatorado noutros polinômios inteiros. Como podemos


provar isto?
Bem, suponhamos que f (x) pode ser fatorado em dois polinômios
inteiros não triviais, p(x) e q(x). Então f(x) = p(x)q(x) para todo o x.
Grande coisa. Mas recordemos que f tem uma propriedade especial:
é a soma de uma espécie de quadrado com 1. Como tirar partido
disso? Podemos pelo menos dizer que f(x) é sempre positivo (e até
que f (x) 2:: 1), mas isto, além de implicar que tenham o mesmo sinal,
não nos diz muito sobre p e q. Mas temos outro bocado de informação:
f não é apenas um qualquer quadrado mais um; trata-se do quadrado
de um produto de fatores lineares. Poderemos usar esses x - ai em
nosso proveito?
O melhor fator que podemos desejar é o O, porque ele faz com que
toda a expressão seja O. (De fato, também há ocasiões, como quando
queremos cancelar um fator, em que ele ser O é o pior que· nos pode
acontecer.) (x - ai) é O quando ... x é ai, e portanto eis uma ideia:
ponhamos ai no lugar de x. Obtemos

Regressando a p( x) e q( x), isto implica que

Que significa isto? Muito pouco, a menos que nos lembremos de


que p e q têm coeficientes inteiros, e que os ai são também inteiros. A
como mostra o exemplo n = 3, a 1 = a 2 = a 3 = O: nesse caso, x6' +1 = (x 2 +
1)(:r'1 - :r: 2 + 1).
64 Exemplos da álgebra e da análise

conclusão é que p(ai) e q(ai) são ambos inteiros. Temos assim que o
produto de dois inteiros é 1, o que só pode ocorrer quando ambos são
1 ou ambos são -1. Em escrita abreviada,

para todo i = 1, 2, ... , n.

Devemos aqui ter um pouco de cautela com a notação± sabemos


que p(a 1 ) e q(ai), por exemplo, têm o mesmo sinal, mas, pelo que
sabemos até agora, p(a 1 ) e p(a2 ) podem ter o mesmo sinal ou sinais
opostos.
Determinamos (a menos de sinais) os valores de p(a 1 ), ••. , p(an)
e q(ai), ... , q(an), e portanto cada polinômio está agarrado por n
pontos. Mas os polinômios com coeficiente líder ±1 têm um número
de graus de liberdade igual a seu grau2. Como pq = f, a soma dos
graus de p e de q é igual ao de f, que é 2n. Isto significa que um dos
polinômios, digamos p, tem grau quando muito n. Em suma, temos
um polinômio de grau no máximo n que está restringido em n pontos.
Com alguma sorte isto pode 'levar-nos a uma contradição, que é aquilo
que procuramos.
O que é que nós sabemos de um polinômio que tenha grau no
máximo n? Bem, tem no máximo n raízés. Sabemos alguma coisa
sobre as raízes de p? Como p é um fator de f, as raízes de p são
também de f. Quais são as raízes de f? Não há nenhuma! (Nenhuma
na reta real, pelo menos.) f é sempre positivo (de fato, nunca desce
abaixo de 1) e portanto não pode ter raízes. Por sua vez, isfo implica
que p não pode ter raízes. E o que acontece a um polinômio que não
tem raízes? Como nunca cruza o valor O, não pode mudar de sinal. Em
outras palavras, p é sempre positivo ou sempre negativo. Há portant(!
2 N.
do T. Em geral, o ~úmero de graus de liberdade de um polinômio é igual
ao número de seus coeficientes, que por sua vez é igual ao grau do polinômio mais
1. Em nosso caso, porém, o fato de ser j = p q limita a escolha do coeficiente líder
(coeficiente do termo de m~ior grau) em p e em q a 1 ou -1, e retira assim a cada
um deles um grau de liberdade.·
3.2 Polinômios 65

dois casos, mas podemos poupar algum trabalho se observarmos que


um deles implica o outro. De fato, se tivermos uma fatoração f(x) =
p(x)q(x), obtemos automaticamente outra: f(x) = (-p(x)) (-q(x)).
Assim, se p for sempre negativo, podemos considerar a outra fatoração
e trocá-lo por -p, que é sempre positivo.
Supomos então, sem perda de generalidade, que p é sempre posi-
tivo. Sabíamos já que p(ai) é 1 ou -1, e agora sabemos também que
p( ai) é positivo: logo, terá de ser igual a 1 para todo o i. E q( ai) é
forçado a ser igual a p(ai), e portanto q(ai) é também 1 para todo o
i. E agora?
Bem, p(x) e q(x) são obrigados a tomar o valor 1 pelo menos n
vezes. Isto pode ser reformulado em termos de raízes do seguinte
modo: p(x) - 1 e q(x) - 1 têm pelo menos n raízes. Mas p(x) - 1
tem no máximo grau n, pois o mesmo sucede com p( x). Logo, o único
modo de p(x) - 1 ter n raízes é que seu grau seja seja exatamente n.
Isto, por sua vez, faz com que p(x) tenha grau n, e portanto também
q(x) tem grau n. · ·
Recapitulemos o que já fizemos. Supusemos que f(x) = p(x)q(x);
p e q são ambos polinô~ios inteiros de grau n; e p(ai) = q(aí) = 1 (ou,
em alternativa, p(ai) - 1 = q(ai) - 1 = O) para todo o i. Dispomos
das raízes de p(x) - 1: são os ai, Não há outras porque p(x) - 1 não
pode ter mais do q11e n raízes. Isto implica que p(x) -1 seja da forma

e, de modo análogo, também

onde r e s são certas constantes. Para descobrirmos mais acerca de r


e s, lembremos quepe q são polinômios inteiros. O coeficiente líder
de p(:r) - 1 é r, e o de q(x) - 1 és; logo, ressão inteiros.
66 Exemplos da álgebra e da análise

Inserimos agora estas expressões para p e q em nossa fórmula ori-


ginal J(x) = p(x)q(x), obtendo

(x - a1)2(x - a2) 2 · · · (x - an) 2 + 1 = (r(x - ai)(x - a2) · · · (x - an) + 1)


x (s(x - a1)(x - a2) · · · (x - an) + 1).

Esta equação relaciona dois polinômios definidos explicitamente.


O melhor agora é comparar seus coeficientes.
Comparando os coeficientes de x 2 n, obtemos rs = l; e, como r e s
são inteiros, isto implica que seja r = s = l ou r = s = -1. Suponha-
mos primeiro que r = s = l. Nossa equação polinomial fica então

(x - a1)2(x - a2) 2 · · · (x - an) 2 + 1 = ((x - ai)(x - + 1)


a2) · · · (x - an)
x ((x - ai)(x - a2) · · · (x - an) + 1).

Efetuando o produto à direita e cancelando, temos

o que é um disparate (pois tem que valer p·ara todo o x). O caso
r = s = - l é análogo, e a prova ·está concluída.

Exercício 3.7. Mostre que nenhum polinômio da forma J(x) = (x -


ai)(x - a2 ) • • • (x - an) - l, onde os ai são inteiros distintos, pode
ser fatorado em polinômios inteiros de menor grau. (Sugest~o: se for
J(x) = p(x)q(x), considere a soma p(x) + q(x). Vale notar que esta
mesma estratégia poderia ser aplicada no Problema 3.4, mas nesse
caso não é muito eficaz.)

Exercício 3.8. Seja f(x) um polinômio com coeficientes inteiros, e


sejam a e b inteiros. Mostre que f(a) - f(b) só pode ser igual a 1
quando a e b forem consecutivos. (Sugestão: fatore f(a) - J(b).)
CAPÍTULO 4

Geometria euclidiana

Arquimedes há de ser lembrado quando Ésquilo estiver esquecido,


porque as línguas morrem e as ideias matemáticas não.
G. H. Hardy, A Mathematician's Apology

A geometria euclidiana foi o primeiro ramo da matemática a be-


neficiar de um tratamento a que poderíamos chamar moderno (com
postulados, definições, teoremas, e assim por diante); e mesmo hoje a
geometria é desenvolvida de um modo muito lógico e coeso. Há vários
resultados básicos que podem ser empregados para atàcar e resolver de
modo sistemático questões sobre objetos ou ideias geométricas. Esta
ideia pode ser levada ao extremo com a geometria das coordenadas,·
que transforma pontos, retas, triângulos e circunferências numa profu-
são quadrática de coordenadas e equações, convertendo grosseiramente
a geometria em álgebra. Mas a verdadeira beleza da geometria está
em podermos mostrar, pela aplicação repetida de fatos óbvios, como
68 Geometria euclidiana

um fato à primeira vista nada óbvio é indiscutivelmente verdadeiro.


Tomemos como exemplo o teorema de Tales (Euclides III, 31):

Teorema 4.1 (teorema de Tales). O ângulo subtendido por um diâ-


metro é um ângulo reto'. Em outras palavras, temos L APB= 90º .1

AI'---~--~--~____, B
o

Demonstração. Traça~d_o o segmento OP, dividimos o triângulo


em dois triângulos isósceles (porque IOPI = IOAI e IOPI = IOBj).
Usando agora o fato de os ângulos da base de um triângulo isósceles
serem iguais, e de a soma de ângulos num ~riângulo ser 180º, temos

LAPB = LAPO + LOPB = LPAO + LPBO = LPAB + LPBA


= 180º - L.APB,
e portanto L. APB tem que ser um ângulo reto. ■
A geometria está cheia de coisas como esta: resultados que podem
ser verificados medindo-se ângulos e comprimentos numa figura, mas
que não são imediatamente óbvios, como o teorema de que os pontos
médios dos lados de um quadrilátero compõem sempre um paralelo-·
gramo. Há na verdade um não sei quê de especial em fatos como
este.
do T. L APB desig~a o ângulo entre as semirretas PA e PB e sua medida
1 N.

em graus.
69

Problema 4.1 (Australian Mathematics Competition, 1987, p. 12).


O triângulo ABC está inscrito numa circunferência. As bissetrizes
dos ângulos em A, B e C inter~etam a circunferência em D, E e F,
respectivamente. Mostre que AD é perpendicular a EF.

O primeiro passo, evidentemente, é desenharmos uma figura devi-


damente legendada:

Tomei a liberdade de chamar I ao incentro (ponto onde se inter-


setam as três bissetrizes, e que é capaz de nos ser útil), e M ao ponto
de interseção de AD com EF (vértice do ângulo que queremos mostrar
que é reto). Podemos assim traduzir nosso objetivo por uma equação:
queremos mostrar que L AMF= 90º.
Este problema parece acessível: o diagrama é fácil de desenhar;
e a conclusão é confirmada pela figura. Com um tal problema, uma
abordagem direta deve funcionar bastante bem.
Precisamos determinar o ângulo em M. A primeira vista, o ponto M
pouco tem de especial. Mas, depois de reunirmos os dados disponíveis,
70 Geometria euclidiana

vemos que já conhecemos uma fartura de outros ângulos, o que se


deve principalmente a toda essa quantidade de bissetrizes, triângulos
e círculos que temos na mão. Talvez só calculando muitos ângulos
consigamos determinar L. AMF. Afinal, temos todo um arsenal de
teoremas pronto a ser usado: a soma dos ângulos de um triângulo é
180º; dois ângulos inscritos que subtendam a mesma corda são iguais;
as bissetrizes são concorrentes.
Para começar precisamos de alguns ângulos. Como o triângulo
principal é ABC, e a volta dele temos todas essas bissetrizes e circun-
ferências e tudo o mais, talvez seja melhor começarmos com os ângulos
a= L BAC, {3 = LABC e 'Y = L BCA (é tradicional usar letras gregas
para denotar ângulos). Claro que temos a+ {3 + 'Y = 180º. Podemos
determinar ainda muitos outros ângulos: por exemplo L CAD= a/2.
(Cada leitor de~e fazer sua figura e etiquetar ele próprio os ângu-
los.) Em seguida, podemos usar o fato de os_ ângulos dum triângulo
somarem 180º, e calcular alguns ângulos no interior da figura. Por
exemplo, e considerando 9 incentro I ·de ABC (ponto de interseção
de AD, BE e CF), vemos facilmente, observando o triângulo AIC, que
L AIC= 180º -a/2-"( /2. De fato, conseguimos calcular praticamente
todos os ângulos importantes - exceto aqu~les em M, que são os que
na verdade queremos. Temos portanto que exprimir de algum modo
nosso ângulo em M à custa de ângulos que não envolvam M. Ora, isto
é fácil: podemos dizer, por exemplo, que L IMF, que queremos que
seja igual a 90º, se escreve como

L IMF= 180º - L MIF-L IFM= 180º - L AIF-L CFE.

Isto é um bom progresso, pois estes ângulos L. AIF e L CFE são.


muito mais fáceis de determinar. De fato, temos

L AIF= 180º - L.-AIC= a/2 + 'Y /2,

e, dado que cordas iguàis subtendem ângulos iguais,


71

L CFE= L CBE= (3 /2.

Daqui

L IMF= 180º - a/2 - (3 /2 - '"Y /2 = 180º - 180º /2 = 90º,

como queríamos.
Este é um modo bonito de resolver problemas geométricos: sim-
plesmente calcular os ângulos. É mais fácil lidar com eles do que com
os lados (que são governados por fórmulas desagradáveis como as leis
dos senos e dos cossenos), e suas regras são mais fáceis de recordar.
São mais indicados para questões que não refiram o comprimento dos
lados, e em que disponhamos de muitos triângulos e circunferências
para manipular. Porém, para chegarmos a ângulos mais recônditos,
temos usualmente que calcular uma grande quantidade de outros ân-
gulos.

Problema 4.2 (Taylor, 1989, p.8, Ql). Num triângulo ABC, a bis-
setriz do ângulo em B interseta AC em D, a bissetriz do ângulo em C
interseta AB em E, e as duas bissetrizes intersetam-se em O. Suponha
que IODI = IOEI. Mol'ltre que L BAC= 60º ou ABC é isósceles de
base BC.

e e
72 Geometria euclidiana

Começamos por fazer uma figura. É um pouco complicado porque


convém que 0D e OE pareçam ter igual comprimento, mas fazemos
um pouco de trapaça, desenhando ABC isósceles ou L.. BAC= 60º (já
que, no fim de contas, é isso mesmo que esperamos que aconteça). Isto
nos dá duas configurações possíveis:
Nossa informação resume-se a um único dado, IODI = IOEI, e
queremos demonstrar um resultado peculiar: uma escolha entre duas
propriedades de um triângulo. Mas ambas as propriedades têm a ver
com ângulos (os triângulos isósceles têm os ângulos da base iguais, e
as bissetrizes têm obviamente a ver com ângulos), e por isso podemos
supor (pelo menos a princípio) que este é um problema de ângulos.
Visto que decidimos usar ângulos para atacar o problema, resta-nos
reformular a condição dada, IODI = IOEI, em termos de ângulos. A
saída óbvia é diz~r que, sendo ODE isósceles, temos L.. ODE= L. OED.
Pode parecer promissor, mas é algo difícil ig~alar os ângulos L.. ODE
e L.. OED a quaisquer outros ângulos. Gostaríamos também que tais
ângulos viessem expressos_ à, custa de· a = L. BAC, f3 = L. ABC e
"Y = L. BCA, pois o que queremos é provar que a = 60º ou f3 = "Y.
(Acresce que ABC é o triângulo principal: todos os outros pontos
nascem dele. É lógico usá-lo como sisterqa de referência; todas as
quantidades devem ser expressas às custas do triângulo principal.)
Observemos 0D e OE. Queremos relacionar estes comprimentos
com nossos ângulos a, f3 e "Y· Há vários modos de relacionar lados
com ângulos: trigonometria básica, triângulos semelhantes, triângulos
isósceles ou equiláteros, leis dos senos e dos cossenos, só para citar
alguns. A trigonometria básica exige ângulos retos ou circunferências,
e não dispomos de nada disso. Também dispomos de poucos triângulos
semelhantes, e já consideramos o uso de triângulos isósceles. A lei dos·
cossenos costuma complicar em vez de simplificar, e apenas introduz
novos comprimentos desconhecidos. A única alternativa viável que nos
resta é a lei dos senos -:-- que, afinal, relaciona ângulos e comprimentos
de modo bastante direto.
73

Bem, para usarmos a lei dos senos precisamos de um ou dois tri-


ângulos, de preferência que contenham OD e OE e dos quais já conhe-
çamos alguns ângulos. Observando os diagramas, e medin~o alguns
ângulos, talvez adivinhemos que os triângulos AOD, COD, AOE e
BOE podem ser úteis. Os triângulos AOE e AOD têm um lado em
comum, o que deve tornar o problema mais simples, e por isso deve-
mos tentar primeiro esses triângulos (devemos estar sempre de olho
em possíveis conexões. Saber que duas quantidades são iguais não nos
ajuda a menos que possamos de algum modo conectá-las). Visto que
estamos apenas considerando quatro dos seis pontos (A, D, O e E),
podemos desenhar uma figura simplificada para lidar com eles (afinal,
para que sermos atrapalhados por lixo inútil?).
Sabemos que L. EAO= L DAO= a/2, e do fato de a soma dos
ângulos num triângulo ser 180º podemos deduzir que L. AEO= 180º -
a-"( /2 = fJ +"i /2, e de modo semelhante também vemos que L. ADO=
180º - n - fJ /2 = "/ + fJ /2. Podemos ainda calcular mais um par de
74 Geometria euclidiana

ângulos envolvendo A, D, O e E, e finalmente nossa figura simplificada


fica com o aspecto acima (após ser rodada e ampliada para maior
legibilidade).
Podemos agora apelar à lei dos senos. Para obtermos uma expres-
são aceitável para IODI e IOEI (e é por esse motivo que usamos a lei
dos senos), podemos dizer que

1001 IOAI IDAI


sen(a/2) sen('Y + ,B/2) sen( a/2 + ,B /2)
e que
IOEI IOAI IEAI
sen( a/2) sen(,B + 'Y /2) sen(a/2 + 'Y /2)"
Agora que dispomos de uma equação, talvez a condição dada
(IODI = IOEI) ~e converta em alguma coisa útil. O comprimento
IOAI aparece em ambas as sequências de eq~ações, e por isso talvez
devamos exprimir IODI e IOEI à custa de IOAI. Obtemos

1ooi" =IOAI sen(a/2)


sen('Y + ,B /2.)

IOEI =IOAI sen(a/2)


sen(,B + 'Y /2)'

e portanto IODI = IOEI se e só se sen('Y + ,B/2) =sen(,B + 'Y/2). (Mas


poderia também acontecer algo de disparatado como sen(a/2) = O.
Depressa concluímos, porém, que tal situação só pode ocorrer em ca-
sos extremamente degenerados, e é melhor tratar à parte tais anorma-
lidades. Mas é bom estarmos sempre atentos a essas coisas.)
Transformamos ass~m uma igualdade de comprimentos numa igual-·
dade de ângulos. E o mais assinalável é que tais ângulos são relevantes
para nossq objetivo (que envolve os ângulos a, ,B, "f). Parece portanto
que vamos no caminqo certo. A questão agora é quase puramente
algébrica.
75

Em suma, dois certos ângulos têm senos iguais. Isto significa uma
de duas coisas:

ou

'Y + /3 /2 = 180º - (/3 + 'Y /2) .

Estamos cada vez mais perto de nosso objetivo: os senos sumi-


ram-se, e pela primeira vez temos uma proposição que envolve a par-
tícula ou. Agora não é difícil vermos que o primeiro caso conduz a
/3 = 'Y, enquanto que o segundo dá /3 + 'Y = 120º e portanto a = 60º.
E ainda conseguimos, estranhamente, dar de cara com nosso objetivo.
E aqui temos o que queríamos. Por vezes, podemos simplesmente
pegar nos dados e martelá-los até termos uma equação que se asseme-
lhe ao objetivo (neste caso, algo que envolvesse os ângulo~ a, /3, 1), e
em seguida aplicar alguma álgebra simples para lhe dar a forma que
buscamos. É a isto que se chama uma abordagem direta, e funciona
bem quando o objetivo _é uma relação simples envolvendo ·partes do
problema que sejam fáceis de calcular, porque então podemos ter uma
ideia de como abordar o problema por um processo de simplificação
gradual dos dados,. transformando-os em coisas que se pareçam mais
e mais com o objetivo. Quando o objetivo é obscuro, poderemos ter
de o trabalhar antes de podermos adivinhar qual a direção a tomar.
É isso que nosso próximo problema ilustra.

Problema 4.3 (Australian Mathematics Competition, 1987, p. 13).


(*) Sejam ABFE um retângulo e D a interseção de suas diagonais AF
e BE. Uma linha reta por E interseta a reta AB em G e a reta BF
cm C, sendo IDCI = IDGI. Mostre que IABI/IFCI = IFCI/IGAI .
IGAI/IAEI,
76 Geometria euclidiana

Com problemas geométricos, tanto podemos usar a abordagem di-


reta, avançando a partir dos dados (medindo ângulos e lados de modo
sistemático), como a inver~a, tentando refazer o caminho a partir
do objetivo (reformuland~ o resultado pretendido em qualquer coisa
equivalente mas mais manejável). Às vezes· o simples ato de dese-
nhar uma figura e tentar adivinhar as conclusões ajuda, mas neste
caso a tal figura é bem difícil de desenhar. Como conseguir que seja
IDCI = IDGI? Umas quantas tentativas, mais uma olhadela à con-
clusão IABI/IFCI = IFCI/IGAI = IGAI/IAEI, permitem finalmente
desenhar uma figura aceitável (ver acima).
Vamos tentar a abordagem direta. Usar a força bruta da geome-
tria das coordenadas nos leva por caminhos longos e aborrecidos, e
sujeita-nos a complicações e erros abissais; fica como último recurso
(embora o ângulo reto em A sugira que essa é uma localização tenta-
dora para um sistema de eixos). E, com equações como IDCI = IDGI,
a geomet~ia vetorial também não é muito apropriada (mas as ver-
sões vetoriais são em geral mais limpas do que as correspondentes
equações com coorde~adas). Como fazer para medir ângulos e com-
77

primentos? Sabemos só que temos quatro ângulos retos no retângulo,


e que IDCI = IDGI. Sim, é verdade que DCG é isósceles, mas isso não
acrescenta muito. Baixar a perp~ndicular a CG por D ou executar ou-
tras construções semelhantes também não é de grande ajuda. (Como
veremos adiante, há uma certa construção que de fato ajuda, mas não
é intuitivamente óbvia na abordagem direta.)
Tentemos então o caminho inverso. Queremos provar que três cer-
tos quocientes são iguais. Isto faz lembrar triângulos semelhantes.
Será possível formar um triângulo com, digamos, AB e FC? Não pre-
cisamente; mas podemos formar um triângulo com FE e FC, e FE
tem comprimento igual ao de AB. Uma vez que localizamos um dos
triângulos, os outros dois não devem ser muito difíceis. Basta obser-
var o triângulo FCE na figura para reconhecermos (e demonstrarmos
facilmente) que ele é semelhante a BCG e a AEG; e que portanto

IEFI/IFCI = IGBI/IBCI = IGAI/IAEI.

Tendo em mente nosso objetivo, isto pode ser convertido em

IABI/IFCI = IGBI/IBCI = IGAI/IAEI. (15)

Verificamos assim que dois dos três quocientes que nos foram dados,
IABI/IFCI e IGAI/IAEI, são iguais. O t~rceiro quociente, IFCI/IGAI,
não se materializa facilmente num triângulo. Mas observemos o termo
intermediário de (15): esses dois segmentos parecerri vagamente rela-
cionados com FC e GA. De fato, FC é uma porção de·BC e GA é uma
porção de BG. Palpita-nos que poderá ser mais fácil provar que

IFCI/IGAI = IGBI/IBCI
do que

IABI/IFCI = IFCI/IGAI ou IFCI/IGAI = IGAI/IAEI.


78 Geometria euclidiana

Além do mais, esta formulação é mais simétrica e envolve uma


única equação.
Mas, mesmo com nossa formulação possivelmente mais simples,
ainda não parece haver triângulos semelhantes que possamos usar.
Precisamos ainda manipular um pouco mais nosso problema. Uma
manobra óbvia a tentar é rearranjarmos os quocientes, talvez nos li-
vrando dos denominadores:
IFCI X IBCI = IGAI X IGBI

ou trocando numeradores por denominadores:

IFCI/IBGI = IGAI/IBCI.

Esta última equação não parece ser uma grande melhoria, mas
na anterior os produtos IFCI x IBCI e IGAI x IGBI talvez nos sejam
familiares. De fato, pode vir a mente o seguinte resultado (que aparece
geralmente em livros de texto para o ensino médio mas é raramente
usado):

Teorema 4.2. Se P estiver no exterior de uma circunferência de cen-


tro O e raio r, e uma semirreta de origem P intersetar a circ'l),_nferência
em dois pontos Q e R, então
IPQI X IPRI = IPTl 2 = IPOl 2 -r2,
onde T é o ponto onde uma das tangentes por P toca à circunferência.

Demonst_ração. Notamos que .PQT é semelhante a PTR, e por-


tanto IPQI/IPTI = IP'TI/IPRI. Além disso, o teorema de Pitáµ;oras dá
IPOl 2 = IPTl 2 +r2 , e d.aí vem a conclm,ão. ■
79

Para usarmos o Teorema 4.2, precisamos primeiro criar uma circun-


ferência. Como nos interessam os produtos IFCI x IBCI e IGAI x IGBI,
a circunferência deverá incluir os pontos F, B e A. Acontece que a
circunferência que passa por F, B e A tem como centro o ponto D
(Teorema 4.1!). Logo, pelo Teorema 4.2, temos

onde ré o raio da circunferência. E como nos disseram, muito conve-


nientemente, que IDCI = IDGI, o resultado fica demonstrado.
É deste modo que as questões puramente geométricas funcionam:
com dados aparentemente insuficientes para avançar,· e qualquer coisa
de recôndito a demonstrar, há que proceder de maneir·a especial. Uma
construção ou qualquer outra coisa podem clarificar a questão. Esteja-
mos de olhos em coisas que tragam alguma lembrança. Por exemplo, se
nalgum problema geométrico temos que provar que L ABC= L ADC,
talvez possamos em vez disso mostrar que ABDC é cíclico (o que é
equivalente se B e D estiverem do mesmo lado da reta AB). Ou, se
tivermos que provar que jABI > IACI, podemos, equivalentemente,
80 Geometria euclidiana

provar que L ACB > L ABC (desde que A, B e C não sejam coli-
neares). Ou, se nos surgir uma questão sobre áreas de triângulos,
apelamos a fatos como o de triângulos com a mesma base e mesma
altura terem a mesma área, ou de que, se reduzirmos a base de um
triângulo à metade, a área também é dividida por dois. Isto não sig-
nifica que devamos construir todas as possíveis extensões da figura
que nos ocorram, ou anotar uma grande profusão de fatos (a menos
que estejamos mesmo emperrados), mas que um palpite informado e
uns tantos diagramas podem funcionar. Por vezes, podemos recor-
rer a casos especiais ou extremos para tentar encontrar o caminho
(no problema atrás, por exemplo, poderíamos ter considerado os ca-
sos em que ABEF é um quadrado, ou em que ABEF degenerasse, ou
em que fosse IDCI = IDGI = O). Mas sempre mantendo em vista os
dados (IDCI = IDGI, ABEF é um retângulo) e o objetivo a alcançar
(IFCI x IBCI = IGAI x IGBI, ou outra qualque_r formulação). E tente-
mos conduzir nossos passos na direção dos dados ou objetivos menos
usuais (no nosso caso, a estranha iguáldade IDCI = IDGI). Afinal,
é de esperar que precisemos de todos os dados para atingir todos os
objetivos, e portanto cada pedaço de informação deverá ser de algum
modo invocado.
O segredo aqui foi recordar um certo resultado em geometria eu-
clidiana, neste caso o Teorema 4.2. Com experiência suficiente em
geometria estas coisas podem nos ocorrer depois de termos observado
cada parte do problema e assimilado a natureza da questão (estas
coisas em geral também só ocorrem depois de todos os outros meios
terem falhado). Sem uma tal inspiração, devemos nos restringir à ge-
ometria das coordenadas ou mesmo à das pseudo-coordenadas (de D,
baixamos perpendiculares a AB e AC, digamos, e usamos o teorema·
de Pitágoras para representar IDCI e IDGI - o que é, no fundo, fazer
geometria- das coordenadas sem fixar um sistema de eixos).

Problema 4.4. Dad~ três retas paralelas, construa (mlll n-.g11a n


81

compasso) um triângulo equilátero com um vértice em cada uma das


retas.

li
?
l2 B
-[>

l3

A primeira vista, este problema parece simples e direto (os bons


problemas, em regra, são assim). Mas logo que tentamos desenhar uma
figura (começando por traçar as retas paralelas), vemos como é na ver-
dade complicado ajustar nas retas um triângulo com tantas restrições
como o equilátero. Depois de experimentarmos com circunferências,
ângulos de 60º e coisas do gênero, devemos concluir quê précisamos de
algo especial. Seja como for, tentemos desenhar a melhor figura que
pudermos (talvez desenhando primeiro o triângulo equiláter~, e depois
apagando-o) e em seguida etiquetar tudo.
Uma tentativa óbvia é usarmos a geometria das coordenadas. Bom,
isso é possível, mas confuso. Acabaríamos por recorrer à fórmula re-
solvente para calcular a posição dos pontos, e essa maneira de fazer
as coisas não é a melhor (nem a mais geométrica). Como de costume,
guardamos as coordenadas como último recurso.
O procedimento usual em problemas de construções é tomar um
dos objetos que queremos encontrar (um ponto, uma reta, um triân-
gulo ou talvez alguma outra coisa) e determinar um lugar geométrico
ou alguma outra propriedade fácil de caracterizar.
Mas, antes de fazermos isso, fixemo-nos no diagrama e vejamos
o que conseguimos fazer com ele. Uma coisa que se nota é que um
t.riàngulo equilátero, caso haja algum, pode deslizar ao longo das retas
82 Geometria euclidiana

paralelas sem perder suas propriedades. Assim, se o triângulo for


ABC, a posição de A é na verdade arbitrária, desde que seja na reta
li. Claro que B e C vão ter de depender da posição de A. Em suma,
podemos pôr A onde quisermos sem nos preocuparmos se com isso
perdemos alguma coisa, e só depois tratarmos de B e C. Um pouco
de reflexão mostra-nos então que a reta li já não tem importância: é
necessária para restringir a posição de A; mas, uma vez usada para
nela escolhermos um ponto arbitrário, já não precisamos mais de li.

Ancorado como está agora o triângulo no ponto A, sua posição está


mais restrita. Talvez isso deixe só um número limitado de posições
para B e C. Ainda não sabemos.

O triângulo equilátero tem agora apenas dois graus de liberdade:


sua orientação e seu tamanho. Mas obedece a duas restrições: um dos
pontos, B, está em h, e o outro, C, em h. Em teoria, isto deveria
ser suficiente para fixar o triângulo, mas com um objeto tão complexo
como o triângulo é difícil ver· como devemos prosseguir. O que pode-
mos fazer é converter o que é pedido em alguma outra coisa mais fácil
de calcular. Pedem-nos um triângulo equilátero. E que tal algo mais
simples? O objeto geométrico mais simples é um ponto. Assim, pode-
ríamos tentar localizar B em vez do triângulo completo. B só dispõe
de um grau de liberdade, uma vez que tem que estar em 12 • Qual é a
âncora que prende B? O fato de o triângulo equilátero de base AB ser
obrigado a ter seu terceiro vértice (ou seja, C) em h. Esta ·restrição
é complicada e ainda envolve o triângulo equilátero. Há algum modo
mais fácil de representar C à custa de A e de B? Sim: C é a imagem
de B por uma rotação de 60º com centro em A (no sentido horário ou
anti-horário). O problema fica então reduzido ao seguinte:

Dados .A e duas retas paralelas 12 e h que não passam por A,


determinar um ponto B em h que, ao ser rodado 60º em torno de A,
vá parar a I::i.
83

A
?
_ _ _ _ _ _ l2 --t>
______ ___ _.__ [3
e
Temos agora um só objeto por determinar: o ponto B. Há portanto
menos graus de liberdade, e o problema deve agora ser mais simples.
Queremos que B obedeça a dois requisitos:

(a) B está em 12 ;
(b) a imagem de B pela rotação de 60º em torno de A está cm 13 .

Tal como está, a condição (b) não é usável; precisamos de inver-


tê-la:

(b') B está na imagem de h pela rotação inversa de 60º em torno de ,

Ou seja, B está em 1;, onde 1; é a reta que resulta de 13 rodando-a


60º no sentido oposto (seja ele horário ou anti-horário) em torno de
A. Nossas propriedades ficam então:

(a) B está em 12 ;
(b') B está em 1;.

Em outras palavras, B é o ponto de interseção de 12 e 1;. E é tudo!


Construímos B explicitamente, e o resto do triângulo é fácil.
Por uma questão de completude, eis aqui o resumo da construção:
Tomar um qualquer ponto A em 11 . Rodar 60º a reta 13 em torno de
A (no sentido horário e no sentido anti-horário: há duas soluções B
para cada ponto A) e chamar B a interseção da reta rodada com h-
Rodar 60º o ponto B no sentido contrário para encontrar C.
84 Geometria euclidiana

Podemos notar que esta construção também funciona se as retas


não forem paralelas, desde que não façam um ângulo de 60º entre si.
O paralelismo era efetivamente uma pista falsa!
Com problemas de construções, a ideia é, tal como em álgebra,
resolver em relação às incógnitas (só que as incógnitas são nesse caso
objetos geométricos, como pontos ou retas). Aqui nossa incógnita era
B, e reformulamos progressivamente os dados até eles tomarem a forma
B é.... Para dar uma analogia algébrica, suponhamos que queríamos
resolver os seguintes dados em relação a b e a e:

• b + 1 é par;

• bc = 48;

• e é uma po.tência de 2.

Se resolvermos em relação a b estas três equações, e eliminarmos


e, obtemos:

• b é igual a um número par menos 1 (ou ·seja, b é ímpar);

• b é igual a 48 dividido por uma potência de 2.

Comparando o conjunto dos números ímpares com o conjunto dos


números obtidos dividindo 48 por potências de 2, concluímos que b é
3. Muitas vezes, o modo mais fácil de resolver problemas com várias
variáveis é eliminá-las uma a uma, e o mesmo acontece nas construções
geométricas.

Exercício 4.1. Sejam_k e 1 duas circunferências que se intersetam em


P e em Q. Construa uma reta m por P que não passe por Q e tenha
a seguinte propriedade: se m interseta k em B e P, e interseta 1 em
C e P, então !PB! = jPCj; figura em baixo. (Sugestão: resolva em
relação a B.) ·
85

. ?

---{>

k k

Exercício 4.2. É dada uma circunferência com dois pontos A e B


em seu interior. Inscreva, se possível, um triângulo retângulo na cir-
cunferência de modo a que um de seus catetos contenha A e o outro
contenha B; veja a figura abaixo. (Sugestão: resolva em relação ao
vértice do ângulo reto.)

X
A

?

---t>

Exercício 4.3. (*) São dados quatro pontos A, B, C e D. Construa,


se possível, um quadrado em que cada lado contenha um dos quatro
pontos; ver figura abaixo. (Sugestões: infelizmente, é muito difícil
resolver em relação ao quadrado, e resolver em relação a apenas uni
dos vértices - em analogia com o que o fizemos anteriormente - é só
ligeiramente melhor: o vértice pode ser confinado a uma circunferên-
cia fixa, mas é só. Uma abordagem que de fato produz resultados
(\ a de resolver. em relação a uma das diagonais do quadrado. Uma
86 Geometria euclidiana

diagonal é determinada por diversos parâmetros: a orientação, a po-


sição, as extremidades. Mas uma diagonal determina completamente
o quadrado, coisa que um vértice sozinho não faz. Se estiver mesmo
emperrado, tente fazer um diagrama bem grande em que desenhe pri-
meiro o quadrado e só depois os pontos dados, e onde, em seguida,
marque as circunferências de diâmetros AB, BC, CD e DA e também
as diagonais do quadrado. Tire o máximo proveito possível das cir-
cunferências: calcule ângulos, triângulos semelhantes, e por aí fora.
Uma sugestão de grande ajuda é observar as interseções das diagonais
com as circunferências. Há uma solução alternativa em que se resolve
em relação a um lado, usando rotações, reflexões e translações para
deslocar um certo lado até que ele quase coincida com outro; em suma,
uma solução ao estilo da do problema anterior.)

X
A A
DX D
?

xª --t> B

e e
X

Problema 4.5 (Taylor, 1989, p. 10, Q4). Um quadrado é dividido


em cinco retângulos tal como mostra a figura em baixo. Os quatro
retângulos exteriores R 1 , R 2 R 3 e R 4 têm todos a mesma área. Mostre
J

que o retângulo int?rior, R{i, é um q~rndra<lo.


87

R4
R1 .
Ro

R3
R2

Esta é mais uma daquelas questões com um objetivo pouco usual.


O fato de as áreas dos quatro retângulos exteriores serem iguais não
parece obrigar, a primeira vista, a que os lados do retângulo interior
sejam todos iguais. É que parece haver liberdade a mais nos dados:
afinal, um retângulo com uma área fixada pode ser comprido e magro,
ou curto e gordo. Por que não poderíamos modificar a forma de um
dos retângulos, distorcendo assim o retângulo interior? Uma pequena
tentativa mostra a razão de isto não funcionar: cada um dos retân-
gulos está constrangido pelos retângulos vizinhos. O retângulo R 1 na
figura, por exemplo, está preso ao seu lugar pelos retângulos R 2 e R 4 .
Qualquer mudança em R 1 obrigaria R 2 e R 4 a mudarem também, e
cada um deles daria ordens a R 3 para se mexer.·

R4
R1

Ro
R3

R2
88 Geometria euclidiana

Mas R 3 não pode obedecer a essas duas ordens a menos que elas
sejam no mesmo sentido. Na figura acima, o retângulo R 3 pode ajus-
tar-se ao retângulo R 2 ou ao retângulo R 4 , mas não a ambos (recorde-
mos que R3 também tem que ter a mesma área que R2 e R4 ). Começa
a fazer-se luz sobre o modo como a pergunta funciona: a exigência de
áreas iguais, mais a necessidade de os retângulos se ajustarem sem so-
breposições, deve deixar o quadrado interior como única possibilidade.
Deve ser impossível deformar esse arranjo simétrico em cruz suástica
sem que alguma coisa corra mal, como exemplifica a figura atrás.

Antes de avançarmos precisamos fixar notação: mais precisamente,


precisamos exprimir os lados e as áreas dos retângulos em causa em
função de umas poucas variáveis. De nossa discussão sobre possíveis
deformações deste arranjo concluímos que cada retângulo, por exemplo
Ri, vai determinàr a posição de todos os restantes: Ri vai forçar R 2 e
R 4 a ocupar certas posições fixas, e estes por sua vez vão fixar R 3 (se tal
for possível). Surge aqui portanto uma abordagem algébrica: supomos
que o retângulo Ri tem, digamos, lados a e b, e que o quadrado maior
tem lado 1; e depois determinamos as dimensões de todos os outros
retângulos, em particular as de Ro. Este é o método da força bruta:
o retângulo R 3 (ou talvez Ri, R 2 ou R4 , · se dermos interpretações
diferentes às equações) será governado por duas expressões diferentes,
que, depois de igualadas, devem implicar uma certa relação entre a e b
(o retângulo Ri não pode ter uma forma arbitrária; de fato, temos que
provar que as únicas formas que Ri pode tomar são as que conduzem
a um quadrado no centro). O próximo diagrama resume a situação:

Para que R 3 tenha a área correta, terá que ser (1 ·- ab/(l - a)) x
( 1 - ab / ( 1 - b)) = ab,. e podemos resolver isto em relação a a, b e·
daí concluir que R 0 é um quadrado. Este método funciona, mas é
um pouco .trabalhoso, e por isso vamos tentar algo mais simples, mais
intuitivo, e menos baseado no uso de coordenadas (como seria na re-
alidade a abordagem e~ cima).
89

a 1-a

ab/(1-a)
. R4
b R1

Ro
1-ab/(1-a)
R3
1-b R2

ab/(1-b) 1-ab/ (1 -b)

Queremos mostrar que a única forma de todas as condições serem


satisfeitas é R 0 ser um quadrado. Mas isso é complicado de provar.
Já mostramos que conseguimos exprimir tudo à custa, digamos, do
retângulo R 1 . Nesse sentido, podemos chamar R 1 de figura principal:
aquela de que todas as outras dependem. Uma vez que dispomos desse
ponto de referência, podemos nos concentrar em ui:n só retângulo.
Assim, em vez de tentarmos demonstrar algo sobre o retângulo R 0
que, ao contrário dos outros, não é fácil de promover a figur.a principal
- podemos tentar fazê-lo sobre R 1 , o que deve ser mais fácil.
A figura atrás parece sugerir que a+ b é igual a 1. De fato, se
a+ b for igual a 1, então R 2 tem o lado vertical igual a 1 - b = a, e
por ter área igual à de R 1 seu lado horizontal é b, logo o retângulo R:1
tem o lado horizontal igual a 1 - b = a, e assim por diante. Deduz-se
assim que o arranjo dos retângulos é simétrico, formando uma espécie
de suástica onde facilmente se vê que R 0 é um quadrado de lado b - a.
Conseguimos pois identificar uma meta intermédia, a de mostrar que
a+ b = l. Heuristicamente, esperamos que esta meta seja mais fácrl
de atingir, pois podemos exprimir tudo à custa de a e de b, mas não
seria fácil fazê-lo à custa do retângulo R 0 •
Em resumo, já demonstramos a segunda implicação· na seguinte
cadeia:
90 Geometria euclidiana

Ri, ... , R4 têm áreas iguais ⇒ a+ b = 1 ⇒ Ro é um quadrado


e só nos falta agora demonstrar a primeira implicação.
Como pudemos ver pela abordagem com coordenadas, os dados
podem facilmente ser traduzidos em fórmulas, mas das fórmulas não
chegamos facilmente a nosso objetivo. Apesar de a condição das áreas
iguais parecer ser simpática e fácil de manejar, ela comporta-se na
verdade como um obstáculo nesta questão, pois temos um punhado de
produtos iguais cujos fatores estão relacionados por equações aditivas.
Mas podemos trabalhar da frente para trás, quer seja tentando mostrar
que
a+ b #- 1 ⇒ Ri, ... , R4 não têm áreas iguais
quer seja tentando uma demonstração por redução ao absurdo:

a+ b #- 1 ~ Ri, ... , R4 têm áreas iguais ⇒ Contradição

Notemos que, numa demonstração por absurdo, dispomos de mais


dados para começar, mas o resultado final não tem limites definidos.
É uma estratégia que combínà bem com nossa abordagem qualitativa
anterior: a de ser impossível deformar o arranjo inicial sem que os
retângulos deixem de encaixar uns nos outros. Vamos pois concen-
trar-nos na demonstração por absurdo.
Suponhamos então que a+b é grande demais (ou seja, maior do que
1), mas que ainda assim os retângulos conseguem ter a mesma área.
Queremos obter uma contradição daqui. O que na verdade temos é
um retângulo sobre o grande, Ri. Que consequências isto tem? Força
o retângulo R 2 , digamos, a ser comprido. De fato, seu lado vertical
é 1 - b, menor do que a. Logo R2 é mais estreito do. que Ri. Mas,
como os dois têm área igual, R 2 tem que ser mais comprido do que Ri.
Assim, R2 é mais esguiõ do que Ri. Mas notemos agora o retângulo
R 3 : um argumento análogo o obriga a ser mais esguio do que R 2 2 •
Pelas mesrrias razões, R4 é mais esguio do que R3 . E, por último, Ri
20 argumento aplica-se à R2 · porque a soma de seu lado horizo11t.al com o
91

é mais esguio do que R 4 . Mas isto significa que R 1 é mais estreito e


mais comprido do que ele próprio, o que é absurdo. Já temos nossa
contradição. Um fenómeno semelhante acontece quando a+ b é menor
do que 1: os triângulos ficam mais largos e mais curtos (ou seja, mais
atarracados), até que finalmente concluímos que R 1 tem que ser mais
atarracado do que ele próprio, o que nos dá a desejada contradição. 3
Este problema ilustra bem como uma figura pode valer por mil
equações. E é bom termos em mente que por vezes as desigualdades
são mais fáceis e mais eficientes de usar do que as igualdades.

Exercício 4.4. Determine todos os números reais positivos x, y, z e


todos os inteiros positivos p, q, r tais que
xP + yq = yr + zP = Zq + Xr = 2.
(Sugestão: esta questão não tem nada de geométrico, mas ainda
assim é semelhante ao Problema 4.5.)

Problema 4.6 (AMOC, Problemas por correspondê_nciaJ 1986-1987,


fascículo 1, pergunta 1). Sejam ABCD um quadrado, k a circunfe-
rência com centro B e passando por A, e 1 a semicircunferência de
vertical é também maior do·que 1, já que a desigualdade a+b > 1 implica (1 - b) +
ab/ (1 - b) > l. De modo semelhante, no caso a+ b < 1 temos também (1 - b) +
ab/ (1 - b) < l.
3 N. do T. Há uma resolução alternativa em que os dois casos a+b > 1 e a+b < 1

são reunidos em um só. De fato, se for a+b =/- 1, então um dos vizinhos de R 1 (i.e.,
R 2 ou R 4) é mais esguio do que R1: ou seja, admitindo que a :S b, esse tal vizinho
tem dimensões a' x b', onde a' < a (mais estreito) e b' > b.(mais comprido). Se
for a+ b < 1, então aquele dos vizinhos com um lado igual ~ 1 - a (> b) é mais
comprido do que R 1 (logo, tendo a mesma área, é também mais estreito); e, se for
a+ b ·> 1, então aquele dos vizinhos com um lado igual a 1 - b ( < a) é mais estreito
do que R 1 (logo, tendo a mesma área, é também mais comprido). Dado que em
nenhum dos retângulos R 1, R2, R3, R4 a soma de lados é 1 (pois, como já vimos, se
fosse 1 em algum deles seria 1 em todos), o mesmo argumento mostra que cada um
deles tem um vizinho mais esguio. Mas isto significa que, começando por exemplo
com R. 1 , podemos construir uma sucessão infinita de retângulos Ri. cada vez mais
Psµ;nios, o que é absurdo pois dispomos somente de quatro retângulos.
92 Geometria euclidiana

diâmetro AB contida no quadrado. Sejam E um ponto de 1 e F o


ponto onde a semirreta BE interseta k. Mostre que L DAF= L EAF.

Começamos, como sempre, por fazer uma figura:

D e

Temos que relacionar dois ângulos. Como no enunciado não se


fala de comprimentos, dá impressão que podemos resolver a ques-
tão usando apenas ângulos. Afinal, as circunférências tratam sempre
muito bem os ângulos. Mas estes ângulos- em particular, L. DAF e
L EAF, parecem mal posicionados. Para que possamos relacioná-los
um com o outro, será necessário exprimir estes ângulos recônditos em
função de outros mais amigáveis.
Consideremos inicialmente·o ângulo L DAF. Ele não parece estar
ligado a nenhum triângulo interessante, mas tem alguma relação com
a circunferência k. Vem a propósito um pequeno teorema de Euclides
(Euclides III, 32) que diz que o ângulo que uma corda ·subtende numa
circunferência é igual ao ângulo que a corda faz com a tangente: po-
demos então dizer que L DAF= L APF, onde Pé um ponto qualquer
da circunferência k que não esteja·no arco AF da figura; temos por
exemplo L DAF= L A~f. Mas L. ACF é quase tão chato como L DAF
- só que está inscrito mmia: circunferência, e portanto é metade do
93

ângulo ao centro correspondente; ou seja, L ACF= ½LABF. O ângulo


L ABF parece ser mais corriqueiro, e está relacionado com diversos
círculos e triângulos; por isso, o resultado L DAF= ½L ABF parece
ser bastante satisfatório.
Tratamos agora de L EAF. Este ângulo ainda é mais desajeitado
do que L DAF : não está diretamente ligado a coisa nenhuma. Mas
tem o vértice em comum com outros ângulos mais simpáticos, como
L DAB e LEAB e outros, de modo que podemos representá-lo usando
esses ângulos mais amigáveis, pondo por exemplo

LEAF= LBAF-LBAE
ou talvez

LEAF= LDAB-LDAF-LBAE.

A primeira igualdade usa um ângulo simpático (~ B~E) e outro


um pouco pior ( L BAF). A segunda formulação, porém, tem várias
vantagens: L DAB é 90º, e já calculamos L DAF, de modo que obte-
mos

LEAF= 90º - ½LABF-LBAE.

Mas L BAE e L ABF são ângulos do· mesmo triângulo ABE. Visto
que já escrevemos tanto L DAF como L EAF em função dos ângulos
de ABE, esta é claramente uma boa altura para nos focarmos neste
triângulo.
Bom, ABE está inscrito numa semicircunferência. Isso deve recor-
dar o teorema de Tales (Teorema 4.1), que nos diz que L BEA= 90º.'
Portanto, os dois ângulos L ABF e L BAE ficam ligados um ao outro,
já que a soma dos ângulos de um triângulo é 180º. Mais precisa-
mente, temos L ABF + L BAE+ L BEA= 180º, e portanto L BAE__:_
90º - L ABF. Podemos inserir isto em nossa expressão para L EAF,
94 Geometria euclidiana

obtendo

LEAF =90º - ~LABF- LBAE = 90º - ~LABF

- (90º - L.ABF) = ~LABF.

Mas esta é precisamente a expressão que obtivemos para L. DAF.


Provamos então que L DAF= L EAF. Claro que convém ter as coisas
arrumadinhas quando apresentarmos a demonstração; provavelmente
gostaríamos de dispôr tudo como uma longa sucessão de igualdades:
LDAF =:

= L~AF.
Mas não temos que ser tão formais assim. quando buscamos uma
solução. Manipular os ângulos L DAF e L EAF na esperança de que
alguma vez eles se encontn::m não é assim tão tolo, desde que saibamos
o que estamos fazendo. Se tentarmos a cada passo simplificar os dados
e relacioná-los uns com os outros, temos boas· chances de tropeçar na
solução. (Supondo que ela existe, claro, e. a maioria dos problemas
não está brincando conosco.)
CAPÍTULO 5

Geometria analítica

O raciocínio geométrico não está tão estreitamente ligado à


geometria que não possg, ser desviado e transferido para ou'tras áreas
do conhecimento. Tratados de moral, política ou crítica, ou talvez
mesmo simples passagens de retórica, serão mais elegantes, ainda
que não se distingam no resto, se.forem moldados pela mão da
Geometria.
Bernard le Bovier de Fontenelle (1729), Préface sur l'Utilité
des Mathématiques e~ de la Physique

Este capítulo trata de problemas que, embora envolvam conceitos e


objetos geométricos, têm soluções que apelam a ideias de outros ramos·
da matemática: álgebra, desigualdades, indução e por aí fora. Um
truque que por vezes funciona muito bem é reescrever os problemas
geométricos em termos de vetores e depois usar as regras dà aritmética
vetorial. Eis um exemplo desses.
96 Geometria analítica

Problema 5.1 (Australian Mathematics Competition, 1987, p. 14).


Um polígono regular com n vértices está inscrito numa circunferência
de raio 1. Seja L o conjunto de todos os possíveis comprimentos de
todos os segmentos ligando dois vértices do polígono. Qual é a soma
dos quadrados dos elementos de L?

Em primeiro lugar, tratamos de dar um nome mais curto à soma


dos quadrados dos elementos de L: chamamos-lhe X. Nossa tarefa é
portanto a de calcular X. Este é o que pode ser chamado um problema
praticável. Não é um problema de mostrar que ou verificar se, mas de
calcular um número que deve poder ser encontrado por uma aplicação
direta da trigonometria e talvez do teorema de Pitágoras. Quando
n = 4, por exemplo, temos um quadrado inscrito numa circunferência
unitária. Os comprimentos possíveis são o do lado ( -/2) e o da diago-
nal (2), e portanto X= (-/2) 2 + 22 = 6. Analogamente, quando n = 3,
o único comprimento que temos é o do lado, igual a y3. O caso n = 5
não é simples a menos que conheçamos uma mão cheia de valores do
seno e do cosseno, e por isso saltamos para n = 6. O comprimento do
lado é 1, o da diagonal menor é v3, e o da maior (que é um diâmetro)
é 2. Portanto neste caso X= 12 + (v3) 2 + 22 = 8. Finalmente, temos
n = 2, caso um tanto degenerado em que o polígono é simplesmente
um diâmetro e X= 22 = 4. Temos assim já calculados alguns casos:

n X
2? 4?
3 3
4 6
6 8

Marcamos o caso n = 2 com um ponto de interrogação por ser um


pouco duvidoso falarm?s de um polígono com dois lados.
Esta curta tabela não rios dá muitas pistas sobre corno sC'rá. a rcH-
97

posta no caso geral. A primeira coisa é desenharmos uma figura.


Talvez seja aconselhável dar nomes aos vértices. Para um valor fixo
de n (5 ou 6, digamos), poderíamos chamar-lhes A, B, C, etc., mas no
caso geral é talvez mais conveniente chamar-lhes A 1 , A2 , A3 , ... , An
como vemos na figura:

Começamos por enumerar algumas observações e palpites:


(a) Talvez faça difere.nça o fato de n ser par ou ímpar. Se n é
par, temos que levar uma diagonal longa (igual ao diâmetro) em
consideração. Assim, quando n é par há n/2 diagonais diferentes,
e quando é ímpar há (n - 1) /2; ·

(b) Talvez o resultado seja sempre um inteiro. Esta conjectura não


tem ainda uma base forte, pois só lidamos com casos muito es-
. peciais - o quadrado, o hexágono regular, o triângulo equilátero
- em que as medidas são dadas por raízes quadradas. Contudo,.
fica a esperança de que a solução geral não seja muito horrorosa;

(c) Estamos somando os quadrados dos comprimentos, e não os com-


primentos em si. Isto coloca imediatamente o problema no âm-
bito da geometria analítica e fora da geometria pura: sugere o
98 Geometria analítica

uso de vetores, ou da geometria das coordenadas, ou ainda dos


números complexos (embora estas três abordagens sejam basica-
mente as mesmas). A geometria das coordenadas acabará forne-
cendo, devagar mas seguramente, uma solução envolvendo somas
trigonométricas, mas tanto o uso da geometria vetorial como o
dos números complexos são promissores (com vetores podemos
usar o produto escalar, e com números complexos podemos usar
a exponencial complexa);

(d) O problema é quase impossível de tratar numa abordagem di-


reta, porque não estamos somando os quadrados dos comprimen-
tos de todas as diagonais, mas só daquelas que têm comprimen-
tos distintos. Mas podemos rapidamente reformular a questão
de um moçlo que seja mais fácil de traduzir por uma equação.
(As equações são matemática palpável. ~ão são tão inspiradoras
como as figuras ou as ideias, mas são mais fáceis de manipular.
Em geral, devemos ~empre exprímir nosso objetivo por algum
tipo de equação. Mas há possíveis exceções em combinatória e
teoria dos grafos.) Se nos restringirmos às diagonais saindo de
um único vértice do polígono, elas ir~o cobrir todos os compri-
mentos de que precisamos.

Nesta figura, por exemplo, em que n é par, estão marcadas diago-


nais com os quatro possíveis comprimentos. Considerando só a semi-
circunferência superior, cada comprimento de diagonal aparece exata-
mente uma vez: os valores IA1A2I, IA1A3I, IA1A4I e IA 1A 5 I abrangem
todos os comprimentos possíveis. Em outras palavtas, o valor X é
dado pela seguinte expressão: IA1A2l 2 + IA1Aal 2 + jA 1 A 4l2 + IA 1A 5 l2_-
De forma mais geral, o que queremos é calcular a soma IA1A 2l2 +
· · · + IA 1 Aml 2 , onde m = (n + 2)/2 se n for par, em= (n + 1)/2 se
n for ímpar. Temos assim um modo mais explícito de apr<)S<'ntar o
problema:
99

Um polígono regular com n vértices A 1 , A2 , ... , An está inscrito


numa circunferência de raio 1. Seja m = (n + 2)/2 se n for par, e
m = (n + 1)/2 se n for ímpar. Calcular a quantidad~ X . IA 1A 2l2 +
· · · + IA1Aml 2,
É um tanto inconveniente que a soma IA1A2l 2+ · · · + IA1Aml 2 seja
interrompida em Am, eni vez de ir até An, como seria natural. Mas
(como no Problema 2.6) podemos duplicar a soma; a simetria garante
que IA1Ail = IA1An+2-il, de modo que

X=~ (IA1A2l 2 + IA1A3I 2 + · · · + IA1Ami 2 + IA1Anl 2 + IA1An-1l 2 + · · ·

· · · + IA1An+2-ml 2) ·
. .
Notemos que, quando n é par, a parcela IA 1Aml 2 = 4, correspon-.
dente à diagonal, aparece duas vezes nesta soma. Podemos dar um
jeito nisto, e somar ainda a parcela IA1A 1I2 por razões de simetria,
obtendo então

(16)
100 Geometria analítica

quando n é ímpar, e

(17)

quando n é par (o 2 provém do termo extra IA 1 Aml 2 = 4 quando


multiplicado por ½). É natural introduzirmos agora a quantidade

(18)

e tentarmos calcular Y em vez de X. Há algumas vantagens neste


procedimento:

• Uma vez conhecido Y, as igualdades (16) e (17) nos dizem de


imediato quanto é X;

• A fórmula de Y é mais bonita que a de X, e por isso esperamos


que seja mais fácil de calcular;

• Quando calcularmos .Y, não teremos de separar o caso em que n


é par do caso ímpar, o que talvez nos poupe algum trabalho.

Podemos revisitar nossa, anterior tabela com os casos n = 2, 3, 4 e


6, e calcular Y nesses mesmos casos (usando (16) e (17), por exemplo):

n X y
2? 4? 4?
3 3 6
4 6 8
6 8 12
Da tabela podemos conjecturar que Y = 2n; isto implicaria, por
(16) e (17), que fosse X= n quando n é ímpar e X= n + 2 quando n é
par. É provável que seja esta a resposta certa, mas falta provar.
É altura de usarmo_s a geometria vetorial, já que ela nos fornece
alguns utensílios úteis para manipular expressões como (18). Dado qtw
101

o quadrado do comprimento de um vetor v não é mais que o produto


escalar v · v do vetor com ele próprio, podemos reescrever Y como

Y = (A1 - A1) · (A1 - A1J + (A1 - A2) · (A1 - A2) + · · ·


· · · + (A1 - An) · (A1 - An),

onde agora tratamos os Ai como vetores e não como pontos. Podemos


escolher a origem de nosso referencial onde bem quisermos, mas a
escolha mais lógica é colocá-la no centro da circunferência. (A segunda
melhor escolha seria pôr a origem em A 1 .) Uma vantagem imediata
de termos a origem no centro da circunferência é que todos os vetores
A1, ... , An têm comprimento 1, e portanto A1 · A1 = A2 · A2 = · · · =
An · An = l. Em particular, podemos usar a aritmética vetorial para
obter

e em seguida desenvolver a expressão de Y, o que dá

Agrupando os termos e simplificando, temos

Nossa conjectura foi de que Y = 2n. Conseguimos demonstrá-la se


mostrarmos que a soma vetorial A 1 + A 2 + · · · + An é zero. Mas isso é
claro por considerações de simetria. (Os vetores puxam em todas as
direções com a mesma força, e portanto a resultante das forças tem
que ser zero. Em alternativa, podemos alegar que o centróide de um
polígono regular coincide com seu centro. Devemos sempre tentar tirar
proveito da simetria.) Temos então que Y = 2n, de modo que X= .n
quando n é ímpar e X= n + 2 quando n é par.
102 Geometria analítica

Se não estivermos satisfeitos com a demonstração intuitiva de que


A 1 + · · · + An = O, podemos usar trigonometria ou números complexos
para dar uma prova mais concreta. Mas eis uma outra prova por
simetria que talvez seja mais satisfatória do que a anterior: seja v =
A1 + · · · + An. Façamos rodar tudo 360º /nem torno da origem. Isto
faz com que cada vértice avance para a posição do seguinte, mas não
altera a soma v = A 1 + · · · + An. Em outras palavras, quando v
é rodado 360º / n em torno da origem ele fica exatamente na mesma
posição. O único modo de isto acontecer é que seja v = O, e portanto
A1 + · · · + An = O, como queríamos mostrar.
O argumento precedente pode ser reinterpretado à luz da física.
De fato, a soma dos quadrados dos comprimentos dos vetores (i.e., Y)
é na prática o momento de inércia em A 1 ; e, com a ajuda do teorema
de Steiner sobre eixos paralelos (Borchardt, 1961, p. 370), podemos
depois deslocar Õ ponto de rotação para o centróide.

Exercício 5.1. (**) Demonstre que a.área da projeção ortogonal de


um cubo unitário em qualquer plano é igual ao comprimento de sua
projeção na perpendicular do mesmo plano .. (Sugestão: há uma re-
solução elegante com vetores, mas os diversos produtos a efetuar têm
que ser escolhidos com cautela. Fixe um bom sistema de coordenadas,
e selecione em seguida os vetores que sejam mais manejáveis. Escreva
o objetivo e depois manipule-o com produtos escalares e produtos ve-
toriais, tirando partido dos muitos pares de vetores ortogonais a sua
disposição. Explore também ·o fato de muitos vetores v tetem com-
primento 1 (ou seja, v · v = 1). Alcançada a solução, é boa ideia
reescrever a prova para apreciar como uma resolução vetorial pode ser
tão elegante em seu carácter abstrato.)

Problema 5.2. (*) É ·dada uma partição de um retângulo em retân-


gulos me°:ores. Cada um dos retângulos pequenos tem pelo menos
um lado de comprime?to inteiro. Mostre que o retângulo maior tem
também pelo menos um lado de comprimento inteiro.
103

Este problema tem um aspecto simpático, e por isso talvez tenha


uma solução igualmente simpática. Mas a conclusão é um pouco estra-
nha: se todos os retângulos pequenos têm um lado inteiro (ou até mais
do que um), por que é que o retângulo maior terá também. um? Se es-
tivéssemos lidando com segmentos de reta, e não com retângulos, seria
fácil: o segmento grande é composto por segmentos pequenos, e cada
um deles tem comprimento inteiro; logo, o comprimento do· segmento
grande é uma soma de inteiros e é por isso, evidentemente, um inteiro.
Este caso unidimensional não dá à primeira vista qualquer ajuda no
caso bidimensional, embora nos forneça a seguinte pista: temos que
usar o fato de que uma soma de inteiros é um inteiro. Um modo de
tirarmos partido imediato deste fato é munirmo-nos de uma terminolo-
gia conveniente: por lado inteiro queremos dizer lado. de comprimento
inteiro.
Mas o problema tem também pinceladas de topologia, combina~
tória e outras coisas piores, e isso se deve sobretudo à palavra parti-
ção, de uma generalidade excessiva. Para termos por onde pegar no
problema, consideremos primeiro uma partição (não trivial) o mais
simples possível, como esta:
104 Geometria analítica

Temos dois sub-retângulos, e sabemos que cada um deles tem pelo


menos um lado inteiro. Mas esses lados, pelo que sabemos, podem ser
horizontais ou verticais. Suponhamos que o sub-retângulo da esquerda
tem um lado vertical inteiro. Mas seu lado vertical coincide com o
lado vertical do retângulo grande, e por isso vemos que neste caso o
retângulo grande tem de fato um lado inteiro: Podemos então supor,
em vez disso, que o retângulo da esquerda tem o lado horizontal inteiro.
Mas, repetindo o raciotíri.io para o retângulo da direita, podemos
alegar que ele tem lado horizontal inteiro, e portanto o lado horizontal
do retângulo grande é também inteiro, por ser a soma de dois lados
inteiros. Resolvemos assim o problema para o caso especial de uma
partição com dois retângulos. Mas como é que isto funcionou? (Os
exemplos só têm alguma utilidade quando nos dão alguma pista de
como funciona o caso geral.) Inspecionando a demonstração em cima,
notamos dois ingredientes-chave:

(a) Temos que considerar casos, pois o lado inteiro de cada um dos
sub-retângulos tanto pode ser vertical como horizontal;

(b) O único modo de provarmos que o retângulo grande tem, diga-


mos, o lado vertical inteiro,. é existir uma cadeia de pequenos
retângulos, cad& um deles com lado vertical inteiro, cuja soma
seja d.e algum mÔdo igual ao retángulo grnnd<'. Na lig11rn 1·st.ít.
105

um exemplo onde os lados horizontais de todos os retângulos


sombreados são inteiros, e portanto o mesmo vale para o lado
horizontal do retângulo gr?nde:

Tendo então essa meta um pouco vaga em mente, podemos for-


mular uma estratégia igualmente vaga: Encontrar uma cadeia de re-
tângulos com lados horizontais inteiros (ou uma cadei•a de retângulos
com lados verticais inteiros) que possam ser de algum modo soma-
dos, fazendo assim com que o lado horizontal (ou o lado v~rtical) do
retângulo grande seja inteiro.
Mas temos que encontrar uma destas cadeias para cada possível
partição. As partições são coisas muito feias. E" cada retângulo pode
escolher como seu lado inteiro tanto o lado vertical quanto o horizontal.
Alguns até podem escolher ambos. Como podemos construir algo que
funcione com todas essas possibilidades?
E em qualquer caso, como é que funcionam essas cadeias? Se vários
pequenos retângulos têm o lado horizontal inteiro, o retângulo grande
terá também lado horizontal inteiro se, como acontece na figura atrásr
os pequenos formarem uma cadeia que una as duas extremidades do
grande, pois nesse caso o comprimento da aresta grande é a soma
dos comprimentos das menores. (Em outras palavras, se empilharmos
alguns tijolos, a altura total da pilha é a soma das alturas dos tijolos.)
106 Geometria analítica

Uma das dificuldades em encontrarmos essas cadeias vem de não


sabermos quais os retângulos que têm lado horizontal inteiro e quais
têm lado vertical inteiro. Para visualizarmos as possibilidades, ima-
gine-se que pintamos de verde cada retângulo com lado horizontal
inteiro, e de vermelho cada um com lado vertical inteiro. (Retângulos
com ambos os lados inteiros são um bónus: podem ser pintados de
qualquer uma das cores.) Cada sub-retângulo está então pintado de
verde ou de vermelho, e temos agora que encontrar uma cadeia verde
ligando as duas arestas verticais, ou uma vermelha ligando as duas
horizontais.
Uma demonstração direta parece ser inacessível, e por isso vamos
tentar a redução ao absurdo. Suponhamos que as duas arestas verti-
cais não estão ligadas por retângulos verdes. Por que razão elas não
podem ser conec~adas? Porque não há retângulos verdes que bastem:
os vermelhos devem ter bloqueado o caminho_. Mas o único modo de
evitar que os retângulos verdes vão de uma aresta a outra é que exista
uma barricada intranspon.ível de retângulos vermelhos. Uma barri-
cada assim intransponível terá que unir as duas arestas horizontais.
Portanto, ou os verdes fazem uma ponte entre as duas arestas verti-
cais, ou os vermelhos fazem outra entre as ~restas horizontais. (Quem
conhecer o jogo do Hex notará algumas semelhanças aqui.)
(Convém dizer que, embora o argumento do parágrafo anterior
seja muito intuitivo, uma prova formal exige algum trabalho e apela
a noções topológicas. Em suma: a região pintada de verde ,pode ser
dividida em subconjuntos conexos. Supondo que nenhum desses sub-
conjuntos vai de uma ponta à outra, consideremos a reunião da aresta
vertical esquerda corri todos os subconjuntos que tooam nela. Então
há uma faixa estreita, t_ocando esse conjunto por fora, que está pintada·
de vermelho, faixa essa que define uma cadeia de retângulos vermelhos
ligando o topo à base.)
Mas será verdade ,que a existência de uma cadeia de retângulos
verdes ligando as duas ·arestas verticais obriga a aresta horizo11t.al do
107

retângulo grande a ser inteira? Se a cadeia tiver retângulos supérfluos,


eles podem ser descartados; se dois retângulos se tocarem só num
canto, também não há problema; e o caso de a cadeia retroceder é
também fácil de tratar (nesse c·aso subtraímos inteiros em vez de os
somarmos). Mas, quando dois retângulos da cadeia têm em comum
parte de uma aresta, como na figura em baixo, seus lados não se somam
nem se subtraem:

Mesmo que os dois retângulos a sombreado tenham lados horizon-


tais inteiros, nada obriga a que o mesmo aconteça com -o retângulo
grande. Para evitar casos como este, reformulamos toda a nossa co-
loração do seguinte modo. Os interiores dos retângulos com lados ho-
rizontais inteiros são pintados de verde, e os interiores daqueles com
lados verticais inteiros são pintados de vermelho. O interior de cada
aresta (ou seja, a aresta sem as extremidades) é pintado de verde se
ela for vertical, e de vermelho se for hocizontal. Os cantos que ainda
não tenham sido coloridos (por não pertencerem ao interior de qual-
quer aresta) podem ser pintados com qualquer uma das cores. Deste
modo cada ponto do retângulo grande é agora verde.ou vermelho - e,
tal como antes, ou existe um caminho verde entre seus lados verticais,
ou um caminho vermelho entre seus lados horizontais. No primeiro
caso, os lados horizontais dos retângulos que constituem o caminho
podem ser somados (ou, às vezes, subtraídos) para concluirmos que o
lado horizontal do retângulo grande é inteiro; e coisa análoga se passa
no segundo caso quanto -aos lados verticais. Na figura em cima, por
108 Geometria analítica

exemplo, poderíamos ter o seguinte caminho vermelho entre os lados


horizontais do retângulo:

Problema 5.3 (Taylor, 1989, p. 8). É dado no plano um conjunto


finito de pontos de que não há três colineares. Alguns desses pontos
estão ligados por .segmentos de reta, mas cada um dos pontos per-
tence no máximo a um desses segmentos. Em seguida executamos
esta operação: tomamos dois desses segmentos que se intersetem, cha-
memos-lhes AB e CD, retii:amo-los e substituímo-los por AC e BD.
Será possível repetirmos a operação indefinidamente?

Em primeiro lugar, devemos nos assegu~ar de que esta operação


não conduz a nenhuma situação ambígua ou degenerada; particular-
mente, não queremos segmentos de comprimento nulo ou dois segmen-
tos que coincidam. Foi por isso que impusemos a condição de cada
um dos pontos pertencer no máximo a um dos segmentos. S~ja como
for, a verificação é fácil mas deve ser feita (pode ser coisa delicada!).
Depois de tentarmos alguns exemplos, parece plausível que a res-
posta seja não. Ao fim·de algum tempo, os segmentos.parecem todos
afastar-se para os cantos, deixando de se intersetar uns aos outros.
Isto é fácil de dizer em linguagem corrente, mas como traduzi-lo em
matemática?
Temos que arranjar. um jeito de dizer que esse afastamento dos
segmentos aumenta de cada· vez que repetimos a operação. Mas o
109

afastamento não pode aumentar indefinidamente porque o sistema só


admite um número finito de configurações. Em algum momento o afas-
tamento atingirá um máximo e o processo terá que terminar. (Quer
dizer, as coisas têm que parar quando não podem ir mais longe.)
Temos portanto duas tarefas:

(a) Encontrar alguma característica do sistema que possa ser ex-


pressa por um número. Pode ser o número de interseções, o
número de segmentos de reta, ou alguma espécie de pontuação
(como num jogo de dardos). Deve refletir o afastamento; ou
seja, deve ser maior quando todas as arestas estão espalhadas
no bordo da configuração;

(b) Esta característica deve crescer (ou então manter-se estacionária,


mas isso seria muito mais fraco) a cada vez que repetimos nossa
operação.

[Por exemplo, quem conheça o jogo Sprouts (em português, bro-


tos) pode recordar que, a cada vez que se faz uma jogada. (unindo
dois pontos com uma linha e acrescentando um terceiro ponto a meio
dessa linha), o número de saídas ainda não usadas (usar uma saída é
ligar uma aresta a um ponto; cada ponto permite um máximo de três
saídas) decresce uma unidade (duas são 'gastas com a nova aresta e
uma é criada pelo novo ponto). Isto mostra que o jogo não pode durar
indefinidamente, pois as saídas acabam por se esgotar.]
Temos que encontrar uma característica que satisfàça (a) e (b). A
solução não é única: há diversas características que satisfariam (a) e
(b). Mas só precisamos de uma. É melhor tentar adivinhar alguma
simples e esperar que funcione.
Tentemos primeiro as que são realmente simples. Que tal o número
de pontos? É algo que nunca se altera, e por isso não serve. Número
de segmentos falha pelo rriesmo motivo. Número de interseções pa-
110 Geometria analítica

rece promissor, mas esse número nem sempre decresce a cada jogada
(embora a longo prazo deva decrescer), como mostra a figura:

--t>
D
D-------- B

Um só ponto de interseção deu origem a três.


Afinal, o que ,é que decresce quando transformamos dois segmentos
concorrentes em dois que não se intersetam? .De algum modo os seg-
mentos ficam mais afastados um do o~tro. Nesta altura poderíamos
tentar algo como soma das ·distâncias entre os segmentos, mas esse
não é um conceito fácil. Dentro do mesmo espírito, talvez acabemos
por pensar na soma dos comprimentos dos segmentos: é que eles não
ficam só mais afastados, ficam também mais curtos. (A desigualdade
triangular - em qualquer triângulo cada lado é menor do que a soma
dos outros - mostra facilmente isso mesmo.) Isto significa que o com-
primento total dos segmentos deve decrescer após cada operação, e por
isso o processo não pode entrar em ciclo nem prolongar-se indefinida-
mente (pois há só um número finito de possíveis segmentos ligando
os vértices que fixamos desde o início). O problema está portanto
resolvido.
Visto que alteramos dois segmentos de reta de cada vez, qualquer
característica que considerássemos deveria depender dos segmentos
individualmente, e não de suas interseções ou de outras propriedades
semelhantes. Acontee~ que as propriedades individuais dos segmen-
tos de reta se resumem na realidade a três: comprimento, posição <~
111

orientação. A posição e a orientação não são características que pu-


déssemos usar, pois elas de fato não podem crescer nem decrescer: por
exemplo, é pouco provável que, após cada operação, a orientação total
(seja lá o que isso for) mude no sentido horário. Mas se assim fosse,
por que horário e não anti-horário? Não há uma distinção real entre
horário e anti-horário, mas há uma diferença bem precisa entre mais
comprido e mais curto. Tendo isso em mente, somos quase forçados a
usar a ideia do comprimento total.

Problema 5.4 (Taylor, 1989, p. 34, Q2). Um rapaz está no centro de


uma piscina quadrada, e o professor (que não sabe nadar) está junto
a um dos cantos. O professor corre a uma velocidade três vezes maior
do que aquela a que o rapaz nada, mas o rapaz corre mais depressa do
que o professor. Será que o rapaz pode fugir ao professor? (Suponha
que ambos têm uma capacidade de manobra ilimitada.)

Comecemos por desenhar uma figura e assinalar a~gun~ pontos:

A~---------~ª

o
X

D e

O rapaz parte então de O, e o professor parte de um dos cantos,


seja ele A. Podemos ainda escolher a unidade de medida de forma a
que os lados da piscina ténham comprimento 1.
112 Geometria analítica

Ora, para resolvemos este problema, convém primeiro que deci-


damos qual será a resposta. (Não podemos realmente procurar uma
solução sem sabermos do que estamos à procura.) As provas circuns-
tanciais são um pouco incertas: se o rapaz pudesse escapar, então ele
teria de seguir alguma estratégia ganhadora. Caso contrário, existi-
ria uma estratégia ganhadora para o professor, que conseguiria sem-
pre manobrar para interceptar o rapaz em todas as circunstâncias.
Matematicamente, esta última possibilidade é um pouco assustadora:
teríamos que encontrar uma estratégia que frustrasse todas as possí-
veis manobras do rapaz, que tem muitíssimas opções (já que se pode
movimentar em duas dimensões, ao passo que o professor está na prá-
tica limitado a uma). Mas, para não ficarmos dependentes de muitas
tentativas, a primeira opção é a melhor: temos que tentar, inteligen-
temente, adivinh~r uma estratégia, e depois provar que ela funciona.
É com certeza mais fácil provar que uma certa.estratégia funciona do
que provar que todas elas falham. Vamos por isso supor que o rapaz
consegue escapar. Parece ser essa a mais'fácil das duas opções: experi-
mentemos primeiro as alternativas mais acessíveis, com isso talvez nos
livremos de uma carga de trabalhos mais tarde. (Isto não é preguiça,
mas espírito prático. Desde que o trabalho seja bem feito, fazê-lo da
maneira mais fácil é sempre melhor do que da maneira mais difícil.)
O rapaz corre mais depressa do que o professor, e portanto conse-
gue escapar se atingir a borda da piscina sem ser interceptado. Logo,
seu primeiro objetivo é sair da piscina. Uma vez que estabelecemos
isso, a velocidade de corrida do rapaz é irrelevante.
Antes de tentarmos adivinhar uma estratégia, usemos um pouco de
senso comum para elirriinar estratégias más e destacar algumas táticas
promissoras. Em primeiro lugar, o jovem deve deslocar-se à velocidade
máxima: mesmo que haja alguma recôndita vantagem em desacelerar,
o professor pode neutralizá-la abrandando também. Argumento seme-
lhante mostra que para.r é inútil: o professor pode esperar quieto até
que o rapaz se mexa novamente. (Do ponto de vista do rapaz, um
113

impasse não é uma vitória.) Em segundo lugar, podemos supor que o


professor não é bobo, e se manterá colado à borda da piscina. (Para
que se afastar dela? Só se atrasaria.) Em terceiro lugar, como o rapaz
está tentando chegar rapidamente na borda (ou pelo menos mais rapi-
damente do que o professor), as linhas retas (que são os trajetos mais
curtos e por isso mais rápidos) devem ser parte da resposta, embora
o rapaz talvez possa tirar proveito de mudanças bruscas de direção.
Por último, a estratégia não pode ser completamente predeterminada,
devendo depender em parte do comportamento do professor: afinal, se
o professor souber que o rapaz dá umas voltas antes de finalmente se
dirigir ao canto B, então ele corre para lá e espera que o rapaz chegue
- como aconteceria se o rapaz seguisse um plano tão estupidamente
predeterminado.
Em suma: a melhor estratégia do rapaz envolve arranques em li-
nha reta à velocidade máxima; além disso, a estratégia deve ser sufi-
cientemente flexível para poder levar em conta o comportamento do
professor.
Tendo em mente estas diretrizes gerais, podemos ensaiar algumas
estratégias. É óbvio que o rapaz terá que se afastar do professor: ir
diretamente para A não é uma jogada inteligente. A resposta intuitiva
seria ir para C, que é o ponto mais afastado de A. O rapaz tem que
nadar um comprimento de -/2/2 ~ 0,707, enquanto o professor tem
que correr de A para B e daí para C, ou de A para D e daí para
C, totalizando dois comprimentos da piscina para atalhar o rapaz na
saída. Como o professor é três vezes mais veloz, quando ele tiver
chegado a C o rapaz só terá percorrido um comprimento de 2/3 ~
0,667.' Esta estratégia não funciona: o professor chega primeiro.
Fugir não basta; falta um pouco de astúcia. Afinal, as pessoas
que tentam sair de uma piscina dirigem-se para as arestas e não para
os cantos. Tentemos, por exemplo, rumar ao ponto médio M de BC.
Nesse caso o rapaz só precisa nadar um comprimento de· 1;2 = 0,5.
Mas o professor também rião precisa correr tanto: de A para B e daí
114 Geometria analítica

para M a distância é de 1,5. O professor, sendo três vezes mais veloz,


tem tempo certinho para agarrar o rapaz quando ele pula da piscina.
O professor quase deixa escapar o rapaz quando este se dirige per-
pendicularmente para a borda: se o professor demorasse mais uma
fração de tempo o rapaz escaparia. Isto sugere uma de duas coisas:

• a velocidade do professor é a mínima possível para conseguir


apanhar o rapaz;

• a velocidade do professor é a máxima possível que permite que


o rapaz escape.

Isto complica um pouco as coisas, pois a velocidade do professor


parece se equilibrar no fio da navalha. Se o professor fosse um pouqui-
nho mais lento, o .rapaz escaparia nadando perpendicularmente para
a borda. Se o professor fosse significativamente mais rápido, talvez
conseguisse copiar os movimentos do rapaz, deslocando-se no sentido
horário quando ele o fizesse; e assim por diante. O senso comum não
consegue chegar a uma conclusão; precisamos fazer alguns cálculos.
Se o rapaz se dirige para uma aresta, o professor tem que correr
sem parar para se manter a par dele. Em outras palavras, o rapaz
condiciona completamente os movimentos do professor com a simples
ameaça de se mover numa certa direção. Sermos capazes de, até certo
ponto, controlar os movimentos do inimigo é dispormos de uma arma
poderosa. Será possível usá-la?·
Suponhamos que o rapaz se dirige para o ponto médio M à velo-
cidade máxima. O professor não tem remédio senão correr para B e
depois para M. Se o professor retroceder, ou fizer qualquer outra coisa,
o rapaz pode prosseguir. e chegar a borda antes do professor. Mas o
rapaz não tem que fazer o caminho todo até M - a ameaça é suficiente
para lançar -o professor numa corrida. A conclusão é que o rapaz pode
forçar uma situação com.o a da figura, se nadar à velocidadP 111,í.xima
até um certo ponto intermediário X:
115

O X
X-

D e
O professor tem que estar no ponto Y (onde IAYI = 3IOXI): ele
não é rápido suficiente para ter ultrapassado Y no momento em que
o rapaz atinge X; e, caso ainda não tenha atingido Y, o rapaz pode
fazer o resto do percurso até M e assim escapar. Portanto, o professor
tem que ir até Y.
Temos agora que o rapaz está em X e o professor foi obrigado a
ir para Y. Haverá necessidade real de o rapaz continuar até M? A
ameaça de atingir M foi suficiente para pôr o professor onde ele está
agora, mas ameaça e conrretização são coisas diferentes. Agora que o
professor está encalhado na aresta AB, por que é que o rapaz não se
desvia abruptamente para CD, a aresta oposta? Só precisa percorrer
meia unidade para chegar lá - e, ao contrário do que sucedeu quando
o rapaz se dirigiu pela primeira vez para uma aresta, o professor está
mal posicionado. De fato, se a distância de X a M for superior a
1/4, mostra-se sem dificuldade que o professor está lorige demais para
apanhar o rapaz. O rapaz pode assim escapar facilmente.
Exercício 5.2. (*) Mostre que, se o professor correr a uma velocidade
seis vezes maior do que aquela a que o rapaz nada, então o rapaz não
pode escapar. (Sugestão: desenhe um quadrado imaginário de lado
1/6 centrado em O. Logo que o rapaz abandone esse quadrado, o
professor fica em posição de vantagem.)
116 Geometria analítica

Exercício 5.3. (**) Suponha que a piscina é circular em vez de qua-


drada. Nesse caso é claro que o rapaz pode escapar (basta nadar para
o ponto diametralmente oposto ao professor). Mas e se o professor for
mais veloz? Mais precisamente, qual é a velocidade mínima do pro-
fessor para garantir que apanha o rapaz? Este problema exige algum
lampejo de criatividade (ou um conhecimento de cálculo das varia-
ções) para encontrar um minorante (ou seja, planejar estratégias de
fuga para o rapaz) e para calcular um majorante (o que exige uma
sequência de movimentos sem falha por parte do professor). (Podería-
mos fazer a mesma pergunta para a piscina quadrada, mas o problema
ficaria ainda mais intricado do que no caso circular.)
CAPÍTULO 6

Exemplos variados

A matemática é às vezes considerada como uma entidade vasta, se-


melhante a uma árvore, çom grandes ramos que correspondem a suas
grandes divisões e que por sua vez se ramificam em assuntos especia-
lizados, até que chegamos às extremidades da árvore, onde brotam as
flores e os frutos.
Mas não é fácil arrumar toda a matemática em compartimentos
tão bem definidos: há regiões nebulosas de fronteir1;1 entre os diver-
sos ramos, e também pedaços de matemática que n~o encaixam em
nenhuma. das áreas clássicas.
.
Os próximos problemas são simples entretenimento, e não perten-
cem exatamente nem à teoria dos jogos, nem à combinatória, nem à
programação linear.

Problema 6.1 (Taylor, 1989, p. 25, Q5). Numa certa ·ilha há 13


camaleões cinzentos, 15 ca:maleões castanhos e 17 vermelhos. Se dois
118 Exemplos variados

camaleões de cores diferentes se encontram, mudam ambos para a ter-


ceira cor (por exemplo, um castanho e um cinzento mudariam ambos
para vermelho), mas não mudam de cor em mais nenhuma situação.
Será possível que em algum momento os camaleões fiquem todos da
mesma cor?
Esta questão tem limites vagos, por causa da expressão em algum
momento. Isto significa que temos que decidir se o conjunto de todas
as combinações de cores possíveis dos camaleões inclui alguma em que
todos tenham a mesma cor.
Heuristicamente, deveríamos primeiro tentar a possibilidade de a
resposta ser não. Se a resposta for sim, então deverá haver um proce-
dimento específico para alcançar este objetivo. Isso parece mais uma
questão de contas do que um problema matemático, e como a per-
gunta apareceu nwna competição matemática o mais plausível é que
não seja essa a resposta certa. Tentemos portanto provar o não.
Para fazermos isso, é provavelmente boa ideia sabermos quais as
configurações de cores que .podem ser alcançadas por este procedi-
mento e quais talvez não possam sê-lo. Logo que detectemos um pa-
drão, haverá algo bem definido para demonstrar. Como pudemos ver
nos capítulos anteriores, para resolver um problema em matemática
é em regra útil adivinhar um resultado intermédio, que implique a
conclusão mas não seja logicamente equivalente a ela. Embora, de
um ponto de vista lógico, estejamos substituindo uma meta por outra
mais forte, há a vantagem prática de termos um objetivo que está mais
próximo dos dados e que pode guiar nossos esforços numa direção mais
clara. E generalizar a conclusão tende a fornecer um bônus adicional,
que é o de descartarmos informação supérflua.
Como exemplo simples, suponhamos que, cm um tabuleiro de xa-
drez, temos um bispo num canto (os bispos movem-se na diagonal)
e queremos .mostrar que ele nunca pode mover-se para um canto ad-
jacente (qualquer um dos dois cantos que não lhe são opostos). Em
vez de demonstrarmos isso, poderíamos demonstrar a afirn1a<~;u1 111ais
119

geral de que o bispo terá de se manter sempre em casas da mesma


cor. (As casas do tabuleiro são alternadamente brancas e pretas.) Do
ponto de vista lógico, o que queremos provar agora é mais; mas agora é
muito fácil vermos como continuar (cada jogada do bispo mantém-no
num quadrado da mesma cor; portanto, por muitas jogadas que faça,
nunca mudará para a outra cor).
Comecemos então por fixar uma notação decente (com números
e equações). Em qualquer altura, os únicos dados importantes são o
número dos camaleões que são cinzentos, o número dos que são cas-
tanhos, e o número dos que são vermelhos. (Este modo de apresentar
o problema não permite aos camaleões tomar qualquer outra cor.)
Podemos representar de modo eficiente esta informação por meio de
um vetor tridimensional: o estado inicial é assim (13, 15, 17), e o pro-
blema é saber se, com a operação de mudança de cor, podemos atingir
(45, O, O), (O, 45, O) ou (O, O, 45). Essa operação consiste em subtrair 1
a duas das coordenadas e adicionar 2 à terceira. Temos assim uma
formulação vetorial que de fato nos fornece um modo de resolver o
problema.
(Para um esboço da _demonstração, pomos a = (-1, -1, 2), b =
(-1, 2, -1) e e= (2, -1, -1). O encontro de dois camaleões se traduz
pela soma de a, b ou e ao vetor da configuração atl).al. Assim, qualquer
configuração que o :;;istema possa atingi~ corresponde a um vetor da
forma (13, 15, l 7)+la+mb+nc, onde l, me n são inteiros. Basta então
mostrar que um vetor como (45, O, O) não pode ser representado dessa
forma, o que é fácil de fazer com a regra de Cramer ou por simples
manipulação diofantina.) ·
. .
Tentemos o método mais elegante que delineamos em cima: o de
determinar todas as possíveis combinações de cores dos camaleões.
Em primeiro lugar, o número total de camaleões mantém-se inalte-
rado. Isto não é de grande ajuda neste caso (embora, em questões
semelhantes, ter em conta o total da população possa ser boa ideia).
Em segundo lugar, dois camaleões de cores diferentes fundem-se numa
120 Exemplos variados

terceira cor. Podemos nos focar nesta fusão. Quando temos água em
dois recipientes desiguais ligados pela base, os dois níveis de água fun-
dem-se num mesmo nível intermédio. Mas a quantidade total de água
permanece constante. Fará sentido dizer que a quantidade total de
cor permanece constante?
Temos obviamente que definir quantidade total de cor para que isto
seja boa matemática. Tomemos o exemplo de um camaleão cinzento
e outro vermelho fundindo-se em dois camaleões castanhos. Se dis-
sermos, por exemplo, que cada camaleão cinzento vale O pontos, cada
castanho vale 1 ponto, e cada vermelho vale 2, então esta jogada pre-
serva a pontuação total (ou a cor total). (Um O e um 2 combinam-se
em dois 1's.) Mas isto já não funciona quando tentamos fundir um
camaleão vermelho com um castanho, por exemplo. Não parece haver
sistema de pontuação que sirva para as três possibilidades de fusão
(ou mesmo só para duas delas).
A dificuldade tem a ver com o caráter. cíclico das jogadas. Mas não
vamos desistir já. Uma terttátiva não completamente bem sucedida
pode vir a ser parte de urna abordagem de sucesso. (Mas também um
sucesso insignificante não é coisa que nos deva entusiasmar muito.)
Pensemos nas cores primárias: vermelho, azul e verde. Se um feixe
de luz vermelha coincidir com outro de luz verde, temos um feixe com
dupla intensidade de luz roxa, que é a cor oposta ao azul (ou anti-azul).
As cores primárias funcionam também em ciclos. Conseguiremos tirar
partido desta analogia com os feixes coloridos?
Bem, a diferença essencial é que, com a luz, vermelho e verde com-
binam-se não para dar azul mas sim anti-azul. Mas um momento!
Podemos fazer com que o azul e o anti-azul sejam iguais usando a
aritmética modular. Com isso em mente, podemos tentar reduzir nos-
sos vetores módulo 2: começamos na posição (1, 1, 1), e queremos ver
que daí não conseguimos chegar a (1, O, O), (O, 1, O) ou (O, O, 1). Infeliz-
mente, isto não funciona. M~s agora o gênio saiu da garrafa: podc~rnos
tentar outros módulos. Rapidamente pensamos no m<'>d11lo :1 (ali11al,
121

são três as cores em ciclo). Podemos optar agora por uma das duas
táticas seguintes para resolver o problema:

• Abordagem vetorial. Nosso vetor inicial, (13, 15, 17), converte-se


em (1, O, 2) (mod 3), e uma curta inspeção mostra que as mudan-
ças de cor podem levar-nos apenas aos vetores (1, O, 2), (O, 1, 2)
e (1, 2, O) - e portanto nunca chegaremos aos vetores (45, O, O),
(O, 45, O) ou (O, O, 45), todos eles iguais a (O, O, O) (mod 3);

• Abordagem da cor total. O método que tentamos para calcular


a quantidade total de cor foi o de atribuir uma certa pontuação
a cada cor. Agora que nos viramos para os módulos, por que não
atribuir uma pontuação modular? Dizemos que cada cinzento
vale O (mod 3) pontos, cada castanho vale 1 (mod 3), e cada
vermelho 2 (mod 3). Isto de fato funciona: a pontuação total tem
que se manter constante (porque nenhuma das possibilidades de
fusão vai alterá-la -- mas é melhor verificar); mas, enquanto a
pontuação total inicial é 13 x O+ 15 x 1 + 17 x 2 = 1 (mod 3),
todas as posições que pretendemos alcançar (45 cinzentos, 45
castanhos ou 45 vei:melhos) têm pontuação total O (mód 3).

Exercício 6.1. Seis músicos reúnem-se num festival de música. Em


cada um dos concertos tocam alguns des~es músicos, enquanto os ou-
tros se juntam à audiência para ouvir. Qual é o número mínimo de
concertos a ser programado para que cada músico -tenha oportuni-
dade de ouvir, como parte da audiência, cada um dos restantes cinco?
(Sugestão: é claro que num só concerto nem todos se conseguem ou-
vir uns aos outros, e por isso é preciso mais do que um concerto para
esgotar todas as possibilidades de audição. Se seguir esta linha de
ideias, e usar também a ideia da pontuação, consegue um minorante
razoável para o número de concertos necessários. Para terminar é só
encontrar um exemplo em .que esse mínimo seja alcançado.)
122 Exemplos variados

Exercício 6.2. Há três gafanhotos alinhados. Em cada segundo, um


(e só um) dos gafanhotos pula sobre um dos outros. Mostre que, após
1985 segundos, os gafanhotos não podem estar em posição igual à
inicial.

Exercício 6.3. Quatro peças de damas estão inicialmente situadas no


plano de modo que formam um quadrado de lado 1. Em cada jogada
pode pegar uma peça e fazê-la pular sobre outra, colocando-a à mesma
distância dessa outra mas no sentido oposto. Pode fazer um número
ilimitado de jogadas. Não há limite de distância entre duas peças
envolvidas numa jogada: por muito afastadas que estejam, uma delas
pode sempre pular sobre a outra. Será possível movimentar as peças
de modo que elas passem a formar um quadrado de lado 2? (Há uma
solução particularmente elegante, se pensarmos do modo correto.)

Problema 6.2. (*) Alice, Betty e Carol fizer~m as mesmas provas.


Em cada prova houve uma nota x, uma nota y e uma nota z, onde x,
y e z são inteiros positivos µistintos. Feitas todas as provas, a soma
das notas de Alice foi 20, a de Betty foi 10, e a de Carol foi 9. Se Betty
foi a primeira classificada em Álgebra, quem teve a segunda melhor
nota em Geometria?

Há pouquíssima informação neste problema: parece que sabemos


pouco mais do que as classificações totais. E como podemos determi-
nar as classificações parciais a partir das totais? Podemos sim, pois
nos são fornecidos outros dados. Para começar, de cada vez que houve
uma prova (e ainda não sabemos quantas houve) uma das moças foi
classificada com x, outra com y, e outra com z. Este é u.ma informação
de um tipo pouco usual. Como podemos explorá-la?
Em primeiro lugar, podemos tentar combiná-la com nosso terceiro
pedaço de informação, de que Betty foi a primeira em Álgebra. Isso
significa que Betty teve a mais alta das três notas possíveis :r, y ou
z. Para facilitar as coisas, supomos que x é a maior da:-; 1r{•:-; 1• :; a
123

menor: ou seja, x > y > z (lembremos que x, y e z são distintos).


Não perdemos muito, e ganhamos em simplicidade: podemos dizer
que Betty teve x em Álgebra. .
Mas, quanto às outras provas, ainda não sabemos muito sobre as
várias possibilidades. Por exemplo, Alice pode ter tido z, Betty x e
Carol y, ou talvez Alice tenha tido x, Betty y, Carol z. Há alguma
coisa que se mantenha fixa no meio de tantas possibilidades? Bem, a
soma das notas em cada prova é sempre a mesma. Seja lá como for
que se distribuam as notas x, y e z, a soma delas por prova é x+y+z.
Sabemos mais alguma coisa sobre essas classificações totais? Como a
soma das classificações em todas as provas é 20 + 10 + 9 = 39, obtemos

N-(x+y+z)=39,

onde N é o número de provas. Dispomos assim de uma fórmula que


envolve o número de provas, número esse de que antes pouco sabíamos.
Isto deve revelar-se útil.
Esta equação isolada parece não ser suficiente. Contudo, devemos
ter em mente que N, x, y e z são inteiros positivos e não números
reais quaisquer. E temo~ um outro pedaço de informação: x, y e z são
distintos.
Bom, uma vez que sabemos que N, x, y e z são inteiros positivos,
a equação em cima é da forma

(inteiro positivo) x (inteiro positivo) = 39,


e portanto N e x + y + z têm de ser divisores de 3!t Mas os únicos
fatores de 39 são 1, 3, 13 e 39, o que nos deixa quatro possibilidades:

(a) N = l ex+ y + z = 39;


(b) N = 3 e x + y + z = 13;
(c) N = 13 ex+ y + z = 3;
(d) N = 39 ex+ y + z = l.
124 Exemplos variados

Mas nem todas estas alternativas são viáveis. A hipótese (a), por
exemplo, diz que houve apenas uma prova. Isto contradiz o enunciado
do problema, que menciona pelo menos duas provas (Álgebra e Geo-
metria). E as hipóteses (c) e (d), além de contemplarem um número
suicida de provas, não funcionam se queremos que x, y e z sejam in-
teiros, positivos e distintos (o que obriga x + y + z a ser no mínimo
6). A única possibilidade que resta é (b): houve portanto três provas,
e a soma x + y + z é 13.
As possibilidades estão assim fortemente restringidas. Há duas coi-
sas que ainda não sabemos e que devem ser importantes: não sabemos
os valores exatos de x, y e z; e não sabemos que nota teve cada moça
em cada prova. A primeira pergunta é parcialmente respondida pelo
fato de x, y e z serem inteiros positivos distintos cuja soma é 13, e
quanto à segunda a resposta parcial que temos é que Betty teve x em
Álgebra. Como me.lhorar estas respostas parciais?
Uma informação de que ainda não tiramos todo o proveito é a das
classificações totais individuais.
.. Observando essas somas, vemos que
Alice teve resultados bastante melhores do que Betty e Carol, o que
deve implicar que ela teve boas notas (ou seja,· x ou y) em todas as
provas. Mas Betty teve a nota mais alta numa das provas, e portanto
Alice não pode só ter tido x's. No máximo teve dois x's e um y.
Analogamente, é pouco provável que Carol tenha tido a classificação
máxima x em alguma das provas, e é mais que provável que as notas
dela tenham sido sobretudo z's. Haverá como dar forma matemática
a estas especulações?
Para começar, a resposta é talvez. Consideremos, por exemplo, as
notas de Alice. Quando muito, elas somam 2x + y. Talvez possamos
provar que somam exatamente 2x + y: afinal, Alice tem um total
bastante superior aos dàs outras duas: 20 é muito mais do que 10
ou 9. Que outras possibilidades h~ para as notas de Alice? Eis a
lista delas: 2x + z, x + 2y, x + y + z, x + 2z, 3y, 2y + z, y + 2z e
3z. As últimas parece~ dar totais demasiado baixos para atingirem
125

20, e devem poder ser eliminadas. Mas, para tentarmos provar isto
rigorosamente, precisamos de majorantes razoáveis para x, y e z. Esta
é portanto nossa próxima tarefa: delimitar os valores de x, y e z para
que possamos eliminar várias das alternativas.
Tudo o que sabemos é que x, y, z são inteiros, que x + y + z = 13,
e que x > y > z. Mas isso é suficiente para delimitarmos bastante
bem os valores de x, y e z. Tratemos do z, por exemplo: ele não pode
subir muito alto, porque senão x e y teriam ainda de subir mais alto,
e com isso x + y + z provavelmente atingiria um valor superior a 13.
Concretizando, y é pelo menos z + l e x é pelo menos z + 2, de modo
que
13 = x + y + z ~ (z + 2) + (z + 1) + z = 3z + 3,
o que impõe z S 3. Esta majoração z S 3 é a melhor que conseguimos
sem usarmos informação adicional, pois há a possibilidade x = 6, y = 4
e z = 3.
Tentemos agora o y. Podemos fazer coisa semelhante a de cima,
minorando x por y + l, mas quanto a z só podemos dizer que é no
mínimo 1. Isso, porém, é suficiente:

13 = x + y+ z ~ (y + 1) + y + l = 2y + 2,
e daí y s 5. O exemplo x = 7, y = 5 e z = l mostra que isto é outra
vez o melhor possível. Por fim, podemos majorar o x: z é no mínimo
1 e y é no mínimo 2, de modo que 13 = x + y + z ~ x + 2 + 1, e
portanto x S 10. E isto é o melhor possível, pois poderia ser x = 10,
y=2ez=l.
Sabemos portanto que z S 3, y S 5 e x s 10. Mas podemos
fazer ainda melhor. Lembremos que Betty teve um x e mais duas
notas; como só totalizou 10, sabemos que x não pode chegar a 10.
Se chegasse, Betty teria tido zero nas outras duas provas, o que é
impossível: todas as classificações foram inteiras e positivas, como
sabemos. De fato, x nem pode chegar a 9, pois caso contrário nas
126 Exemplos variados

outras duas provas Betty só totalizaria 1: assim, teria tido zero numa
delas, o que é outra vez contraditório. Temos assim de fato x :S 8.
Está na hora de começarmos a eliminar possibilidades. Primeiro vemos
que as notas de Alice têm que ser 2x + y: todas as outras alternativas
ficam abaixo de 20. Por exemplo, 2x + z é no máximo 2 x 8 + 3 = 19.
De modo que Alice teve dois x's e um y. Uma vez que Betty
teve x em Álgebra, Alice teve y nessa prova. Podemos reunir toda a
informação já coligida numa tabela:
Exame Alice Betty Carol Total
Álgebra y X ? 13
Geometria X ? ? 13
Outro X ? ? 13
Total 20 10 9 39
Vemos agora que Carol deve ter tido z em Álgebra, pois essa é a
única nota que falta atribuir.
Estamos nos aproximando de nosso objetivo; sabemos que uma das
duas, Betty ou Carol, teve a segunda melhor nota, y, em Geometria.
Mas ainda não chegamos lá. Olhando para a colÚna de Alice na tabela,
a
há outra informação a usar, que é de ser y + x + x = 20. Recordando
que x > y e x :S 8, isso dá-nos só duas soluções: x = 8 e y = 4, ou
x = 7 e y = 6. Mas, como x + y + z = 13, não pode ser x = 7 e
y = 6, pois isso imporia z = O. Só podemos então ter x = 8 e y = 4,
o que impõe z = l. Este foi um grande avanço, pois determinamos
completamente os valores de x, y e z. Podemos então atualizar nossa
tabela:
Exame Alice Betty Carol Total
Álgebra 4 8 1 13
Geometria 8 ? ? 13
Outro 8 ? ? 13
Total 20 10 9 39
127

Agora vemos facilmente que Betty teve 1 tanto em Geometria como


na terceira prova, e que Carol teve 4 em cada uma dessas duas provas.
A resposta é, portanto, que foi Carol quem teve a segunda melhor
nota em Geometria.

Problema 6.3 (Taylor, 1989, p. 16, Q3). Duas pessoas participam


num jogo em que há uma barra de chocolate retangular de tamanho
6 x 10 constituída por 60 peças. O primeiro jogador quebra uma porção
do chocolate pelos sulcos que dividem as peças, e joga fora (ou come)
a parte que tirou. Depois o segundo tira igualmente uma porção do
que sobrou e joga-a fora (ou come-a). O jogo prossegue até que o
chocolate esteja reduzido a uma só peça. O vencedor é aquele que
deixa ao outro essa última peça (ou seja, é o último a jogar). Qual
dos dois jogadores tem uma estratégia ganhadora perfeita?

A propósito: é fácil mostrar que, em qualquer jogo finito, há sempre


uma estratégia ganhadora (ou que garanta o empate) para um dos
jogadores. Isto se faz por indução sobre o comprimento máximo do
jogo. Mesmo o xadrez tem essa restrição, embora ninguém tenha
ainda determinado tal estratégia, que a maioria das pessoa~ acredita
ser extremamente complicada. Como não há empates em nosso jogo,
um dos jogadores tem que ter uma estratégia ganhadora perfeita. Mas
qual deles?
Comecemos por transferir o problema do chocolate para a mate-
mática. Podemos começar por formalizar o processo de quebrar a
barra de chocolate. Qualquer pessoa que alguma vez tenha quebrado
um chocolate retangular sabe que o único modo de o fazer é em dois
retângulos, e não partindo-o por uma linha em ziguezague. Basica-
mente, o que fazemos é reduzir o retângulo 6 x 10 a um retângulo
menor em que um dos lados se mantém igual (veja-se a figura abaixo,
onde a linha tracejada é por onde o retângulo é quebrado); ou seja,
o novo retângulo tem a mesma altura e menor largura, ou a mesma
largura e menor altura. Na figura em baixo, por exemplo, a barra
128 Exemplos variados

6 x 10 fica reduzida a uma barra 6 x 7 (sendo a barra 6 x 3 comida ou


jogada fora).

1
1
1
1
1
1

1
1
1

1
1
1
1
1

.
1

1

1

Precisamos agora de alguma notação para: o retângulo, de pre-


ferência numérica. Como descrever um retângulo de chocolate com
números? Os candidatos óbvios são a altura e a largura, e por isso
nosso retângulo original é 6 x 10, ou talvez (6, 10) em notação veto-
rial. A posição do chocolate não é relevante; só o tamanho importa.
Nosso objetivo é conseguir deixar (1, 1) ao outro jogador. quais as
regras? Podemos tirar um tanto da altura ou um tanto da largura,
mas de modo que reste sempre um número positivo. Por exemplo, do
retângulo (6, 10) podemos passar às seguintes posições;
(6, 1), (6, 2), ... , (6, 9), (1, 10), (2, 10), ... , (5, 10)
Em suma, podemos mover-nos horizontalmente para a esquerda
ou vertical~ente para baixo. O próximo diagrama ilustra isso mesmo
em abstrato, mostrandq duas das posições que podem ser atingidas a
partir de (6, 10):
129

7
(3, 10)
(6,10)

(6,7)

(1, 1)

Agora que temos um modelo matemático manejável do chocolate,


podemos reformular o problema matematicamente (mas menos gulo-
samente) do seguinte modo:
Dois jogadores movem alternadamente um ponto num reticulado 1 ,
deslocando-o ou um número inteiro de passos para a esquerda, ou um
número inteiro de passos para baixo. O ponto não pode cruzar nenhum
dos eixos, e sua posição inicial é (6, 10). O vencedor é o jogador que
move o ponto para (1, 1). Qual dos dois tem uma estratégia ganhadora
perfeita?
Há ainda outra possível formulação:
Dois jogadores, jogando alternadamente, retiram- fichas dispostas
em duas filas. De cada-vez, cada jogador deve retirar peças de uma
só das filas. No início há cinco fichas na fila de cima e nove na fila de
baixo (isto representa o vetor (6, 10)). O vencedor é aquele que retira
a última ficha. Qual dos dois tem uma estratégia ganhadora perfeita?

1 N.
do T. Conjunto de tod?s os pontos de coordenadas inteiras no plano (rela'.-
tivamente a um certo sistema de eixos).
130 Exemplos variados

Nesta última formulação modificamos um pouco as coisas ao sub-


trair 1 tanto da fila de cima como da de baixo. Para quem conheça
o jogo do Nim, ela deve ser fortemente sugestiva: estas pessoas re-
solveriam agora o problema com facilidade. Mas podemos também
resolvê-lo sem qualquer conhecimento do Nim ou da teoria dos jogos
subjacente.
Temos agora notação e um modelo matemático abstrato. O que
nos falta agora é um bom domínio do problema. A dificuldade é
que o retângulo 6 x 10 nos oferece possibilidades demais. Devíamos
começar por experimentar o jogo com outro retângulo muito menor,
por exemplo o 2 x 3.
Nesse caso, o primeiro jogador pode deixar ao outro um dos seguin-
tes retângulos: 1 x 3, 2 x 2, 2 x 1. O 1 x 3 e o 2 x 1 são jogadas idiotas,
pois a seguir o outro jogador poderia retirar tudo exceto o último qua-
drado 1 x 1, e portanto ganhar. Logo, o primeiro jogador deve optar
por deixar um quadrado 2 x 2. Mas então o segundo jogador é forçado
a deixar um retângulo 1 x 2· ou 2 x 1, e depois o primeiro só tem que
quebrar esse retângulo a meio, deixando o quadrado 1 x 1 e ganhando
assim o jogo. Portanto, com um retângulo 2 x 3, é o primeiro jogador
que ganha.
Não ficamos sabendo muito mais, e por isso é melhor analisarmos
outro exemplo. Consideremos o retângulo 3 x 3: o primeiro jogador
tem várias opções: 1 x 3, 2 x 3, 3 x 2 e 3 x 1. Mas, por simetria, podemos
eliminar as duas últimas escolhas. 1 x 3 é uma escolha estúpida, pois o
segundo jogador pode retirar tudo exceto o último quadrado e portanto
ganhar. Mas 2 x 3 não é. menos má, pois então o problema fica reduzido
ao caso do último parágrafo. O segundo jogador só tem então que
usar a estratégia que o primeiro usaria no caso anterior: reduzir a um
quadrado 2 x 2, o que não permite outra escolha ao primeiro jogador
senão deixar um retângulo 1 x 2, que o segundo por sua vez reduz
a 1 x 1, assegurando a· vitórta. Portanto com um rctângu lo :~ x :~ o
primeiro jogador perde.
131

Resolvemos o caso 3 x 3 socorrendo-nos do caso 2 x 3. Isto sugere


uma abordagem indutiva para o caso geral. Suponhamos, por exem-
plo, que queremos resolver o ca.so 3 x 4, e que já sabíamos que nos
casos 3 x 1, 3 x 2, 1 x 4 e 2 x 4 é o primeiro a jogar quem vence,
enquanto no caso 3 x 3 é ele quem perde. Então a estratégia do pri-
meiro jogador no caso 3 x 4 seria a de deixar um quadrado 3 x 3 para
o segundo, que fica então em posição perdedora. Assim, a estratégia
para o primeiro jogador é deixar o segundo com retângulos que são
perdedores confirmados para quem os tem que quebrar. E por que
razão são estes retângulos perdedores? Porque, independentemente
de como os quebramos, eles se transformam em retângulos ganhado-
res para o outro jogador. E esses retângulos são ganhadores porque
podemos reduzi-los a retângulos perdedores para o outro jogador, e
assim sucessivamente. Nossa estratégia é então a de determinar todos
os retângulos ganhadores e todos os retângulos perdedores.
O retângulo 1 x 1 é um perdedor óbvio para quem o receba: como
não pode ser partido em outro menor, o jogo terminou. Todos os
retângulos 1 x n (onde n > 1) são ganhadores, pois quem os tiver
pode deixar ao outro o quadrado 1 x 1, que é perdedor.- 2 x 2 é
perdedor, porque quem o quebrar tem que deixar o retângulo 1 x 2,
que é ganhador. Portanto, todos os retângulos 2 x n (onde n > 2) são
ganhadores, pois podemos passar ao adversário o retângulo 2 x 2, que
é perdedor. E assim por diante. Notamos que:

• Se a x b for perdedor, então a x e (com e > b) é ~encedor, porque


quem receber a x e pode deixar a x b ao outro jogador. Por
exemplo, como mostramos que 3 x 3 é perdedor, concluímos que
3 x 4, 3 x 5, 3 x 6, etc. são todos retângulos ganhadores;

• a x b é perdedor se e só se todas as possíveis jogadas a partir


dele conduzirem a um retângulo ganhador. Por exemplo, 1 x 4',
2 x 4, 3 x 4 - e, pór simetria, 4 x 1, 4 x 2, 4 x 3 - são todos
132 Exemplos variados

ganhadores, como mostramos em cima, e por isso 4 x 4 tem que


ser perdedor.

Podemos prosseguir numa listagem sistemática até atingirmos o


retângulo 6x 10. Mas por que não fazê-lo dum modo mais matemático?
Deve haver algum padrão para retângulos ganhadores e perdedores.
Que retângulos desses já conhecemos? Os ganhadores já identificados
são os seguintes:
1X 2 1X 3 1X4 1X 5

2 X 1 2x3 2x4 2x5

3X 1 3x2 3x4 3x5

4 X1 4X2 4x3 4x5


Quanto a perdedores, temos já os retângulos 1 x 1, 2 x 2, 3 x 3 e
4 X 4.
Esta amostra é bastante convincente. de que os únicos perdedores
são os retângulos n x n - ou ·seja, os quadrados - e de que todos os
outros retângulos são ganhadores. Já que conjecturamos este resul-
tado, nem sequer temos que o demonstrar (embora isso possa ser feito
por indução); só temos que aplicar a estratégia correspondente. Lem-
bramos que o que queremos é deixar o oponente com um retângulo
perdedor. Uma vez adivinhados quais são os perdedores, a estratégia
em cada jogada é fazer com que o oponente fique com um deles. Se
a estratégia funcionar sempre, está tudo bem. Senão, é porque nosso
palpite estava errado. Em suma, se o palpite estiver certo, a melhor
estratégia é deixar um quadrado ao adversário. No caso do retângulo
6 x 10, o primeiro jogador seguirá então a seguinte estratégia:
Quebramos o chocolate de modo a deixar um quadrado 6 x 6 (que é
um perdedor). Em seguida, seja qual for a jogada do segundo jogador,
convertemos novamente o retângulo em um quadrado. Por exemplo,
se o segundo jogador dei~ar um retângulo 6 x 4, então devolvemos-lhe
o quadrado 4 x 4. Repetimos este procedimento, deixando s«·111pn· 11111
133

quadrado ao adversário, até que finalmente lhe entregamos o quadrado


1 x 1 (o que nos dá a vitória).
Esta estratégia funciona de fato, pois, sempre que o oponente que-
bra um quadrado, o que fica é um retângulo não quadrado que pode
ser facilmente reconvertido num quadrado. E, já que o tamanho do
chocolate decresce a cada jogada, esta conversão em quadrados tem
que produzir a certa altura o quadrado 1 .x 1. Assim, com um pouco
de matemática semirrigorosa, acabamos por conseguir uma estratégia
que funciona, e era só disso que estávamos à procura.
Seja como for, esta é a abordagem padrão em jogos de habilidade:
determinar todas as posições ganhadoras e perdedoras, e depois jo-
gar sempre para uma posição ganhadora. Qualquer jogador de nível
aceitável usa esse método, com a diferença de que em geral tem uma
noção vaga do que são posições ganhadoras ou perdedoras, e fala em
vez disso de posições favoráveis ou desfavoráveis. Afinal, não dizemos
que uma jogada em xadrez é boa ou má quando dá origem a uma
posição favorável ou desfavorável? .Jogar aleatoriamênte; sem procu-
rar melhorar sua posição no jogo, não é receita de sucesso para um
jogador de xadrez.

Exercício 6.4. Dois jogadores participam num jogo em que no início


há uma pilha de 153 fichas . .Jogando alternadamente, a cada jogada
um deles retira da· pilha entre uma e nove fichas. Ganha quem retirar
a última ficha. Algum dos jogadores tem uma estratégia garantida
para ganhar? Em caso afirmativo, qual deles?

Exercício 6.5. Dois jogadores participam num jogô em que de início


há uma pilha de n fichas. .Jogando alternadamente, a cada jogada
um deles retira da pilha um número de fichas que é uma potência
de d. Ganha quem retirar a última ficha. Para os valores seguintes
de d, determine os valores de n em que o primeiro jogador tem uma
estratégia ganhadora,. e os valores de n em que quem ganha é o segundo
jogador.
134 Exemplos variados

(a) d= 2;
(b) d= n;
(c) (*)d=4;
(d) (*) Caso geral.

Exercício 6.6. Repita o exercício anterior, mas agora com o objetivo


de perder, ou seja, de obrigar o outro jogador a retirar a última ficha.
(Se pensar nisto de modo correto, a resposta surgirá facilmente.)

Exercício 6.7. Imagine uma versão tridimensional do Problema 6.3,


em que de início há uma barra de medidas 3 x 6 x 10 que pode ser
encurtada em qualquer uma de suas dimensões. Qual dos jogadores
ganha? Qual é a estratégia ganhadora?

Exercício 6.8. (**) No jogo do Gomuku, os dois jogadores (Branco


e Preto) colocam, à vez, uma pedra da respectiva cor num tabuleiro
19 x 19. Ganha quem conseguir pôr primeiro cinco pedras alinhadas
(em qualquer direção); se ant"es disso o tabuleiro estiver completa-
mente preenchido, o jogo termina empatado. Mostre que o primeiro
jogador dispõe de uma estratégia que lhe garante pelo menos o empate.
(Sugestão: pode argumentar por redução ao ábsurdo, mostrando que,
se o primeiro jogador não puder sequer forçar um empate, então o
segundo dispõe de uma estratégia ganhadora. Em seguida mostre que
o primeiro jogador pode roubar essa estratégia.)

Problema 6.4 (Shklarksy et al., 1962, p. 9). Dois irmãos venderam


um rebanho de ovelhas. Cada ovelha foi vendida por um número
de rublos igual ao número de ovelhas no rebanho, e b total obtido
foi dividido do seguinte .modo: o irmão mais velho tirou 10 rublos,
depois o mais novo tirou 10 rublos, depois o mais velho tirou outros
10 rublos, e·assim sucessivamente. No final da divisão, o irmão mais
novo, a quem cabia a voo_, verificou que restavam menos de 1O rublos,
e portanto embolsou o qtw s·obrava. Para tornar a divisiio j11st.a, o
135

irmão mais velho deu ao mais novo seu canivete, que valia um número
inteiro de rublos. Quanto valia o canivete?

A provável reação inicial é que este problema não parece dar-nos


informação suficiente. Além disso, também não aparenta ser sufici-
entemente rigoroso. Mas seria errado desistirmos antes de fazermos
qualquer tentativa de resolução: lembremos o Problema 6.2, que no
início dava ainda menos informação do que este e mesmo assim pôde
ser resolvido.
Devemos começar por tentar formular o problema à custa de equa-
ções. Para isso precisamos de variáveis. Em primeiro lugar, observa-
mos que o preço do canivete vai acabar por depender do número de
ovelhas, e que este número é nossa única variável independente (ou
seja, se o conhecermos tudo o resto fica determinado). Suponhamos
que há x ovelhas. Cada uma delas foi vendida por x rublos, e portanto
a quantia reunida foi de x 2 rublos.
Estudemos em seguida como se processa a partÜha do dinheiro.
Suponhamos que o total de rublos era 64. O irmão mais velho tirou
10, depois o mais novo tirou 10, e assim por diante. Constatamos que
os últimos 4 rublos vão para o mais velho, e portanto este número 64
não serve ao problema. Lembremos que uma parte de nossos dados é
que quem fica com a última porção do dinheiro é o irmão mais novo.
Como podemos traduzir isto matematicamente?
Para fazer isso, precisamos de umas quantas equações e variáveis
(suficientes para descreverem a situação, mas não tantas que sejam
supérfluas
. ou causem confusão).
.
Suponhamos que o· irmão mais novo
retirou n lotes de 10 rublos antes de chegar ao lote incompleto. Então
o irmão mais velho também retirou n lotes de 10, e ainda um lotê
extra de 10 imediatamente antes de o dinheiro descer abaixo dos 10
rublos, ficando apenas a rublos (a é um inteiro entre 1 e 9, já que o
enunciado nos leva a crer que a não é zero). Portanto, o número total
de rublos terá que ser ·
136 Exemplos variados

x 2 = 10n + 10 + lün + a
ou
x 2 = 10(2n + 1) + a.
Mas o que isto tem a ver com o canivete? A variável dependente
que queremos calcular é p, o preço do canivete. Precisamos de uma
equação que relacione p com qualquer outra coisa, de preferência x, que
é a variável independente. Antes de entregar o canivete, o irmão mais
velho tinha embolsado lün + 10, contra lün +ado mais novo. Uma
vez concretizada a entrega, o lucro do irmão mais velho desceu para
lOn+lO-p, e o do mais novo subiu para lOn+a+p. Para que a divisão
seja equitativa, estas duas quantias têm que ser iguais. Igualando-as,
obtemos por fim a.desejada equação relacionando p com a:

a= 10- 2p. (19)

Podemos agora inserir este· resultado na equação anterior para ob-


ter uma equação que relaciona p com as outras _variáveis:

x2 = 10(2ri + 2) - 2p..
É destas equações que temos que tirar o valor de p. Parece que
não dispomos aqui de informação suficiente, pois não nos foram dados
os valores de x, n ou a. Como. fazer para limitar mais nossa~ possi-
bilidades? A grande dificuldade é que há incógnitas demais à solta.
Algumas delas podem ser eliminadas usando aritmética modular. Por
exemplo, se reduzirmos· a última equação módulo 20, obtemos

·x 2 = -2p (mod 20).

Com isto eliminamos n e ficamos mais perto de nosso objetivo de


calcular p, mas temos atnda que lidar com este incômodo :r:. Feliz-
mente, podemos tirar partido do fato de os quadrado:-; t.011ian•m, rnn
137

aritmética modular, um número restrito de valores diferentes; em mó-


dulo 20, esses valores são O, 1, 4, 5, 9 e 16. Em outras palavras,
temos
-2p = O, 1, 4, 5, 9 ou 16 (mod 20).

Daqui, notando que 2p tem que ser par, tiramos

p = O, 2, ou 8 (mod 10).

Temos assim uma igualdade para p, mas ainda não determinamos


exatamente seu valor. Tudo o que isto nos diz é que o canivete pode
valer O rublos, 2 rublos, 8 rublos, 10 rublos, 12 rublos ... Mas o cani-
vete não pode ser muito caro, certo? Afinal, o irmão mais novo só
ficaria prejudicado em 10 rublos ou menos. Seguindo este raciocínio,
acabamos por nos lembrar que p está não apenas relacionado com n
ex mas também com a, e que a toma valores entre 1 e 9. Por (19),
isto implica que O < p < 5, o que, conjugado com as outra-s condições
sobre p, estabelece para o canivete o preço de 2 rublos. (Notemos que
este argumento funciona mesmo que aceitemos que a possa ser zero.)
Curiosamente, apesar de haver informação suficiente para deter-
minar o preço do canivete, não a há para determinar o preço de cada
ovelha ou o número delas. De fato, o máximo que podemos dizer sobre
x é que x = ±4 (rriod 20); portanto o número de ovelhas pode ser 4,
16, 24, 36, 44, 56, ...
Com quebra-cabeças como este, precisamos de todos os dados em
que possamos pôr as mãos. O melhor a fazer é estendermos à nossa
frente toda a informaçã0 sobre o quebra-cabeça e anotar cada pedaço
dela separadamente, por exemplo assim:
(a) o número de rublos que foi partilhado era um quadrado perfeito;
(b) o irmão mais novo foi prejudicado em seu quinhão;
(c) a diferença foi compensada pelo canivete.
Em seguida devemos de imediato traduzir estes fatos em equações:
138 Exemplos variados

(a) x 2 = 10(2n + 1) + a;
(b) O< a< 10;
(c) a= 10 - 2p.
Devemos tentar fisgar cada pedaço de informação, por mais que
ele nos pareça inútil. Por exemplo, eu poderia ter acrescentado que n
era provavelmente não-negativo, ou que pera provavelmente positivo
(para que mencionar o canivete na questão se ele não tivesse qualquer
valor?), ou que o número de ovelhas era inteiro, e assim por diante.
Logo que tudo esteja encapsulado em equações, as coisas ficam muito
mais fáceis de manipular corretamente.
Referências Bibliográficas

Os livros, tal como os amigos, devem ser poucos e bem escolhidos.


Samuel Paterson, Joineriana

[1] AMOC (Australian Mathematical Olympiad Commitee)


Correspondence Program (1986-1987) - 1. série de problemas.

[2] Australian Mathematics Competition (1984) -- Mathematical


Olympiads: The 1984 Australian Scene, Canberra College of
.
Advanced Education, Belconnen, ACT .

[3] Australian Mathematics Competition (1987) - Mathematical


Olympiads: The 1987 Australian Scene, Canberra College of
Advanced Education, Belconnen, ACT.

[4] Borchardt, W. G. (1961) - A Sound Course in Mechanics,


Rivingston, London.

[5] Greitzer, S. L. (1978) - International Mathematics Olympiads


1959-1977 (New Mathematical Library 27), Mathemàtical Asso-
ciation of America, Washington, DC.
140 Referências

[6] Hajós, G.; Neukomm, G.; Surányi, J. (editores) (1963) - Hunga-


rian Problem Book I, based on the Eõtvõs Competitions
1894-1905 (New Mathematical Library 11), Mathematical As-
sociation of America, Washington, DC.

[7] Hardy, G. H. (1975) - A course of Pure Mathematics, 10. edição,


Cambridge University Press, Cambridge.

[8] Polya, G. (1957) - How to solve it, 2. edição, Princeton Univer-


sity Press, Princeton.

[9] Shklarsky, D. O.; Chentzov, N. N; Yaglom, I. M. (1962) - The


USSR Olympiad Problem Book: Selected Problems and Theo-
rems of Elementary Mathematics, W. H. Freeman and Company,
San Francisco, CA.

[10] Taylor, P. J. (1989), International Matheniatics: Tournament of


the Towns, Questions an_d Solutions, Tournament 6 to 10 (1984
to 1988) - Australian Mathematics Foundation Ltd, Belconnen,
ACT.

[11] Thomas, G. B.; Finney, R. L. (1988); Calculus and Analytic


Geometry, Addison-Wesley, Reading, MA.
Índice Remissivo

2, potências de, 20-28 prova duma igualdade, 79, 92-


9, múltiplos de, 17-19 94
18, múltiplos de, 18, 18 antissimetria, 42, 45, 45
área de um triângulo, 2, 80
absurdo, demonstração por, 90- aritmética modular, Í4, 15
91, 106-107 equações diofantinas, 29-33
adivinhar, 20, 112-115 potências de 2, 26, .47
algarismos quadrados, 136
potências de 2, ??-27 somas de inversos, 40
reordenamento de, 20-28 somas de potências, 36
soma de, 16-20 vetores, 120
álgebra, 49
Bernoulli, polinômios de, 35
polinômios, 58-66
bolso, matemática de, 51
problema das notas na prova,
122-126 cadeias em partições de retângu-
análise de funções, 50-57 los, 104
ângulos calcule ... , problemas do tipo, 2
de triângulos, 69- 71 camaleões, combinações de cores
em circunferências; 68-70 em, 117-120
notação, 4, .70 cancelamento aos pares, 35, 37
142 ÍNDICE REMISSIVO

casos especiais, 6-7 indução forte, 56


em problemas de geometria, desigualdade triangular, 5, 110
80 desigualdades
chocolate, problema de partilha em análise de funções, 51-57
do, 127-133 em geometria euclidiana, 90
cíclicas, jogadas, 120 diagonais, comprimentos das, 96-
cíclicos, quadriláteros, 79 102
circunferências, ângulos em, 68- diagramas, 4
70 diâmetro, ângulos subtendidos por
circunferências, teoremas sobre, um, 68
68, 78, 92 diofantinas, equações, 29-33
concorrência das mediatrizes de divisibilidade
um triângulo, xiv somas de inversos, 38-46
conjecturas, 6 somas de potências, 33-38
consequências, demonstração de, divisão de bens e o preço do ca-
6 nivete, 135-137
construções, 81-84
elegância das resoluções, xiv
coordenadas em geometria, x1v,
eliminação-de variáveis, 84
67, 80
encontre ... , problemas do tipo, 2
uso em construções, 81
equações, uso de, 4-6
cordas, ângulos subtendidos por,
estabelecer resultados, 9
70, 92
existe ou não ... , problemas do tipo,
cossenos, lei dos, 5, 72 2 ..
cubos, soma de, 50 expoente, variáveis em, 31-33

dados fatores, 123


anotá-los, 4 fatores de polinômíos, 59, 62-66
entendê-los, 3 raízes de, 59
omiti-los de um problema, 8 fatoração, método da, 31, 62
demonstração por absurdo, 90- ·Fermat, último teorema de, xviii,
91, 106-107 31, 33
demonstração por indução, 53; 55 fórmula n~solwnt.<•, 1ll. :m. :,!), X1
ÍNDICE REMISSIVO 143

fórmulas, uso de, 4-6 e polinômios, 60-62


funções somas de, 38-46
análise, 50-57 inverter um problema, 8
polinômios, 58-59
jogos de habilidade, 133
generalização, 7, 22
geometria analítica Lagrange, teorema de, 13
aritmética vetorial, 100-102
partições de retângulos, 102- matrizes, álgebra das, 50
108 (mod n), notação, 15
problema da piscina, 111-115 modificação de problemas, 2, 6-9
segmentos de reta, 108-111 mostre que ... , problemas do tipo,
geometria euclidiana, xiv, 67-68 2
ângulos de triângulos, 69- 71 múltiplos de 9, 17-19
ângulos em circunferências, 68- múltiplos de 18, 18-18
70
Nim, 130
compararar ângulos, 92-94
níveis de dificuldade, xiv
construções, 81-84
notação, 4
quadrados e retângulos, 86-
vetorial, 119, 128
91
numeradores reduzidos, 38-39
quocientes, 75-78
números naturais, 15
grau de um polinômio, 58 números primos, 15
graus de liberdade de polinômios,
objetivos de um problema, 2
64

Heron, fórmula de, 5, 9 . p-ádicos, 14


parametrizações, 61
indução, prova por, 53, 55 partições de um retângulo, 102-·
indutiva, abordagem, 131 108
inteiros, retângulos de lados, 102- passos na resolução de problemas,
108 1 .
inversos periodicidade, 34-35
144 ÍNDICE REMISSIVO

piscina quadrada, problema da, ana, 86-91


111-115 quadriláteros, pontos médios dos
Pitágoras, teorema de, 78, 80, 96 lados de, 68
polígonos, comprimentos das di- quocientes em geometria euclidi-
agonais em, 96-101 ana, 75-78
polinômios, 58-59
constantes, 58 raízes de um polinômio, 59
cúbicos, 58 raízes dos fatores de um polinô-
fatoração de, 59, 62-66 mio, 59
homogêneos, 58 raízes quadradas, 30
irredutíveis, 59 retângulos
triviais, 58 e partilha de chocolates, 127-
potências de 2, 20-28 133
reordenamentos dos algaris- num quadrado, 86-91
mos, 20-28 partição de, 102-108
somas dos algarismos, 23-26 reta~ paralelas, 80-84
potências, somas de, 33-38 reformulação de um problema, 6
primos, 15 reordenam~nto de algarismos, 20-
primos entre si, 15 28
problemas finitos, simplificação de, representação dos dados e obje-
19 tivos, 4
produtos indeterminados, 40 restrições físicas, 1, 5
progressão aritmética, lados do rotações, 82-84
triângulo em, 2-11
prova, problema das notas na, 122- segmentos de reta, 108-111
126 senos, lei dos, 5, 72- ??
pseudo-coordenadas em geometria, simetria, 45
80 simplificação de problemas, 6
somas de algarismos, 16-20
quadrados e· aritmética modular, ·somas de cubos, 50
136 somas de inversos, 38-46
quadrados em geometria euclidi- somas de potências, :ti-:JH
ÍNDICE REMISSIVO 145

somas dos comprimentos de seg- problema do chocolate, 128-


mentos de reta, 110 130
Steiner, teorema de, 102
Wilson, teorema de, 14
Tales, teorema de, 68, 93
tangentes a uma circunferência, xadrez, problema do tabuleiro de,
78, 92 118
teoria dos números, 13-15
equações diofantinas, 29-33
reordenamento de algarismos,
20-28
soma de algarismos, 16-20
somas de inversos, 38-46
somas de potências, 33-38
triângulos
ângulos de, 69-71
áreas de, 2, 80
comprimentos dos lados em
progressão aritmética, 2-
11
equiláteros, 7, 81-82
isósceles, 68, 71
mediatrizes dos _lados são con-
correntes, xiv
semelhantes, 77, 78
trigonometria, 72-74

variáveis, 4
eliminação de, 84
em expoente, 31-33
vetores
cores dos camaleões, 119-121
em geometria, 100-102

Você também pode gostar